Res:[obm-l] QUESTÃO ANTIGA

2008-03-19 Por tôpico JoaoCarlos_Junior
1/a + 1/b + 1/c  = 1  bc/abc + ac/abc + ab/abc = 1 a, b, c são naturais distintos de zero. Logo, abc é maior que qualquer dos elementos de {ab, ac, bc, a, b, c}.Pensemos em um retângulo, cuja base tem uma unidade de medida. Dividamo-lo(a) em colunas iguais de largura 1/abc. As bc primeiras colunas são o primeiro termo da expressão matemática acima; as ac colunas seguintes, o segundo termo, e, por último, ter-se-á as ab últimas colunas, o terceiro termo. Ora, isso é sempre possível de ser feito com quaisquer inteiros distintos entre si, e até mesmo se são eles iguais dois a dois ou os três simultaneamente. Fraternalmente, João.[EMAIL PROTECTED] escreveu: -Para: "obm-l" De: "gugolplexj" <[EMAIL PROTECTED]>Enviado por: [EMAIL PROTECTED]Data: 18/03/2008 15:50Assunto: [obm-l] QUESTÃO ANTIGAOlá,  Creio q esse problema já transitou por aqui há algum tempo.  "Quais os números naturais a, b, c diferentes entre sí, tais que  1/a +1/b + 1/c = 1?"  Grato, Jorge. Instruções para entrar na lista, sair da lista e usar a lista em
http://www.mat.puc-rio.br/~obmlistas/obm-l.html
===

Res:[obm-l] Cadastramento OBM-2008

2008-03-18 Por tôpico JoaoCarlos_Junior
Prezada Secretária da OBM, Demais competentes a me respoder: Caso a escola de meu filho não se cadastre, existe alguma forma possível pela qual ele possa participar? Fraternalmente, João.[EMAIL PROTECTED] escreveu: -Para: [EMAIL PROTECTED], Lista de discussao De: Olimpiada Brasileira de Matematica <[EMAIL PROTECTED]>Enviado por: [EMAIL PROTECTED]Data: 17/03/2008 9:18Assunto: [obm-l] Cadastramento OBM-2008Caros sócios e amigos da OBM,Estão abertas as inscrições para participar da OBM-2008.Todas as escolas das redes pública e privada podem participar.Período de cadastramento:15 de março até  5 de maio de 2008apenas no site da OBM www.obm.org.brImportante:  Nosso sistema de cadastramento mudou, por favor, se a sua escolajá participou da competição em anos anteriores, também deve realizar o novocadastramento em 2008.Cordialmente,Secretaria da OBM=Instruções para entrar na lista, sair da lista e usar a lista emhttp://www.mat.puc-rio.br/~obmlistas/obm-l.html=Instruções para entrar na lista, sair da lista e usar a lista em
http://www.mat.puc-rio.br/~obmlistas/obm-l.html
===

Re: [obm-l] Re: [obm-l] [obm-l] questão da OBM

2007-12-17 Por tôpico JoaoCarlos_Junior
Prezado Carlos Shine:
Por obséquio, posta novamente a tua solução, pois não a tenho.
Fraternalmente, João.Ah, no e-mail anterior eu esqueci de provar que os primos de a^29 + 1 e a + 1 não se cortam todos. Mas é só mais um trabalho de mdc (análogo ao anterior) provar que mdc(a + 1; (a^29 + 1)/(a + 1)) divide 29 e ver que a^29+1 é muito maior do que a+1 para mostrar que não se cortam todos. Nesse último mdc, eu admito que usar congruência é mais rápido; todavia, não é absolutamente necessário. []'sShine- Original Message From: vitoriogauss <[EMAIL PROTECTED]>To: obm-l Sent: Friday, December 14, 2007 1:34:07 PMSubject: [obm-l] [obm-l] questão da OBM> Colegas > > A respeito da questão (a^29 - 1)/a-1... para provar que há 2007 fatores primos só por congruência??? > Grato > Vitório Gauss



Be a better friend, newshound, and know-it-all with Yahoo! Mobile. Try it now.Instruções para entrar na lista, sair da lista e usar a lista em
http://www.mat.puc-rio.br/~obmlistas/obm-l.html
===

Re: [obm-l] probabilidade (OFF)

2007-12-05 Por tôpico JoaoCarlos_Junior
É isso aí. Ralph, obrigado! 
Matemática é vida, sem emoção, ela não existe, é morta, e ficamos também amortalhados. 
Põe tua luz para fora, Ralph, auxilinado-nos no despertar da nossa.
Fraternalmente, João. 
Desculpa, Pedro, mas os eventos que você escolheu contar não são igualmente prováveis! É tão provável ter 0,0,0,10 bolinhas de cada cor quanto 3,3,3,1? Não, o segundo evento é bem mais provável! Isso dito, sua contagem combinatória está muito bacana -- a gente tem que **inventar** agora um problema cuja solução seja a sua, de tão bonita que ela é Será que tem alguma maneira natural de sortear soluções da equação x+y+z+t=10 de maneira que todas sejam igualmente prováveis?    Por outro lado, este erro é o mais comum (e sutil!) que há em probabilidade (basta ver os últimos 100 problemas de probabilidade aqui mesmo na lista -- 85.34% deles tem esse erro ;p ;p ;p). A culpa é de nós professores e nossos livros-texto, que marretam na cabeça a fórmula:  PROBABILIDADE = # DE CASOS FAVORÁVEIS / # DE CASOS POSSÍVEIS assim, com letras garrafais, e aí colocam em letras pequeninas no cantinho do rodapé do apêndice que isto só vale se os eventos forem igualmente prováveis. Aí a gente acostuma a contar # de casos para lá e para cá (hábito reforçado por centenas de problemas de contagem combinatória) e erra um monte de problemas de probabilidade apesar de fazer um monte de contas complicadas... Eu estou numa cruzada contra esta apresentação da fórmula acima por este motivo. :) :) :)  Então fica assim: todo mundo, junto comigo: Probabilidade = casos possíveis/casos favoráveis (em fonte pequenininina)APENAS QUANDO OS CASOS CONTADOS FOREM ABSOLUTAMENTE IRREFUTAVELMENTE SEM DÚVIDA TOTALMENTE IGUALMENTE PROVÁVEIS (EM FONTE COLOSSÁÁL)(quando der aula disso, repita a última frase com o entusiasmo com que o cara do Rock Gol diz CLÉÉRSON!!! -- ou sei lá que nome ele diz :) :) :) :) )   Abraço,   Ralph On 12/4/07, Pedro Cardoso <[EMAIL PROTECTED]> wrote:Cmoraes, eu recomendo que você escreve no google "soluções inteiras não-negativas". I) Depende do número de bolinhas. Se houver mais de 9 bolinhas de cada cor, tudo bem. Caso contrário, fica mais complicado, eu acho. Supondo que sejam mais de 9 de cada cor...  Sejam x1 o número de bolinhas verdes, x2  o número de amarelas, x3 o de azuis, x4 o de brancas. P(A) é a probabilidade de ocorrer o evento A. Além disso, o sinal '>=" significa 'maior ou igual'.  x1+x2+x3+x4 = 10  O número de soluções inteiras não-negativas dessa equação corresponde ao número de casos possíveis para os grupos de 10 bolinhas (desde que a ordem das bolinhas não importe). Casos possíveis = 13!/(3!10!) = 260.  P(não haver quatro cores) = 1 - P(haver quatro cores) Para que hajam quatro cores, devemos ter x1,x2,x3,x4>0. Considere y um inteiro maior ou igual a 0.Assim, satisfazendo as condições do problema, x1 = y1+1; x2 = y2+1; x3 = y3+1; x4 = y4+1  Como x1+x2+x3+x4 = 10, (y1+1)+(y2+1)+(y3+1)+(y4+1) = 10 .:. y1+y2+y3+y4 = 6 (y>=0) Os casos favoráveis são as soluções dessa última equação.Número de casos favoráveis = 9!/(3!6!) = 84. P(haver quatro cores) = 84/260 = 21/65 P(não haver quatro cores) = 1 - 21/65 = 44/65. O II é parecido, então, entendendo o I, acho que você consegue resolvê-lo. *Eu considerei x>=10 porque, caso contrário, na equação x1+x2+x3+x4 =10, eu teria que trabalhar com vários casos. A solução x1 =10, x2,x3,x4 = 0 não valeria, por exemplo.  Até. Espero ter ajudado.



Conheça o Windows Live Spaces, o site de relacionamentos do Messenger! Crie já o seu!Instruções para entrar na lista, sair da lista e usar a lista em
http://www.mat.puc-rio.br/~obmlistas/obm-l.html
===

Re: [obm-l] Ajuda em geometria espacial

2007-11-06 Por tôpico JoaoCarlos_Junior
Prezado e nobre Paulo Santa Rita:Então, por minha falta de base, pelo que entendi, atirei num gato e atingi o rato. E ainda, é capaz do rato não ter morrido. (sorriso) É isso?! (sorriso mais largo, sorriso maior... sons de riso). Bem, o que posso dizer?...O tetraedro possui como faces 4 triângulos equiláteros é isso? Mas, (já com ar de interrogação) , a inquirição não fala em ângulos retos? A tentativa que fiz é subconjunto ou está contida na que resolve o problema efetivamente apresentado?Qual o grau de validade de que escrevi? Fraternalmente, João. [EMAIL PROTECTED] escreveu: -Para: obm-l@mat.puc-rio.brDe: "Paulo Santa Rita" <[EMAIL PROTECTED]>Enviado por: [EMAIL PROTECTED]Data: 03/11/2007 8:30Assunto: Re: [obm-l] Ajuda em geometria espacialOla Joao Carlos e demaiscolegas desta lista ... OBM-L,Eu vou apresentar apenas um esboco de solucao para 2). Voce completaos detalhes. Antes acho valido registrar que TETRAEDRO e o nome dopoliedro regular, vale dizer, aquele solido com todos os lados e facesiguais. Acredito que voce esta se referindo a uma piramide triangularcujo vertice e S e cuja base e o triangulo ABC.IMAGINE o vertice S na origem de um sistema de coordenadas cartesianasXYZ. IMAGINE um plano alfa que corta os eixos coordenados nos pontos(p,0,0), (0,q,0) e (0,0,r) onde "p", "q" e "r" sao reais positivos. Eobvio que variando inteligentemente "p", "q" e "r" teremos todas aspiramides possiveis.Sabemos que a equacao deste plano e : (X/p) + (Y/q) + (Z/r) = 1.Tambem sabemos que o vetor V=(1/p,1/q,1/r) e perpendicular ao plano.Pronto. Ta tudo ai para provar ou refutar a conjectura.A) ENCONTRAR O ORTOCENTROE facil ...  sabemos que as coordenadas do BARICENTRO sao dadas pelamedia aritmetica das coordenadas dos vertices. Assim, o baricentro eD=(p/3,q/3,r/3). Encontramos agora o CIRCUNCENTRO. E o ponto deencontro das mediacrizes. Seja A=(p,0,0) , B=(0,q,0) e C=(0,0,r). Oponto medio de AB e (p/2,q/2,0), o ponto medio de BC e (0,p/2,r/2) e oponto medio de AC e (p/2,0,r/2). Tracamos pelo ponto medio de AB umareta perpendicular ao vetor V. Ela estara inteiramente contida noplano alfa. Sua equacao sera :(X - p/2, Y - q/2, Z)*(1/p,1/q,1/r) = 0          EQUACAO (1)Repetindo o mesmo raciocinio para os outros dois pontos medio ( de BCe de AC ) teremos duas outra equacoes, nomeadamente EQUACAO (2) EEQUACAO (3). A solucao do sistema formado por estas tres equacoes e ocircuncentro. Seja E este circuncentro.Sabemos que que o CIRCUNCENTRO, o BARICENTRO e o ORTOCENTRO estaoalinhados ( e a reta de Euler ) e que o baricentro divide internamenteo segmento formado pelo circuncentro e o ortocentro na razao 1/2.Assim, se F e o ortocentro, teremos :F - D = -2*(F - E)B) VERIFICANDO A CONJECTURACom a equacao acima o ortocentro fica determinado. Agora, trace poreste ponto ( o ortocentro que acabamos de calcular ) uma retaperpendicular ao plano alfa. Se a sua conjectura estiver correta estareta deve conter a origem, isto e, o ponto (0,0,0). E acabou.Como falei, fiz apenas um esboco. Uma solucao detalhada vai ficargrandona e eu nao tenho tanto tempo assim.O item b) e trivial. Tambem vou apenas esbocar uma solucao. Para vercomo e possivelisso, basta ver que SAB, SBC e SAC sao traingulosretangulos,. Logo, podemos aplicar o teorema de pitagoras. Suponha,por exemplo, que AB^2 >= BC^2 + AC^2. Usando os REAIS POSITIVOS "p","q" e "r" ja defionidos, teriamos  : p^2 + q^2 >= q^2 + r^2 + p^2 +r^2  =>2*(r^2) =< 0 ... absurdo ! Assim, o quadrado de nenhum lado pode sermaior ou igual a soma dos quadrados dos outros dois, ou seja, otriangulo e acutangulo.Um Abraco a todosPaulo Santa Rita7,0A19,030B07> Amigos estou precisando resolver os seguintes problemas:>> 1) Enunciar os casos de congruência de tetraedros, fazendo uma> correspondência com os casos análogos de congruência de triângulos, mas> ressaltando as diferenças nos dois casos.>> 2) Mostrar que se o tetraedro SABC tem faces formando ângulos retos no> vértice S, isto é, os ângulos ASB, BSC e CSA são retos, então:>> a) A reta SO, ligando o vértice S ao ortocentro do triângulo ABC, é> perpendicular ao plano ABC.>> b) O triângulo ABC é acutângulo.>> Grato desde já.=Instruções para entrar na lista, sair da lista e usar a lista emhttp://www.mat.puc-rio.br/~obmlistas/obm-l.html=Instruções para entrar na lista, sair da lista e usar a lista em
http://www.mat.puc-rio.br/~obmlistas/obm-l.html
===

Re: [obm-l] Ajuda em geometria espacial

2007-11-01 Por tôpico JoaoCarlos_Junior
    Faça-se um triângulo escaleno de papel. Cole-o sobre uma mesa, esse é o triângulo ABC (base do tetraedro). Na realidade, não estou fazendo isso, é só uma descrição lingüística na tentativa de retratar a realidade que parece auxiliar. Coloque-se uma haste de madeira perpendicular à mesa por O: ortocentro de ABC.
    Bem, agora, sejam imaginadas três esferas, cada uma delas tem como diâmetro os lados: AB, BC e AC. Quaisquer pontos dessas esferas, se ligados a A, B e C, formarão com eles triângulos retângulos. Digamos que essas esferas se podem interpenetrar sem que qualquer delas se altere.
    Legal, a haste de madeira vai furar essas três esferas em um único ponto!? Provar isso, e daí avançar, parece ser crucial à resolução do problema. Mas como provar isso? Como daí avançar até o fim do problema? Qual é mesmo o fim do problema? Ah é! Esse mesmo (conjectura) se provarmos que a haste intercepta as esferas em um único, então, acabou.
    Por exemplo, a esfera com diâmetro AB tem raio c/2. Seja M o médio de AB. Formar-se-á o triângulo SMO (retângulo em O). S é o ponto em que a haste fura essa esfera. SO é a raiz quadrada da diferença entre os quadrados de c/2 e OM.  Alguma literatura deve mostrar a distância de O à metade de cada lado de um triângulo em função de a,b,c, quer dizer, pelo menos espero (sorriso). Já que eu não sei essas fórmulas, nem como a elas chegar facilmente, se é que são necessárias, pois, o fato é que se provarmos que: a raiz quadrada da diferença dos quadrados de c/2 e OM é igual à raiz quadrada da diferença de quadrados de b/2 e ON (N médio de AC), então, acabará a questão.
***
    Uma outra idéia: desenhe o triângulo ABC com seu ortocentro O e os segmentos de O a cada um desses vértices. Supunha possível, com o indicador e o polegar de qualquer das mãos, subir e abaixar o ponto O para fora do plano da mesa. Os ângulos da face do tetraedro (a se formar) serão sempre menores que os ângulos AOB, BOC e AOC. Porém, e justamente por isso, há alguma relação entre o quanto se afasta O da mesa (para virar S) e o quanto se reduzem os ângulos AOB, BOC e AOC para virarem ASB, BSC e ASC, mas paremos no ponto em que eles serão retângulos. A questão é provar que para uma mesma altura de afastamento de S da mesa, todos esses três últimos ângulos serão, justamente, retos. É outra idéia. Alguém me ajuda daqui à frente, por obséquio, e também, se houver erro para trás, retifiquem-no.
 
    Hoje é um novo dia, e pela idéia acima, pode-se também presumir que o tetraedro com o vértice assemelhado ao canto de um cubo existirá somente se os ângulos AOB, AOC, BOC forem maiores que 90º.
    Puxemos separadamente o triângulo AOB (por O) para se transformar em ASB reto. Assim c^2 = AS^2 + BS^2 = AO^2 + OS^2 + OS^2 + OB^2. Ou seja, 2OS^2 = c^2 – AO^2 – OB^2. Ora, do mesmo modo podemos elevar o AOC para se transformar em AS´C. A fórmula será a mesma, i.e., 2OS´^2 = b^2 – AO^2 – OC^2.
    OS= OS´ (o que desejamos) ó c^2 – OB^2 = b^2 – OC^2 ó c^2 + OC^2 = b^2 + OB^2. O que é uma verdade, pois transladando AC paralelamente a ele próprio até B, isto é, de forma que A esteja em B, (o ponto C será V, vértice do paralelogramo assim formado) e o segmento OB^2 + c^2 aparecerá no triângulo OBV retângulo. Da mesma forma, transladando AB paralelamente a ele mesmo até C, isto é, de forma que B esteja C, (o ponto A será A´, vértice do paralelogramo assim formado), teremos naturalmente que A´, C, V são colineares, ficará também formado o triângulo A´O V que tenho que provar que é isósceles (A´O = OV) para acabar o problema. Pois bem, OC é perpendicular a A´V e A´C = AB = CV, logo o triângulo que se quer provar que é isósceles é realmente isóscele, o que acaba o problema. 
    Corrijam-me por obséquio.
    Em realidade, agora vejo, após escrever de forma mais concreta, fácil é passá-la à linguagem mais fictícia, um tanto ensinada. Talvez seja mais fácil mostrar aos alunos algo mais pelo lado concreto (sempre que possível, claro) e caso ainda se julgue necessária fazer uma tradução, que seja feita (mas, não me parece necessária?!). Agora, creio também que o universo do que pode ser escrito de forma concreta, no mundo da matemática, é um tanto pequeno, não, creio que sim, não sei?
 
Fraternalmente, João.

Amigos estou precisando resolver os seguintes problemas: 1) Enunciar os casos de congruência de tetraedros, fazendo uma correspondência com os casos análogos de congruência de triângulos, mas ressaltando as diferenças nos dois casos. 2) Mostrar que se o tetraedro SABC tem faces formando ângulos retos no vértice S, isto é, os ângulos ASB, BSC e CSA são retos, então: a) A reta SO, ligando o vértice S ao ortocentro do triângulo ABC, é perpendicular ao plano ABC. b) O triângulo ABC é acutângulo. Grato desde já.Abra sua conta no Yahoo! Mail, o único sem limite de espaço para armazenamento! =

Re: [obm-l] Ajuda em geometria espacial

2007-10-26 Por tôpico JoaoCarlos_Junior
Apanhe três esquadros de 45º, em princípio de tamanhos diferentes. Junte-os todos de forma que o vértice S seja formado. O resultado é naturalmente o canto de um cubo. Bem, na realidade, não estou fazendo o que sugiro, apenas descrevo um método numa linguagem pouco precisa à matemática (estou ciente), mas é de qualquer forma uma realidade (ou tentando ser realista), portanto, somem comigo acrescentando correções.    Bem, se seccionarmos esses planos dos esquadros por outro (formador da base ABC) de forma que os esquadros sejam iguais, então, acabou o problema, pois a base ABC será um triângulo eqüilátero, portanto, acutângulo, e, naturalmente, pela simetria, SO é perpendicular ao plano ABC, (mas esse é o caso mais simples).     Agora, se a secção implica a desigualdade dos lados de ABC comparados dois a dois (o caso genérico)? Faça o desenho do canto de um cubo (agora fiz), imaginando intermináveis essas três arestas desenhadas. Marque três pontos quaisquer (um em cada uma delas), a distâncias distintas de S. ABC serão esses pontos marcados.    Em cada face, os ângulos são agudos, não sei explicar bem por que, mas é por isso que os ângulos da base serão também agudos.    Tentativa: marque o ponto A próximo de S. Imagine os pontos B e C o mais longe possível de S (pior caso). Os ângulos SAB e SAC crescem a medida em que B e C se distanciam de S. No entanto, esses ângulos serão sempre menores que 90º, pois, do contrário AB ou AC serão paralelas às arestas do cubo. Desenhe duas linhas tracejadas que se interceptam em A, porém, uma em cada plano das faces do cubo (ou parte de cubo). Ora, essas linhas tracejadas são também perpendiculares entre si. Agora, começou a ficar difícil o que desejo dizer: o ponto S estará à esquerda (ou à direita, conforme o desenho) dessas retas tracejadas, e, portanto, os lados AB e AC sempre estarão também e ao mesmo tempo à esquerda (ou à direita) dessas retas, assim, o ângulo BAC será sempre menor que 90º, logo, o triângulo ABC será sempre acutângulo.[EMAIL PROTECTED] escreveu: -Para: obm-l@mat.puc-rio.brDe: Zoroastro Azambuja <[EMAIL PROTECTED]>Enviado por: [EMAIL PROTECTED]Data: 24/10/2007 12:11Assunto: Re: [obm-l] Ajuda em geometria espacialZoroastro Azambuja <[EMAIL PROTECTED]> escreveu: Amigos estou precisando resolver os seguintes problemas:  1) Enunciar os casos de congruência de tetraedros, fazendo uma correspondência com os casos análogos de congruência de triângulos, mas ressaltando as diferenças nos dois casos.  2) Mostrar que se o tetraedro SABC tem faces formando ângulos retos no vértice S, isto é, os ângulos ASB, BSC e CSA são retos, então:  a) A reta SO, ligando o vértice S ao ortocentro do triângulo ABC, é perpendicular ao plano ABC.  b) O triângulo ABC é acutângulo.  Grato desde já. Abra sua conta no Yahoo! Mail , o único sem limite de espaço para armazenamento! Abra sua conta no Yahoo! Mail , o único sem limite de espaço para armazenamento! Instruções para entrar na lista, sair da lista e usar a lista em
http://www.mat.puc-rio.br/~obmlistas/obm-l.html
===

[obm-l] (OFF) Considerações aos Matemáticos Amigos

2007-10-18 Por tôpico JoaoCarlos_Junior




Algumas considerações:
1)Os alunos aprendem mais observando erros naturais de seus mestres do
que os acertos. Percebam a força da palavra: “naturais” nesta frase.
2)Uma resolução não é melhor que outra por ser mais curta. Muitos
outros critérios podem ser apontados, por exemplo: 1) Uma solução mais
longa (questão que comporta graus) pode ser mais adequada à solução de
determinado conjunto de problemas que derivam de algum apresentado do que
aqueloutra mais breve; 2) Se a solução mais longa foi também a primeira a
ser apresentada e serviu de degrau à segunda mais curta, então, ambas tem
valores iguais (ou próximos), pois o primeiro degrau de uma escada não é
melhor nem pior que o segundo.
3)O mundo matemático estaria bem mais desenvolvido se os matemáticos
estivessem mais unidos, i.e., fraternos.

Fraternalmente, João.



=
Instruções para entrar na lista, sair da lista e usar a lista em
http://www.mat.puc-rio.br/~obmlistas/obm-l.html
=


Re: [obm-l] Re: IMO 2007 (OFF)

2007-08-23 Por tôpico JoaoCarlos_Junior
A matemática parece então uma seqüência de desânimo e ânimo nessa ordem, pois verificado um problema no caminhar que deseja a solução, eis que surge o desânimo, esse, porém, já é o início do ânimo, simplesmente, escrevendo-se de forma mais otimista o problema anteriormente verificado, e, neste ciclo repetitivo, chega-se à solução desejada, se se conseguir. É como ascender aos Céus por uma escada.
Permita-me dizer, agora, algo em direção ao otimismo e, portanto, à solução da específica questão da IMO 2007: não se deve separar (ou há algum critério de separação) dos nós que possam aumentar o tamanho dos conjuntos cliques.
 
Permita-me ainda não ler sua solução, Ponce, posteriormente, tentarei continuar a minha do degrau onde parei.

Ola' pessoal,ja' respondi ao Joao em OFF, mas acho legal divulgar que as mensagens da lista podem ser acessadas diretamente a partir de http://www.mail-archive.com/obm-l@mat.puc-rio.br/maillist.html []'sRogerio Ponce Flickr agora em português. Você clica, todo mundo vê. Saiba mais. Instruções para entrar na lista, sair da lista e usar a lista em
http://www.mat.puc-rio.br/~nicolau/olimp/obm-l.html
===

[obm-l] Re: IMO 2007 (OFF)

2007-08-20 Por tôpico JoaoCarlos_Junior
Ponce:

Acordei com a idéia de que se todos os elementos de um conjunto 
clique possuem amizade com todos os elementos de um outro conjunto clique 
menos com 1 (ou alguns) desses elementos, então, colocado esse 1 (ou 
alguns) em sala separada, o conjunto clique resultante tornar-se-á maior 
que os anteriores. 
Portanto, eu estava errado.
E digo categórico: eu não sei fazer esse exercício não, e acho que meu 
exemplo analógico (o surto) não serve para resolver a questão.
Como não li tua resolução, não sei dizer se você a acertou. Sem querer, eu 
a deletei e então não pude fazer o que prometera (lê-la), confesso com 
algum grau de vergonha. Mande-me de novo, por obséquio.
Também não te pedirei desculpas, pois em nenhum momento creio que te 
ofendi. Eu estava sego e sem interesse maior pela questão.


Um Fraternal abraço, João. 



[obm-l] Solicitação aos Ilustres Matemáticos Responsáveis pela OBM

2007-08-17 Por tôpico JoaoCarlos_Junior




Aos Ilustres Matemáticos Responsáveis pela Olimpíada Brasileira de
Matemática:

Em especial, aos Professores Nicolau, Raph, Gugu, Shyne, Luciano, E. Poço,
E. Tegan, S.B. Feitosa, R. Barcelar, O. Campos, e outros por esses
indicados.

Nobres senhores:

  Alguns estudam matemática em busca de seu próprio desenvolvimento, o
que é extremamente louvável. Alguns de nós outros, no entanto, desejam
apenas se tornar instrumentos úteis aos nossos próprios filhos.
  Sustentado o interesse de auxiliá-los, reconhecemos, todavia, que por
maior que seja o nosso amor, e por mais que busquemos medrar esse
sentimento ao longo do tempo, ele é instrumento menos eficaz se comparado
ao intelecto de vossas senhorias e de outros que aqui se manifestam.
Há pais que, no afã de auxiliar, tem algumas intuições raras, mas
essa faculdade não pode ser ensinada, não tem critério.
   Prezados professores, nós, pais humildes do interior do país, sem
recursos para transferirmo-nos às cidades onde vossas senhorias residem,
permitam-nos solicitar: “estendam suas mãos às nossas ainda um pouco mais.
Além das belas revistas e livros já por vós publicados, desejamos que
nossos filhos os vejam em movimento, que ouçam suas vozes e vejam suas
felicidades ao versejar com matemática. Gravem, por obséquio, suas aulas e
disponibilizem-nas. É uma rogativa. Abracem a todos nós do interior, e, sem
condições de retribuir, rogamos aos céus as mais divinas bênçãos para todos
vós e seus familiares”.

Fraternalmente, João (Um pai).


=
Instruções para entrar na lista, sair da lista e usar a lista em
http://www.mat.puc-rio.br/~nicolau/olimp/obm-l.html
=


Re: [obm-l] Re: IMO 2007 (agora vai)

2007-08-09 Por tôpico JoaoCarlos_Junior
Ponce: Na realidade, percebi agora sua verdadeira emoção. Confesso que não queria olhar seus escritos. É o egoísmo que implica dessinteresse. Porém, agora, vou fazê-lo pela sua honesta postura.Ponce, será que você não poderia dialogar comigo na linguagem em que me manifesto? Diga-me diretamente se as frases que escrevi estão certas ou erradas?  Um abraço, ficamos mais amigos de alguma forma.Fraternalmente, João.  [EMAIL PROTECTED] escreveu: -Para: obm-l@mat.puc-rio.brDe: [EMAIL PROTECTED]Enviado por: [EMAIL PROTECTED]Data: 09/08/2007 10:47Assunto: Re: [obm-l] Re: IMO 2007 (agora vai)Prezado Ponce: Ola' Joao, nao foi pouco caso: ja' e' a 4a vez que mando esta mensagem, e, ate' agora, neca de pitibiriba - parece que o servidor da lista encruou... Mas, voltando 'a vaca fria, quero assinalar que tentar resolver certos problemas usando analogias pode ser ingrato porque frequentemente voce acaba destruindo (estabelecendo) vinculos e/ou mecanismos  (nao) existentes no problema original. As retificações necessárias para passar da analogia ao original foram feitas, vc que, por algum lapso, não percebeu. Veja que com a historia do barro, voce deixou escapar que os pedacos "retalhados", quando estao na mesma sala, NAO permanecem inertes e "retalhados". Este e' o comportamento do barro, mas nao e' o que acontece com os cliques, que se reagrupam automaticamente. As ligações dos retalhos se estabelecem (na mesma sala) exatamente até ao limite que existiam antes de serem recortados. Isso já havia sido dito. Permita-me dizer: perceba que todas as vezes que escrevi jamais desejei dar uma solução irretocável, mas cada vez que escrevi esforçei-me ao máximo para chegar a ela. São considerações distintas. A busca de solução é um caminhar. Vc próprio poderia do ponto onde parei, em cada vez que escrevi, ter seguido em frente para chegar ao fim. Não tenho mais em que contribuir com o problema, pois creio que ele acabou. Solicitemos auxílio, se ainda julgares necessário. Fraternalmente, João. Assim, por mais sem graca que pareca, experimente usar uma vezinha so' os numeros que estou sugerindo, e verifique o que acontece em cada passo. Consideremos a seguinte situacao: Competidores 1,2,3,4,5,6,7,8,9,10,11,12,13 Cliques existentes: 1,2,3,4 5, 6, 7 8, 9, 10 11, 12, 13 5, 7, 9 5, 7, 11 5, 8, 9 5, 8, 11 5, 9, 12 6, 7, 10 7, 9, 10 7, 11, 13 Agora, vamos seguir o seu proprio raciocinio, usando as salas A e B: - separar  o clique maximo integralmente: {1,2,3,4} - dividi-lo ao meio: {1,2}->A e  {3,4}->B -dividir montes menores em 2 partes proximas da metade (vamos percorrer a lista de cliques, obtendo o seguinte): {5,6,7}:  {5,6}->A  e  {7}->B {8,9,10}:  {8,9}->A  e  {10}->B {11, 12, 13}:  {11,12}->A  e  {13}->B Opa! neste ponto aparece um problema: o que fazer com o clique {5,7,9}? Ele faz parte do grupo "cliques existentes", e voce recomenda uma acao de divisao sobre este clique...so' que voce ja' havia dado destino a cada um dos elementos. E entao, como fica o algoritmo? Vou supor que ele termine aqui. Mas ha' um outro problema pior, pois as salas A e B estao com a seguinte distribuicao de competidores:  A={1,2,5,6,8,9,11,12}  e  B={3,4,7,10,13} Repare que o clique {1,2,3,4} deu lugar a 2 cliques com tamanho 2 ({1,2} e {3,4}), mas voce reagrupou dois cliques (5,8,9 por exemplo) com tamanho 3 em A, enquanto em B, nenhum clique tem mais que 2 elementos. Assim, por enquanto, esta forma de dividir esta' mostrando o contrario do que queremos provar. E a pergunta principal e' : como e' que voce garante que nao vai haver algum reagrupamento maior que a metade do maior clique inicial? Bem, ao final de tudo, qualquer que seja o algoritmo que voce encontre, ele tem que funcionar como prova (conforme o enunciado) e nao como algo que talvez funcione. Significa que, seguindo a logica que voce explicitar, deve ficar muito claro, em todas as transferencias de pessoas, o que aumenta e o que diminui, de forma a mostrar que e' sempre possivel fazer a divisao dos competidores em 2 salas com clique maximo de mesmo tamanho. Vamos la', Joao ! []'s Rogerio Ponce -- JoaoCarlos_Junior escreveu: Se a amizade não existia no conjunto competição, então, ela não passará a existir nas salas. Uma amizade é restabelecida se os recíprocos amigos forem inclusos na mesma sala, mesmo que em momentos distintos. Sim de fato, a amizade somente ficará quebrada (cortada, como queira) somente se os amigos estiverem nas salas distintas. Podemos continuar a escrever algo (que, a meu ver, está cada vez mais próximo de uma resposta integralmente satisfatória) na linguagem da própria pergunta, porém, devemos ser agradecidos com o êxtase - princípio do auxílio? que nos conduziu ou quer nos conduzir à resposta, por analogia. Prefiro a segunda à primeira. Permita-me, assim, expressar-me. Passar da 

Re: [obm-l] Re: IMO 2007 (agora vai)

2007-08-09 Por tôpico JoaoCarlos_Junior
Prezado Ponce:Ola' Joao,nao foi pouco caso: ja' e' a 4a vez que mando esta mensagem, e, ate' agora, neca de pitibiriba - parece que o servidor da lista encruou...Mas, voltando 'a vaca fria, quero assinalar que tentar resolver certos problemas usando analogias pode ser ingrato porque frequentemente voce acaba destruindo (estabelecendo) vinculos e/ou mecanismos  (nao) existentes no problema original.As retificações necessárias para passar da analogia ao original foram feitas, vc que, por algum lapso, não percebeu.Veja que com a historia do barro, voce deixou escapar que os pedacos "retalhados", quando estao na mesma sala, NAO permanecem inertes e "retalhados". Este e' o comportamento do barro, mas nao e' o que acontece com os cliques, que se reagrupam automaticamente.As ligações dos retalhos se estabelecem (na mesma sala) exatamente até ao limite que existiam antes de serem recortados. Isso já havia sido dito.
Permita-me dizer: perceba que todas as vezes que escrevi jamais desejei dar uma solução irretocável, mas cada vez que escrevi esforçei-me ao máximo para chegar a ela. São considerações distintas. A busca de solução é um caminhar.
Vc próprio poderia do ponto onde parei, em cada vez que escrevi, ter seguido em frente para chegar ao fim. 
Não tenho mais em que contribuir com o problema, pois creio que ele acabou. Solicitemos auxílio, se ainda julgares necessário.
Fraternalmente, João.Assim, por mais sem graca que pareca, experimente usar uma vezinha so' os numeros que estou sugerindo, e verifique o que acontece em cada passo.Consideremos a seguinte situacao:Competidores 1,2,3,4,5,6,7,8,9,10,11,12,13Cliques existentes:1,2,3,45, 6, 78, 9, 1011, 12, 135, 7, 95, 7, 115, 8, 95, 8, 115, 9, 126, 7, 107, 9, 107, 11, 13Agora, vamos seguir o seu proprio raciocinio, usando as salas A e B:- separar  o clique maximo integralmente: {1,2,3,4}- dividi-lo ao meio: {1,2}->A e  {3,4}->B-dividir montes menores em 2 partes proximas da metade (vamos percorrer a lista de cliques, obtendo o seguinte):{5,6,7}:  {5,6}->A  e  {7}->B{8,9,10}:  {8,9}->A  e  {10}->B{11, 12, 13}:  {11,12}->A  e  {13}->BOpa! neste ponto aparece um problema: o que fazer com o clique {5,7,9}?Ele faz parte do grupo "cliques existentes", e voce recomenda uma acao de divisao sobre este clique...so' que voce ja' havia dado destino a cada um dos elementos. E entao, como fica o algoritmo? Vou supor que ele termine aqui.Mas ha' um outro problema pior, pois as salas A e B estao com a seguinte distribuicao de competidores: A={1,2,5,6,8,9,11,12}  e  B={3,4,7,10,13}Repare que o clique {1,2,3,4} deu lugar a 2 cliques com tamanho 2 ({1,2} e {3,4}), mas voce reagrupou dois cliques (5,8,9 por exemplo) com tamanho 3 em A, enquanto em B, nenhum clique tem mais que 2 elementos.Assim, por enquanto, esta forma de dividir esta' mostrando o contrario do que queremos provar.E a pergunta principal e' : como e' que voce garante que nao vai haver algum reagrupamento maior que a metade do maior clique inicial?Bem, ao final de tudo, qualquer que seja o algoritmo que voce encontre, ele tem que funcionar como prova (conforme o enunciado) e nao como algo que talvez funcione. Significa que, seguindo a logica que voce explicitar, deve ficar muito claro, em todas as transferencias de pessoas, o que aumenta e o que diminui, de forma a mostrar que e' sempre possivel fazer a divisao dos competidores em 2 salas com clique maximo de mesmo tamanho.Vamos la', Joao ![]'sRogerio Ponce--JoaoCarlos_Junior escreveu:Se a amizade não existia no conjunto competição, então, ela não passará a existir nas salas.Uma amizade é restabelecida se os recíprocos amigos forem inclusos na mesma sala, mesmo que em momentos distintos.Sim de fato, a amizade somente ficará quebrada (cortada, como queira) somente se os amigos estiverem nas salas distintas.Podemos continuar a escrever algo (que, a meu ver, está cada vez mais próximo de uma resposta integralmente satisfatória) na linguagem da própria pergunta, porém, devemos ser agradecidos com o êxtase - princípio do auxílio? que nos conduziu ou quer nos conduzir à resposta, por analogia. Prefiro a segunda à primeira. Permita-me, assim, expressar-me. Passar da linguagem em analogia à do próprio problema me não parece difícil. Então:1) Da massa de argila (toda a competição), podemos separar dela o conjunto clique máximo integralmente. Empós, dividimo-lo no meio, jogando cada metade em duas mesas distintas (as salas).2) Os montes menores também devem ser divididos em duas partes, de forma que cada uma dessas partes cliques sejam de tamanho menor que as metades acima (no máximo, há uma igualdade, não é difícil verificar). Percebamos que se havia anteriormente amizade entre os elementos desses conjuntos menores entre eles próprios e deles para com os elementos do conjunto maior, as amiz

[obm-l] (OFF) Nome de Alguns dos Grandes Professores do País

2007-08-03 Por tôpico JoaoCarlos_Junior




Nehab, Ponce:

Quais são todos professores residentes nos grandes centros que preparam
jovens brasileiros para as Olimpíadas de Matemática, e que sejam também
partícipes desta bela lista?
Desejo-lhes fazer ousada e nobre solicitação.

Fraternalmente, João


=
Instruções para entrar na lista, sair da lista e usar a lista em
http://www.mat.puc-rio.br/~nicolau/olimp/obm-l.html
=


Re: [obm-l] IMO 2007

2007-08-02 Por tôpico JoaoCarlos_Junior
Se a amizade não existia no conjunto competição, então, ela não passará a existir nas salas. Uma amizade é restabelecida se os recíprocos amigos forem inclusos na mesma sala, mesmo que em momentos distintos.Sim de fato, a amizade somente ficará quebrada (cortada, como queira) somente se os amigos estiverem nas salas distintas.Podemos continuar a escrever algo (que, a meu ver, está cada vez mais próximo de uma resposta integralmente satisfatória) na linguagem da própria pergunta, porém, devemos ser agradecidos com o êxtase - princípio do auxílio ? que nos conduziu ou quer nos conduzir à resposta, por analogia. Prefiro a segunda à primeira. Permita-me, assim, expressar-me. Passar da linguagem em analogia à do próprio problema me não parece difícil. Então:1) Da massa de argila (toda a competição), podemos separar dela o conjunto clique máximo integralmente. Empós, dividimo-lo no meio, jogando cada metade em duas mesas distintas (as salas).2) Os montes menores também devem ser divididos em duas partes, de forma que cada uma dessas partes cliques sejam de tamanho menor que as metades acima (no máximo, há uma igualdade, não é difícil verificar). Percebamos que se havia anteriormente amizade entre os elementos desses conjuntos menores entre eles próprios e deles para com os elementos do conjunto maior, as amizades ficarão restabelecidas entre os elementos que já eram previamente amigos, porém, agora, só para aqueles que estão na mesma sala. Esses restabelecimentos, no entanto, não aumentam os tamanhos dos conjuntos cliques cortados.3) depois a massa que sobrou você pode cortá-la ou não (como queira), jogando-a integralmente em uma só sala, ou retalhá-la a gosto, lançando as partes em ambas as salas, sob qualquer critério. Com sinceridade, sem o menor grau de sofisma: agrado tuas contraposições, Ponce, que me impeliram à frente nessa resolução. Se ainda houver alguma(s), por gentileza principalmente a mim, manifeste-a(s).Desculpe-me não ter respondido logo. Obrigado. Fraternalmente, João.   [EMAIL PROTECTED] escreveu: -Para: obm-l@mat.puc-rio.brDe: Rogerio Ponce <[EMAIL PROTECTED]>Enviado por: [EMAIL PROTECTED]Data: 28/07/2007 4:09Assunto: Re: [obm-l] IMO 2007Ola' Joao, suponha a competicao com os competidores numerados de 1 a 13, formando os seguintes cliques: 1, 2, 3, 4 5, 6, 7 8, 9, 10 11, 12, 13 5, 8, 9 5, 8, 11 5, 9, 12 6, 7, 10 7, 9, 10 7, 11, 13 Repare que nao da' para pensarmos em dividir cada conjunto ao meio (ou proximo do meio) de forma independente, pois eles nao sao obrigatoriamente disjuntos. Entao, quando voce faz a divisao de um clique, muitas vezes tambem estara separando (ou agrupando) outro clique. Assim, embora o maior clique tenha inicialmente "2n" elementos , nao e' verdade que a sua forma de dividi-los va' produzir cliques com no maximo "n" elementos (embora essa seja a nossa primeira impressao). Seguindo sua sugestao, poderiamos separar os competidores assim: [1, 2] na sala "A" ,  [3, 4] na sala "B" [5, 6]  na sala "A" ,  [7] na sala "B" [8, 9] na sala "A" ,  [10] na sala "B" [11, 12] na sala "A" ,  [13] na sala "B" Parariamos a divisao neste ponto, uma vez que ja' teriamos dado destino a todos os competidores. Entretanto, na sala "A" existe um clique (5,8,11) com 3 competidores , enquanto os maiores cliques da sala "B" nao passam de 2 competidores. Acho que este exemplo serve de partida para voce elaborar o que pode acontecer durante qualquer outra forma de divisao. []'s Rogerio Ponce -- Alguém, por gentileza, comente o surto abaixo. Ponce, preliminarmente, creio que está correto. Vou olhar com maior atenção. O surto:         Vamos busca modelar (como se modela argila) esse conjunto competição.     Não estou brincando não, falo sério.     Cada conjunto clique desse é um monte de argila. Existe um conjunto maior com 2n elementos.     Esses conjuntos de barro podem estar unidos. Essas uniões são as amizades que ligam os conjuntos clique sem transformá-los num conjunto clique maior. Também podem existir montes sem ligação com nenhum outro.     Ora, sempre é possível dividir todo o conjunto competição, de forma que o maior conjunto clique com 2n participantes seja divido ao meio e os demais também ao meio (se par) ou em dois números inteiros e consecutivos (se ímpares) e, sem tanta preocupação com as amizades inter-cliques, pois elas não aumentam o tamanho de cada conjunto. Assim, sempre será possível se ter aí o que se deseja provar.     Falta precisão, claro, mais essa pode ser simples a partir da idéia acima, creio.   Fraternalmente, João. Alertas do Yahoo! Mail em seu celular. Saiba mais . Instruções para entrar na lista, sair da lista e usar a lista em
http://www.mat.puc-rio.br/~nicolau/olimp/obm-l.html
===

Re: [obm-l] Uma boa de geometria - CALMA !!! (OFF)

2007-08-01 Por tôpico JoaoCarlos_Junior
Percebo em certo grau que a ousadia direcionada à honestidade, à nobreza e à humildade auxilia na resolução de questões.
Fraternalmente, João.
Olá pessoal!Muito obrigado pela colaboração de todos na solução do problema.Enviei a solução para [EMAIL PROTECTED] com as devidas citações ao Nehab eao Marcio. Obrigado pela dica da "estrategia padrao" Marcio!Certamente será muito útil em problemas futuros.Por sinal como foi a sua solução para o problema? Fiquei curioso ecreio que outros também estão.Alguém saberia me dizer se é esse e-mail([EMAIL PROTECTED]) o correto paraenviar as soluções dos problemas propostos da Eureka? Tinha enviadouma outra vez mas não obtive resposta.Abraços,Douglas RibeiroOBS: Desculpe a ousadia Nehab, mas foi foi mais forte que eu!Em 31/07/07, Marcio Cohen<[EMAIL PROTECTED]> escreveu:> Douglas,>> Você certamente fez a parte difícil da questão e merece 100% dos créditos> por isso. Eu tinha feito uma solução por complexos para a questão da Eureka> na aula de treinamento da imo, mas a sua é muito mais legal!!>> Para provar o detalhe final da sua solução, minha estratégia padrão é:>> Seja a=exp(iA), b=exp(iB), c=exp(iC). Então, abc = -1 e como> exp(ix)+exp(-ix) = 2cosx:>> (cosA)^2 + (cosB)^2 + (cosC)^2 = (1/4)*(a^2 + 1/a^2 + b^2 + 1/b^2 + c^2 +> 1/c^2 + 6)> = (1/4)*(a^2 + b^2 + c^2 + (bc)^2 + (ab)^2 + (ac)^2+6);>>> 8cosA*cosB*cosC = (a+1/a)(b+1/b)(c+1/c) = -(1+a^2)(1+b^2)(1+c^2) => -(1+a^2+b^2+c^2+(ab)^2+(ac)^2+(bc)^2+1).>> Substituindo uma na outra, 8cosA*cosB*cosC = -(2+4*( (cosA)^2 + (cosB)^2 +> (cosC)^2 - 6), ou seja,> (cosA)^2 + (cosB)^2 + (cosC)^2 = 1 - 2cosAcosBcosC>> Abraços,> Marcio Cohen>>> On 7/30/07, Douglas Ribeiro Silva < [EMAIL PROTECTED]> wrote:> >> > Olá Nehab!> >> > Primeiramente gostaria de expressar minha satisfação do problema ter> > de fato chamado sua atenção e do Rogério Ponce. Já participo da> > lista(não muito ativamente) há um bom tempo e percebo que assim como> > eu, vocês gostam muito de geometria.> >> > O problema na verdade veio da minha cabeça, mas foi inspirado em um> > problema proposto na última(ou penúltima) Eureka. Originalmente o> > problema pedia para mostrar que XYZ estão alinhados se e somente se> > cosA*cosB*cosC = -3/8. Então pensei em me inspirar nos chineses, que> > gostavam de resolver teoremas usando áreas, e pensei em "zerar" a área> > do triangulo XYZ para chegar na tão esperada relação.> >> > Por um lado estou satisfeito, pois consegui chegar em uma expressão> > que relaciona as áreas corretamente, por outro estou frustrado pois> > não consigo fazer a última passagem, que certamente exige uma> > fatoração ou algo do tipo que não estou conseguindo enxergar.> >> > A relação que eu cheguei foi S(XYZ) = S(ABC)*[7 - 4((cosA)^2 +> > (cosB)^2 + (cosC)^2)].> > Note que a relação é válida nos casos mais triviais em que o triangulo> > é equilatero, retangulo(que, como o Rogerio falou, são respectivamente> > 1:4 e 1:3) ou isosceles com angulo de 120 graus(basta fazer um desenho> > para ver que a área dá zero). Notem que a relação pedida no problema> > da Eureka é satisfeita para este triangulo isosceles.> >> > Aos curiosos que querem saber como eu cheguei nessa relação, segue a> > idéia abaixo:> >> > Construam o triangulo ABC e suas respectivas reflexões XYZ.> > Observe que S(XYZ) = [S(ABC) + S(BCX) + S(ACY) + S(ABZ)] - S(AYZ) -> > S(XBZ) - S(XYC)> >> > S(ABC) = S(BCX) = S(ACY) = S(ABZ) por construção> >> > As áreas de AYZ XBZ e XYC podem ser somadas ou subtraídas, dependendo> > se os ângulos YAB = 3A, XBZ = 3B ou XCY = 3C forem maiores ou menores> > que 180 graus. Para esses triangulos vou usar que S(AYZ) => > bc*sen(3A)/2, S(XBZ) = ac*sen(3B)/3 e S(XYC) = ab*sen(3C)/2.> >> > Então a relação passa a ser S(XYZ) = 4S(ABC) - bc*sen(3A)/2 -> > ac*sen(3B)/3 - ab*sen(3C)/2> >> > Agora substituímos sen(3X) = -4*[sen(X)]^3 + 3*sen(X) em todos e> > substituímos também bc/2 ac/2 e ab/2 respectivamente por> > S(ABC)/sen(A), S(ABC)/sen(B) e S(ABC)/sen(C), devido à mesma fórmula> > de área em função dos lados e do angulo para o triangulo original.> >> > Fazendo as devidas substituições acima, simplificamos os senos e> > ficamos com a relação da soma dos quadrados dos senos. Basta trocar> > [sen(X)]^2 por 1 - [cos(X)]^2 e chegamos em S(XYZ) = S(ABC)*[7 -> > 4((cosA)^2 + (cosB)^2 + (cosC)^2)].> >> > Quando eu enviei o problema ainda não tinha chegado nesse resultado e> > achava que chegaria em uma expressão mais fácil de passar para o> > produto de cossenos.> >> > Qualquer ajuda para terminar o problema eu agradeço bastante e> > certamente darei os devidos créditos quando enviar a solução para a> > Eureka.> >> > Abraços, Douglas> >> >> >> >> > Em 30/07/07, Carlos Eddy Esaguy Nehab<[EMAIL PROTECTED] > escreveu:> > >> > >  Oi, querido Ponce> > >> > >  Naturalmente não se supunha (pelo menos eu) que a relação entre as> áreas> > > independesse do triângulo, mas mesmo assim, confesso que tentei vários> > > caminhos e não encontrei uma solução simples para o problema.> > >

Re: [obm-l] IMO 2007

2007-07-27 Por tôpico JoaoCarlos_Junior

Alguém, por gentileza, comente o surto abaixo. Ponce, preliminarmente, creio que está correto. Vou olhar com maior atenção.
O surto:     
    Vamos busca modelar (como se modela argila) esse conjunto competição. 
    Não estou brincando não, falo sério.
    Cada conjunto clique desse é um monte de argila. Existe um conjunto maior com 2n elementos. 
    Esses conjuntos de barro podem estar unidos. Essas uniões são as amizades que ligam os conjuntos clique sem transformá-los num conjunto clique maior. Também podem existir montes sem ligação com nenhum outro.
    Ora, sempre é possível dividir todo o conjunto competição, de forma que o maior conjunto clique com 2n participantes seja divido ao meio e os demais também ao meio (se par) ou em dois números inteiros e consecutivos (se ímpares) e, sem tanta preocupação com as amizades inter-cliques, pois elas não aumentam o tamanho de cada conjunto. Assim, sempre será possível se ter aí o que se deseja provar.
    Falta precisão, claro, mais essa pode ser simples a partir da idéia acima, creio.
 
Fraternalmente, João.
 
Ola' Shine, Joao e colegas da lista,acho que eu poderia melhorar a explicacao, mas vamos la' assim mesmo...Sempre podemos dividir os competidores da seguinte forma:Coloque o maior clique na sala "A" e todos os outros na sala "B".Se na sala "B" tambem houver um clique com o tamanho da sala "A", a divisao esta' completa. Se nao, execute a etapa X.Etapa X :Passe um competidor da sala "A" para a sala "B".Dessa forma, o clique em "A" diminui de 1 unidade, alguns cliques em "B" crescem de 1 unidade, e outros cliques em "B" nao se alteram.Entao:- Se o(s) maior(es) clique(s) em "B" ainda nao igualou o clique em "A", repita a etapa "X".- Se o(s) maior(es) clique(s) em "B" igualou o clique em "A", a divisao esta' completa.- E se o(s) maior(es) clique(s) em "B" ultrapassou o clique em "A" ?Bem, em cada um desses cliques (o clique formado pelos migrados de "A" nao esta' entre estes cliques, pois o clique original em "A" era par), existe algum competidor que nao estava originalmente em "A" .Passe esse competidor para "A" (faca isso em todos os cliques de "B" que ultrapassaram o valor em "A").Agora a divisao esta'  completa.OBS: Poderia acontecer de todos os jogadores transferidos para "A" formarem um clique independente, superior ao clique em "A" ?Nao, caso contrario eles ja' estariam formando um clique na sala "B" igual ao clique em "A", antes da ultima passagem de alguem de "A" para "B", e o processo ja' teria terminado.Note que o clique original em "A" e' par. Assim, todo o processo descrito termina no maximo quando metade dos competidores em "A" tiver sido transferida para "B".[]'sRogerio PonceCarlos Yuzo Shine <[EMAIL PROTECTED]> escreveu:3. Numa competição de matemática, alguns competidores são amigos. Amizade é sempre mútua. Chame um grupo de competidores de clique se quaisquer dois entre eles são amigos. Em particular, qualquer grupo com menos de dois amigos é um clique. O número de membros de um clique é o seu tamanho.Dado que, nesta competição, o maior tamanho de um clique é par, prove que os competidores podem ser divididos em duas salas tais que o maior tamanho de um clique em uma sala é igual ao maior tamanho de um clique na outra sala.[]'sShineAlertas do Yahoo! Mail em seu celular. Saiba mais. Instruções para entrar na lista, sair da lista e usar a lista em
http://www.mat.puc-rio.br/~nicolau/olimp/obm-l.html
===

Re:Res:[obm-l] IMO 2007

2007-07-27 Por tôpico JoaoCarlos_Junior
Acho que você está certo, vou analisar.[EMAIL PROTECTED] escreveu: -Para: "obm-l" De: "fernandobarcel" <[EMAIL PROTECTED]>Enviado por: [EMAIL PROTECTED]Data: 26/07/2007 21:53Assunto: Re:Res:[obm-l] IMO 2007João,"clique é um grupo de competidores onde quaisquer dois entre eles são amigos".Portanto, a competição pode não ser um clique.Abraços,-- Início da mensagem original ---> > Tentativa ao terceiro problema>         A própria competição (que encerra todos os competidores) é clique, pois> : 1) Há alguns competidores amigos; 2) A amizade é mútua, então, há pelo menos > dois amigos na competição. > ...> > > >    [EMAIL PROTECTED] escreveu: -> >      >      3. Numa competição de matemática, alguns competidores são amigos.>      Amizade é sempre mútua. Chame um grupo de competidores de clique se>      quaisquer dois entre eles são amigos. Em particular, qualquer grupo>      com menos de dois amigos é um clique. O número de membros de um>      clique é o seu tamanho.>      Dado que, nesta competição, o maior tamanho de um clique é par,>      prove que os competidores podem ser divididos em duas salas tais>      que o maior tamanho de um clique em uma sala é igual ao maior>      tamanho de um clique na outra sala.=Instruções para entrar na lista, sair da lista e usar a lista emhttp://www.mat.puc-rio.br/~nicolau/olimp/obm-l.html=Instruções para entrar na lista, sair da lista e usar a lista em
http://www.mat.puc-rio.br/~nicolau/olimp/obm-l.html
===

Res:[obm-l] IMO 2007

2007-07-26 Por tôpico JoaoCarlos_Junior
Tentativa de 1a Duas seqüências de números reais (a1, ..., an) e (x1, ..., xn).    Podemos colocar todos esses números numa seqüência única, e, depois organizá-los, para que o último elemento seja o maior deles; e o primeiro, o menor.    Ora, essa diferença entre o máximo e o mínimo da seqüência com 2n números é sempre maior que a diferença entre quaisquer dos elementos interiores - o que prova a - pois se é maior que a própria diferença é maior que a sua metade. Essa verdade existirá mesmo para a diferença entre os extremos, pois, um número positivo é maior que a sua metade. Diz-se positivo anteriormente, pois pelo enunciado da questão, as diferenças encontradas são sempre positivas, calculadas diretamente ou pelo uso do módulo.Tentativa de 1b Xi! Provavelmente, há algo que não percebi. Talvez (a) necessite de alguma retificação, pois, como pode um número ser igual a sua metade. (sorriso).    Acho que, se em uma dessas seqüências, ou, de repente ambas, tiverem números negativos e positivos (simétricos, talvez), então, vai se chegar nessa igualdade desejada.    Não, isso tem haver com o módulo usado em um cálculo e não no outro. Isso mesmo!    Seja (a1,..., an) a seqüência após organizada de forma an e a1 sejam o maior e o menor, respectivamente. Assim, d (calculado conforme o enunciado) será an - a1. Se a1 é negativo e na é positivo, então, d = an + |a1|.    Agora, pensemos na seqüência (x1, ..., xn) também organizada. Que números podem ser esses de forma que D (a maior diferença entre os elementos que se correspondem) seja a metade de d? Hum... é isso, ou é quase isso, se cada xi é metade de ai, então, então a maior diferença entre |xi ? ai| será para an, i.e., |an ? an/2|, ou seja, an/2.    Não é isso não. Se cada xi é ai +, então, acabou? Bem, essa fórmula dá seqüência crescente para xi. E a diferença xi ? ai é também crescente? Bem, é, pois, xi-ai = (ai ? a1)/2. Logo, a maior diferença desse tipo ocorrerá para xn ? an. E essa diferença será igual a metade de an ? a1. Acabou. Toda essa resolução pode ser melhorada (presumo), mas vou deixar assim por enquanto, depois volto. Vamos tentar o problema 2. Que pontuação poderia ganhar assim do jeito que está?Instruções para entrar na lista, sair da lista e usar a lista em
http://www.mat.puc-rio.br/~nicolau/olimp/obm-l.html
===

Res:[obm-l] IMO 2007

2007-07-26 Por tôpico JoaoCarlos_Junior
Tentativa ao terceiro problema	A própria competição (que encerra todos os competidores) é clique, pois: 1) Há alguns competidores amigos; 2) A amizade é mútua, então, há pelo menos dois amigos na competição. 	No conjunto clique particular não há amigos, haja vista que a amizade é mútua, e, assim, num conjunto que contém dois amigos como subconjunto, sempre esses amigos existirão. Clique particular também pode ser o conjunto vazio.    Pelo primeiro parágrafo e a afirmação de que na competição o maior tamanho de um clique é par, então, a competição possui um número par de jogadores.    Com as afirmações acima, provar que: o conjunto competição pode ser divido em duas partes tais que número de jogadores de uma dessas partes (que encerra dois amigos no mínimo) é igual ao número de jogadores da outra parte. Ora, isso já foi provado acima, do 1º parágrafo ao 3º, haja vista que chegamos à conclusão que na competição há número par de jogadores, com no mínimo dois amigos em toda a competição, e, portanto, uma metade que contém esses dois amigos pode ir a uma sala e a outra metade para outra, não importando saber se essa última metade contém amigos ou não, já que ela será sempre clique, pela definição particular ou genérica de clique. [EMAIL PROTECTED] escreveu: -Para: obm-l@mat.puc-rio.brDe: Carlos Yuzo Shine <[EMAIL PROTECTED]>Enviado por: [EMAIL PROTECTED]Data: 25/07/2007 12:08Assunto: [obm-l] IMO 2007Saiu agora o primeiro dia, no site do Mathlinks: http://www.mathlinks.ro/resources.php?c=1&cid=16&year=2007Traduzindo:1. São dados os números reais a_1, a_2, ..., a_n. Para cada i, 1 <= i <= n, definad_i = max{a_j, 1 <= j <= i} - min{a_j, i <= j <= n}.Seja d = max{d_i, 1 <= i <= n}.a) Prove que, para todos reais x_1 <= x_2 <= ... <= x_n,max{ |x_i - a_i|, 1 <= i <= n} >= d/2 (*)b) Mostre que existem reais  x_1 <= x_2 <= ... <= x_n tais que a igualdade em (*) ocorre.2. Considere cinco pontos A, B, C, D, E tais que ABCD é um paralelogramo e BCED é um quadrilátero cíclico. Seja r uma reta passando por A. Suponha que r corte o interior do segmento DC em F e a reta BC em G. Suponha também que EF = EG = EC. Prove que r é a bissetriz do ângulo DAB.3. Numa competição de matemática, alguns competidores são amigos. Amizade é sempre mútua. Chame um grupo de competidores de clique se quaisquer dois entre eles são amigos. Em particular, qualquer grupo com menos de dois amigos é um clique. O número de membros de um clique é o seu tamanho.Dado que, nesta competição, o maior tamanho de um clique é par, prove que os competidores podem ser divididos em duas salas tais que o maior tamanho de um clique em uma sala é igual ao maior tamanho de um clique na outra sala.[]'sShine      Get the free Yahoo! toolbar and rest assured with the added security of spyware protection.http://new.toolbar.yahoo.com/toolbar/features/norton/index.php=Instruções para entrar na lista, sair da lista e usar a lista emhttp://www.mat.puc-rio.br/~nicolau/olimp/obm-l.html=Instruções para entrar na lista, sair da lista e usar a lista em
http://www.mat.puc-rio.br/~nicolau/olimp/obm-l.html
===

Re: [obm-l] Uma boa de geometria

2007-07-25 Por tôpico JoaoCarlos_Junior
Valeu, Nehab, obrigado. Vou analisar empós o período de trabalho.
Oi, João Carlos, Tive uma idéia que não sei se frutifica, pois não estou muito inspirado, mas veja se ajuda.  (é uma solução não geométrica, no sentido usual): "Dada uma reta fixa, pense na transformação (não linear, é claro) que associa a cada ponto do plano o seu simétrico com relação a tal reta".1) Observe agora que dado um triângulo, a transformação T1, digamos, que toma o simétrico de A, B e C com relação à reta suporte de AB deixa A e B no mesmo lugar e coloca o ponto C onde você quer (ou seja, no simétrico C' etc). Como conseqüência, se você pensar nas 3 transformações T1, T2 e T3 (respectivamente que tomam o simétrico com relação às retas suporte de AB, BC e CA), verá que o triângulo que você construiu tomando os três simétricos é a imagem do triângulo original pela composta das 3 transformações T1, T2 e T3 (concorda?).   Logo, a questão é saber se a tal composta é fácil de analisar.2) Vejamos: uma simetria com relação a uma reta pode ser decomposta como soma de uma simetria (transformação linear com determinante -1)  e uma translação. Pense na T1, por exemplo, assim: se a reta suporte de AB é a reta y = px + q dá para calcular a imagem de um ponto (x; y) por T1:   T1( x, y) = T'(x, y)  + 1/(p^2+1) . (2q; -2pq),  onde a matriz da transformação T' (simetria) possui linha 1 igual a  [2p   p^2 -1]  e segunda linha [ 1 - p^2   2p], ambas divididas por (p^2 +1).  Dica: Imponha que o ponto médio de CC' está nesta reta e que o coeficiente angular da reta  CC' vale -1/p (são ortogonais).3) Agora: será que é fácil analisar a composição de T1, T2  e T3  sem muito trabalho braçal? Não me inpirei, mas fica aí a idéia.   Naturalmente que vou tentar uma solução geométrica mais tarde  :-). Abraços,Nehab At 08:33 25/7/2007, you wrote:Até o momento, não sei como resolver essa questão não.Seja um triangulo ABC com lados a, b, c.X eh a reflexao de A em relacao a reta que passa por BCY eh a reflexao de B em relacao a reta que passa por ACZ eh a reflexao de C em relacao a reta que passa por ABQual a relacao entre as areas de ABC e XYZ?=Instruções para entrar na lista, sair da lista e usar a lista em http://www.mat.puc-rio.br/~nicolau/olimp/obm-l.html=Instruções para entrar na lista, sair da lista e usar a lista em http://www.mat.puc-rio.br/~nicolau/olimp/obm-l.html Instruções para entrar na lista, sair da lista e usar a lista em
http://www.mat.puc-rio.br/~nicolau/olimp/obm-l.html
===

Re: [obm-l] Uma boa de geometria

2007-07-25 Por tôpico JoaoCarlos_Junior
Até o momento, não sei como resolver essa questão não.
Seja um triangulo ABC com lados a, b, c.X eh a reflexao de A em relacao a reta que passa por BCY eh a reflexao de B em relacao a reta que passa por ACZ eh a reflexao de C em relacao a reta que passa por ABQual a relacao entre as areas de ABC e XYZ?=Instruções para entrar na lista, sair da lista e usar a lista emhttp://www.mat.puc-rio.br/~nicolau/olimp/obm-l.html=Instruções para entrar na lista, sair da lista e usar a lista em
http://www.mat.puc-rio.br/~nicolau/olimp/obm-l.html
===

Re: [obm-l] Análise combinatória - número de lutas

2007-07-20 Por tôpico JoaoCarlos_Junior
Você parece ser um Espírito nobre e elevado. Não há em que se desculpar. Aceito as desculpas para deixar feliz o teu coração, mas não são necessárias. Essa mudança do problema restringe-se ao desejo de se encontrar o mínimo.
Okagora entendi.  Vc escolheu dar a solucao pra uma versao adaptada doproblema e nao ao problema proposto.  Infelizmente eu nao sou advinho e sevc tivesse avisado antes, eu nao teria te corrigido...foi mal ae>From: [EMAIL PROTECTED]>Reply-To: obm-l@mat.puc-rio.br>To: obm-l@mat.puc-rio.br>Subject: Re: [obm-l]  Análise combinatória - número de lutas>Date: Fri, 20 Jul 2007 11:32:17 -0400>>É que o problema necessita de uma retificação. Quando se chega a 3>participantes, duas disputas bastam para eliminar 1. E, com 2>participantes, basta uma disputa para eliminar o perdedor e definir o>vencedor.>[EMAIL PROTECTED] escreveu: ->>Para: obm-l@mat.puc-rio.br>De: "Qwert Smith" <[EMAIL PROTECTED]>>Enviado por: [EMAIL PROTECTED]>Data: 20/07/2007 10:13>Assunto: Re: [obm-l] Análise combinatória - número de lutas>>??? de onde vc tirou 3(n-3)+3 pra minimo.>>Para eliminarmos n-1 participantes numa competicao onde a elimicao se da>com>d derrotas sao necessarias (n-1)*d partidas.>> >From: [EMAIL PROTECTED]> >Reply-To: obm-l@mat.puc-rio.br> >To: obm-l@mat.puc-rio.br> >Subject: Re: [obm-l]  Análise combinatória - número de lutas> >Date: Fri, 20 Jul 2007 09:48:53 -0400> >> >Bem, encontramos: mínimo: 24. Máximo: 29.E ainda, as regras gerais:>mínimo:> >3(n-3)+3  máximo: (n-1)*3+ 2> >[EMAIL PROTECTED] escreveu: -> >> >Para: obm-l@mat.puc-rio.br> >De: "Qwert Smith" <[EMAIL PROTECTED]>> >Enviado por: [EMAIL PROTECTED]> >Data: 20/07/2007 8:36> >Assunto: Re: [obm-l] Análise combinatória - número de lutas> >> >Acho que o problema e bem mais simples que isso.> >> >Para que um lutador seja eliminado ele perde 3 vezes.  Para que 9>lutadores> >sejam eliminados sao necessarias pelo menos 9 x 3 lutas.> >> >Logo o minimo e 27.> >> >O numero de lutas e sempre 27 + n.  'n' e o numero de lutas que o campeao> >perdeu.  Mas o campeao so pode perder no maximo 2 lutas ou nao seria o> >campeao.  Logo o maximo de lutas e 29.> >> >> > >From: [EMAIL PROTECTED]> > >Reply-To: obm-l@mat.puc-rio.br> > >To: obm-l@mat.puc-rio.br> > >Subject: Re: [obm-l]  Análise combinatória - número de lutas> > >Date: Fri, 20 Jul 2007 08:15:50 -0400> > >> > >> > >> > >> > >> > >Tentativa> > >> > >    Bem, duas considerações preliminares: 1) 1 é imbatível; 2)>Alguns> > >outros sempre perdem. Estamos assim em busca do mínimo.> > >    1 ganha de 2. 2 perde de 3. 2 perde de 4. 2 está fora.> > >   1 ganha de 5. 5 perde de 6. 5 perde de 7. 5 está fora.> > >    1 ganha de 8. 8 perde de 9. 8 perde de 10.8 está fora.> > >    9 lutas. Restam 7 contentores. Renumerando-os, temos:> > >    1 ganha de 2. 2 perde de 3. 2 perde de 4. 2 está fora.> > >   1 ganha de 5. 5 perde de 6. 5 perde de 7. 5 está fora.> > >    15 lutas acumuladas. 5 contentores:> > >    1 ganha de 2. 2 perde de 3. 2 perde de 4. 2 está fora.> > >    1 ganha de 5. 5 perde de 3. 5 perde de 4. 5 está fora.> > >    1 ganha de 2. 2 perde de 3. 2 está fora. Aqui, com três> >lutadores,> > >razoável parece a quebra da regra: 2 saiu com duas derrotas.> > >    1 ganha de 2. 2 está fora. 1 é o campeão. Houve: 24 jogos. Esse>é> >o> > >mínimo.> > >    Agora, vamos a busca do máximo... (parece mais difícil). Bem,>se> > >distribuirmos o mais igualitariamente vitórias e derrotas, então,> > >atingiremos o máximo, cremos. Logo: 1 ganha de 2, que perde de 3, que> >perde> > >de 4, ... Hum: é um ciclo, com o ponteiro D (de derrota) apontando para> >os> > >jogadores. O torneio acaba quando cada jogador é apontado três vezes,>com> > >exceção de um, que é apontado duas vezes. Logo, a resposta é: 9.3 + 1.2>=> > >29.> > >    Fácil é inferir uma regra geral para o máximo, mas é para o> >mínimo?> > >    Bem, para o mínimo, vejamos: colocando-os em linha reta, e> > >renumerando-os a cada três jogos, ao final dos quais o segundo sempre> >sai,> > >até que fiquem três jogadores, a partir de quando, com três contendas> >acaba> > >o torneio. Então, uma regra geral para n jogadores é 3(n-3) + 3.> > >> > >Fraternalmente, João.> > >> > >> > >> > >> > >> > >> > >> > >> > >> > >Estou com duvidas neste problema, gostaria de propo-lo aos  colegas.> > >> > >> > >Em um torneio de judo hah 10 contendores. Cada luta prossegue ateh que>os> > >jurados declarem um vencedor, nunca hah empate. O contendor que perder>3> > >vezes (seguidas ou nao) eh eliminado. O torneio prossegue ateh que>reste> >um> > >unico contendor, que eh, entao, declarado campeao. Seja n o numero de> >lutas> > >realizadas ateh a declaracao do campeao. Qual o menor e qual o maior> >valor> > >que n pode assumir?> > >> > >> > >Abracos> > >Artur> > >> >> >=

Re: [obm-l] Análise combinatória - número de lutas

2007-07-20 Por tôpico JoaoCarlos_Junior
É que o problema necessita de uma retificação. Quando se chega a 3 participantes, duas disputas bastam para eliminar 1. E, com 2 participantes, basta uma disputa para eliminar o perdedor e definir o vencedor.[EMAIL PROTECTED] escreveu: -Para: obm-l@mat.puc-rio.brDe: "Qwert Smith" <[EMAIL PROTECTED]>Enviado por: [EMAIL PROTECTED]Data: 20/07/2007 10:13Assunto: Re: [obm-l] Análise combinatória - número de lutas??? de onde vc tirou 3(n-3)+3 pra minimo.Para eliminarmos n-1 participantes numa competicao onde a elimicao se da com d derrotas sao necessarias (n-1)*d partidas.>From: [EMAIL PROTECTED]>Reply-To: obm-l@mat.puc-rio.br>To: obm-l@mat.puc-rio.br>Subject: Re: [obm-l]  Análise combinatória - número de lutas>Date: Fri, 20 Jul 2007 09:48:53 -0400>>Bem, encontramos: mínimo: 24. Máximo: 29.E ainda, as regras gerais: mínimo: >3(n-3)+3  máximo: (n-1)*3+ 2>[EMAIL PROTECTED] escreveu: ->>Para: obm-l@mat.puc-rio.br>De: "Qwert Smith" <[EMAIL PROTECTED]>>Enviado por: [EMAIL PROTECTED]>Data: 20/07/2007 8:36>Assunto: Re: [obm-l] Análise combinatória - número de lutas>>Acho que o problema e bem mais simples que isso.>>Para que um lutador seja eliminado ele perde 3 vezes.  Para que 9 lutadores>sejam eliminados sao necessarias pelo menos 9 x 3 lutas.>>Logo o minimo e 27.>>O numero de lutas e sempre 27 + n.  'n' e o numero de lutas que o campeao>perdeu.  Mas o campeao so pode perder no maximo 2 lutas ou nao seria o>campeao.  Logo o maximo de lutas e 29.>>> >From: [EMAIL PROTECTED]> >Reply-To: obm-l@mat.puc-rio.br> >To: obm-l@mat.puc-rio.br> >Subject: Re: [obm-l]  Análise combinatória - número de lutas> >Date: Fri, 20 Jul 2007 08:15:50 -0400> >> >> >> >> >> >Tentativa> > > >    Bem, duas considerações preliminares: 1) 1 é imbatível; 2) Alguns> >outros sempre perdem. Estamos assim em busca do mínimo.> >    1 ganha de 2. 2 perde de 3. 2 perde de 4. 2 está fora.> >   1 ganha de 5. 5 perde de 6. 5 perde de 7. 5 está fora.> >    1 ganha de 8. 8 perde de 9. 8 perde de 10.8 está fora.> >    9 lutas. Restam 7 contentores. Renumerando-os, temos:> >    1 ganha de 2. 2 perde de 3. 2 perde de 4. 2 está fora.> >   1 ganha de 5. 5 perde de 6. 5 perde de 7. 5 está fora.> >    15 lutas acumuladas. 5 contentores:> >    1 ganha de 2. 2 perde de 3. 2 perde de 4. 2 está fora.> >    1 ganha de 5. 5 perde de 3. 5 perde de 4. 5 está fora.> >    1 ganha de 2. 2 perde de 3. 2 está fora. Aqui, com três >lutadores,> >razoável parece a quebra da regra: 2 saiu com duas derrotas.> >    1 ganha de 2. 2 está fora. 1 é o campeão. Houve: 24 jogos. Esse é >o> >mínimo.> >    Agora, vamos a busca do máximo... (parece mais difícil). Bem, se> >distribuirmos o mais igualitariamente vitórias e derrotas, então,> >atingiremos o máximo, cremos. Logo: 1 ganha de 2, que perde de 3, que >perde> >de 4, ... Hum: é um ciclo, com o ponteiro D (de derrota) apontando para >os> >jogadores. O torneio acaba quando cada jogador é apontado três vezes, com> >exceção de um, que é apontado duas vezes. Logo, a resposta é: 9.3 + 1.2 => >29.> >    Fácil é inferir uma regra geral para o máximo, mas é para o >mínimo?> >    Bem, para o mínimo, vejamos: colocando-os em linha reta, e> >renumerando-os a cada três jogos, ao final dos quais o segundo sempre >sai,> >até que fiquem três jogadores, a partir de quando, com três contendas >acaba> >o torneio. Então, uma regra geral para n jogadores é 3(n-3) + 3.> > > >Fraternalmente, João.> > > > > > > > > > > >> > > >> >> >Estou com duvidas neste problema, gostaria de propo-lo aos  colegas.> >> >> >Em um torneio de judo hah 10 contendores. Cada luta prossegue ateh que os> >jurados declarem um vencedor, nunca hah empate. O contendor que perder 3> >vezes (seguidas ou nao) eh eliminado. O torneio prossegue ateh que reste >um> >unico contendor, que eh, entao, declarado campeao. Seja n o numero de >lutas> >realizadas ateh a declaracao do campeao. Qual o menor e qual o maior >valor> >que n pode assumir?> >> >> >Abracos> >Artur> >> >=> >Instruções para entrar na lista, sair da lista e usar a lista em> >http://www.mat.puc-rio.br/~nicolau/olimp/obm-l.html> >=Instruções> >para entrar na lista, sair da lista e usar a lista em> >http://www.mat.puc-rio.br/~nicolau/olimp/obm-l.html> >>>_>Don't get caught with egg on your face. Play Chicktionary! >http://club.live.com/chicktionary.aspx?icid=chick_hotmailtextlink2>>=>Instruções para entrar na lista, sair da lista e usar a lista em>http://www.mat.

Re: [obm-l] Análise combinatória - número de lutas

2007-07-20 Por tôpico JoaoCarlos_Junior
Bem, encontramos: mínimo: 24. Máximo: 29.E ainda, as regras gerais: mínimo: 3(n-3)+3  máximo: (n-1)*3+ 2[EMAIL PROTECTED] escreveu: -Para: obm-l@mat.puc-rio.brDe: "Qwert Smith" <[EMAIL PROTECTED]>Enviado por: [EMAIL PROTECTED]Data: 20/07/2007 8:36Assunto: Re: [obm-l] Análise combinatória - número de lutasAcho que o problema e bem mais simples que isso.Para que um lutador seja eliminado ele perde 3 vezes.  Para que 9 lutadores sejam eliminados sao necessarias pelo menos 9 x 3 lutas.Logo o minimo e 27.O numero de lutas e sempre 27 + n.  'n' e o numero de lutas que o campeao perdeu.  Mas o campeao so pode perder no maximo 2 lutas ou nao seria o campeao.  Logo o maximo de lutas e 29.>From: [EMAIL PROTECTED]>Reply-To: obm-l@mat.puc-rio.br>To: obm-l@mat.puc-rio.br>Subject: Re: [obm-l]  Análise combinatória - número de lutas>Date: Fri, 20 Jul 2007 08:15:50 -0400>>Tentativa> >    Bem, duas considerações preliminares: 1) 1 é imbatível; 2) Alguns >outros sempre perdem. Estamos assim em busca do mínimo.>    1 ganha de 2. 2 perde de 3. 2 perde de 4. 2 está fora.>   1 ganha de 5. 5 perde de 6. 5 perde de 7. 5 está fora.>    1 ganha de 8. 8 perde de 9. 8 perde de 10.8 está fora.>    9 lutas. Restam 7 contentores. Renumerando-os, temos:>    1 ganha de 2. 2 perde de 3. 2 perde de 4. 2 está fora.>   1 ganha de 5. 5 perde de 6. 5 perde de 7. 5 está fora.>    15 lutas acumuladas. 5 contentores:>    1 ganha de 2. 2 perde de 3. 2 perde de 4. 2 está fora.>    1 ganha de 5. 5 perde de 3. 5 perde de 4. 5 está fora.>    1 ganha de 2. 2 perde de 3. 2 está fora. Aqui, com três lutadores, >razoável parece a quebra da regra: 2 saiu com duas derrotas.>    1 ganha de 2. 2 está fora. 1 é o campeão. Houve: 24 jogos. Esse é o >mínimo.>    Agora, vamos a busca do máximo... (parece mais difícil). Bem, se >distribuirmos o mais igualitariamente vitórias e derrotas, então, >atingiremos o máximo, cremos. Logo: 1 ganha de 2, que perde de 3, que perde >de 4, ... Hum: é um ciclo, com o ponteiro D (de derrota) apontando para os >jogadores. O torneio acaba quando cada jogador é apontado três vezes, com >exceção de um, que é apontado duas vezes. Logo, a resposta é: 9.3 + 1.2 = >29.>    Fácil é inferir uma regra geral para o máximo, mas é para o mínimo?>    Bem, para o mínimo, vejamos: colocando-os em linha reta, e >renumerando-os a cada três jogos, ao final dos quais o segundo sempre sai, >até que fiquem três jogadores, a partir de quando, com três contendas acaba >o torneio. Então, uma regra geral para n jogadores é 3(n-3) + 3.> >Fraternalmente, João.> > > > > >> >>>Estou com duvidas neste problema, gostaria de propo-lo aos  colegas.>>>Em um torneio de judo hah 10 contendores. Cada luta prossegue ateh que os >jurados declarem um vencedor, nunca hah empate. O contendor que perder 3 >vezes (seguidas ou nao) eh eliminado. O torneio prossegue ateh que reste um >unico contendor, que eh, entao, declarado campeao. Seja n o numero de lutas >realizadas ateh a declaracao do campeao. Qual o menor e qual o maior valor >que n pode assumir?>>>Abracos>Artur>>=>Instruções para entrar na lista, sair da lista e usar a lista em>http://www.mat.puc-rio.br/~nicolau/olimp/obm-l.html>=Instruções >para entrar na lista, sair da lista e usar a lista em>http://www.mat.puc-rio.br/~nicolau/olimp/obm-l.html>_Don't get caught with egg on your face. Play Chicktionary!  http://club.live.com/chicktionary.aspx?icid=chick_hotmailtextlink2=Instruções para entrar na lista, sair da lista e usar a lista emhttp://www.mat.puc-rio.br/~nicolau/olimp/obm-l.html=Instruções para entrar na lista, sair da lista e usar a lista em
http://www.mat.puc-rio.br/~nicolau/olimp/obm-l.html
===

Re: [obm-l] Análise combinatória - número de lutas

2007-07-20 Por tôpico JoaoCarlos_Junior
Tentativa Bem, duas considerações preliminares: 1) 1 é imbatível; 2) Alguns outros sempre perdem. Estamos assim em busca do mínimo.    1 ganha de 2. 2 perde de 3. 2 perde de 4. 2 está fora.   1 ganha de 5. 5 perde de 6. 5 perde de 7. 5 está fora.    1 ganha de 8. 8 perde de 9. 8 perde de 10.8 está fora.    9 lutas. Restam 7 contentores. Renumerando-os, temos:    1 ganha de 2. 2 perde de 3. 2 perde de 4. 2 está fora.   1 ganha de 5. 5 perde de 6. 5 perde de 7. 5 está fora.     15 lutas acumuladas. 5 contentores:    1 ganha de 2. 2 perde de 3. 2 perde de 4. 2 está fora.    1 ganha de 5. 5 perde de 3. 5 perde de 4. 5 está fora.    1 ganha de 2. 2 perde de 3. 2 está fora. Aqui, com três lutadores, razoável parece a quebra da regra: 2 saiu com duas derrotas.    1 ganha de 2. 2 está fora. 1 é o campeão. Houve: 24 jogos. Esse é o mínimo.    Agora, vamos a busca do máximo... (parece mais difícil). Bem, se distribuirmos o mais igualitariamente vitórias e derrotas, então, atingiremos o máximo, cremos. Logo: 1 ganha de 2, que perde de 3, que perde de 4, ... Hum: é um ciclo, com o ponteiro D (de derrota) apontando para os jogadores. O torneio acaba quando cada jogador é apontado três vezes, com exceção de um, que é apontado duas vezes. Logo, a resposta é: 9.3 + 1.2 = 29.    Fácil é inferir uma regra geral para o máximo, mas é para o mínimo?    Bem, para o mínimo, vejamos: colocando-os em linha reta, e renumerando-os a cada três jogos, ao final dos quais o segundo sempre sai, até que fiquem três jogadores, a partir de quando, com três contendas acaba o torneio. Então, uma regra geral para n jogadores é 3(n-3) + 3.  	Fraternalmente, João. 
 
Estou com duvidas neste problema, gostaria de propo-lo aos  colegas.Em um torneio de judo hah 10 contendores. Cada luta prossegue ateh que os jurados declarem um vencedor, nunca hah empate. O contendor que perder 3 vezes (seguidas ou nao) eh eliminado. O torneio prossegue ateh que reste um unico contendor, que eh, entao, declarado campeao. Seja n o numero de lutas realizadas ateh a declaracao do campeao. Qual o menor e qual o maior valor que n pode assumir?AbracosArtur=Instruções para entrar na lista, sair da lista e usar a lista emhttp://www.mat.puc-rio.br/~nicolau/olimp/obm-l.html=Instruções para entrar na lista, sair da lista e usar a lista em
http://www.mat.puc-rio.br/~nicolau/olimp/obm-l.html
===

Re: [obm-l] OPM-1979- 2ª Fase

2007-07-19 Por tôpico JoaoCarlos_Junior
Para cada um desses meses, dias distintos implicam somas diferentes.
Agora, a medida que se soma 1 ao mês m, iniciando por m=1, os valores das somas serão também todos distintos para cada dia. 
Ou seja, para cada dia do ano, não há soma igual, logo, nunca haverá dúvida de quando é a dada do aniversário.
Bom dia amigos, estou com dificuldades para resolver o item b desta questao,alguem tem uma ideia de como sair dela?Questao 5 - Peça a qualquer amigo que multiplique o dia de seu aniversariopor 12 e o mes do aniversario po 31 e some os dois resultados.a) Suponha que seu amigo seguiu suas instruçoes e a soma deu 368. Quando é oaniversario dele?b) Demostre que, dada a soma, a data é determinada de modo unico, isto é,dada a soma, nunca havera dúvida quando é o aniversario.Desde ja agradeço._Mande torpedos SMS do seu messenger para o celular dos seus amigoshttp://mobile.msn.com/=Instruções para entrar na lista, sair da lista e usar a lista emhttp://www.mat.puc-rio.br/~nicolau/olimp/obm-l.html=Instruções para entrar na lista, sair da lista e usar a lista em
http://www.mat.puc-rio.br/~nicolau/olimp/obm-l.html
===

Re: [obm-l] Comentários por favor

2007-07-18 Por tôpico JoaoCarlos_Junior
A cada duas partidas, ganhar e perder uma vez (em qualquer ordem) se obtém o mesmo resultado que com dois empates. No entanto, com essa última opção, tem-se o mínimo de derrotas, que é o desejado pelo problema.
Logo, como foram obtidos 24 pontos em 40 jogos, podemos, preliminarmente, supor, para facilitar o raciocínio que quer conduzir ao que se deseja, que tenha havido 24 empates (25 ou mais, não é possível, pois, por derrotas, pontos não são retirados). Então, 16 (40-24) são as derrotas. Pois, para o que se deseja, ocorreu: 0 vitórias, 24 empates e 16 derrotas.
Prezados. Segue uma questão que gostaria dos comentários dos amigos. Achei a resposta 16, mas a minha explicação não esta muito bem argumentada.Em um campeanoto de futebol, cada equipe recebe dois pontos por vitória, um ponto por empate e zero ponto por derrota. Sabendo que ao final do campeonato cada equipe disputou 40 partidas e que uma determinada equipe obteve 24 pontos, o número mínimo de derrotas sofridas por esta equipe foi:a) 28b) 16c) 15d) 14e) 12Grande abraço a todos.Marcelo Roseira.



Novo Yahoo! Cadê? - Experimente uma nova busca. Instruções para entrar na lista, sair da lista e usar a lista em
http://www.mat.puc-rio.br/~nicolau/olimp/obm-l.html
===

[obm-l] Re: Iberoamericana 2004

2007-07-16 Por tôpico JoaoCarlos_Junior




(Iberoamericana-2004). Considera-se no plano uma
circunferência de centro O e raio r, e um ponto A exterior a ela. Seja M um
ponto da circunferência e N o ponto diametralmente oposto a M. Determinar o
lugar geométrico dos centros das  circunferências que passam por A, M e N
quando M varia.

Tentativa de Solução

  C (centro da circunferência AMN) é o encontro das mediatrizes de MN e
NA.
Quando MN é perpendicular a AO, C estará no médio de AO. Seja P esse médio
fixo.
Se provarmos uma das condições a seguir, então o problema estará acabado.
As condições:
1)Os ângulos COP e CPA são iguais;
2)Os ângulos OCP e PCA são iguais;
Seja s a perpendicular a AO por O. COP é igual a MOT. T é uma das
interseções de s com a circunferência de centro O e raio r.
Resta provar que CAO é igual a MOT.
Naturalmente, o LG procurado tem simetria em relação a reta AO. Logo, o LG
contém C´, simétrico de C em relação a AO.
Ora, C´AO é igual a MOT, pois AC´ é perpendicular a MN; e AO, a OT.
Portanto, CAO é igual a MOT.
A reta CC´ é o LG procurado.

Fraternalmente, João.



=
Instruções para entrar na lista, sair da lista e usar a lista em
http://www.mat.puc-rio.br/~nicolau/olimp/obm-l.html
=


[obm-l] (OFF) Inquirição aos Professores Olímpicos dos Grandes Centros

2007-07-02 Por tôpico JoaoCarlos_Junior

Ilustríssimos Professores (residentes em grandes centros) de alunos olímpicos:
 
 Há em vós a intenção de atender a interesses (que supomos existir) de alunos olímpicos e interessados não-residentes em grandes centros, por meio da gravação de vossas aulas em meio eletrônico adequado, disponibilizadas na internet, mesmo que de forma onerosa? Há possibilidade de se efetivar tal intenção?
 
Fraternalmente, João.
 Instruções para entrar na lista, sair da lista e usar a lista em
http://www.mat.puc-rio.br/~nicolau/olimp/obm-l.html
===

Re: [obm-l] (OF TOPIC) Auxílio a brasileiros não-residentes em grandescentros

2007-06-25 Por tôpico JoaoCarlos_Junior
Ronaldo, obrigado:
   Sabe Ronaldo, outros, seria ótimo, realmente um sonho, ver em teleconferência, para diversas cidades, aulas dos professores de olímpiada dos grandes centros.
Fraternalmente, João.
Olá João.  Acho que uma solução seria semudar para tal localização.Em alguns casos,pessoas geniais conseguem progredir sozinhasapenas com pesquisando na Web/ baixandopapers em universidades e estudando  livros.Mas devemos lembrar que essas pessoas sãoa *exceção*.  Não a regra.Mesmo matemáticosque já se destacaram em olimpíadas precisamde orientador, pois dois cérebros pensam melhorque um ...[]sRonaldo[EMAIL PROTECTED] wrote:> Senhores:>> O que se realiza para minimizar a diferença entre o efetivo resultado em> olimpíadas de um jovem brasileiro não residente em grande centro e um> suposto resultado melhor, caso residisse em tal localização?>> Fraternalmente, João.>> => Instruções para entrar na lista, sair da lista e usar a lista em> http://www.mat.puc-rio.br/~nicolau/olimp/obm-l.html> ==Instruções para entrar na lista, sair da lista e usar a lista emhttp://www.mat.puc-rio.br/~nicolau/olimp/obm-l.html=Instruções para entrar na lista, sair da lista e usar a lista em
http://www.mat.puc-rio.br/~nicolau/olimp/obm-l.html
===

[obm-l] (OF TOPIC) Auxílio a brasileiros não-residentes em grandes centros

2007-06-22 Por tôpico JoaoCarlos_Junior




Senhores:

O que se realiza para minimizar a diferença entre o efetivo resultado em
olimpíadas de um jovem brasileiro não residente em grande centro e um
suposto resultado melhor, caso residisse em tal localização?

Fraternalmente, João.


=
Instruções para entrar na lista, sair da lista e usar a lista em
http://www.mat.puc-rio.br/~nicolau/olimp/obm-l.html
=


Re: [obm-l] RE: [obm-l] Olímpiada. Nível 2. Fase 3.

2007-06-20 Por tôpico JoaoCarlos_Junior
Prezado Ralph, muito obrigado. Efetivamente, simlpes e claro o teu raciocínio.
Fraternalmente, João.
-Original Message-From: Ralph TeixeiraSent: Thu 6/7/2007 3:35 PMTo: obm-l@mat.puc-rio.brCc:Subject: RE: [obm-l] Olímpiada. Nível 2. Fase 3.Eu gosto mais de fazer assim:Seja R o jogador (ou um dos, em caso de empate) que mais partidas ganhou. Vou mostrar que R ganhou de todo mundo...Caso contrario, teriamos RX somos forcados a ter S>X (caso contrario, seria RO raciocinio para o que mais perdeu eh analogo.Abraco,Ralph=Instruções para entrar na lista, sair da lista e usar a lista emhttp://www.mat.puc-rio.br/~nicolau/olimp/obm-l.html=Instruções para entrar na lista, sair da lista e usar a lista em
http://www.mat.puc-rio.br/~nicolau/olimp/obm-l.html
===

Re: [obm-l] Fw: Resultados da Cone Sul

2007-06-18 Por tôpico JoaoCarlos_Junior
Professor Shine, outros:
O que atualmente se realiza no intuito de se minimizar dificuldades relacionadas à origem dos estudantes brasileiros?
Fraternalmente, João.
Oi gente, recebi boas notícias lá do Uruguai! Parabéns a todos! []'sShine- Forwarded Message From: Yuri Lima <[EMAIL PROTECTED]>To: OCM-L <[EMAIL PROTECTED]>; [EMAIL PROTECTED]; [EMAIL PROTECTED]; Carlos Shine <[EMAIL PROTECTED]>; Olimpíada de Matemática <[EMAIL PROTECTED]>Sent: Saturday, June 16, 2007 8:03:46 PMSubject: [conesul2006] Resultados da Cone SulOlá Chicos, A XVIII Olimpíada de Matemática do Cone Sul está terminada, e tivemos um saldo de: - 1 ouro (Renan); - 3 pratas. As provas foram realizadas na quinta e na sexta e as coordenaccoes e distribuiccao de medalhas foram feitas hoje de manha. Houve uma situaccao inusitada, com 5 medalhas de ouro, 6 de prata e 6 de bronze. Os 4 peruanos sacaram ouro, o que acabou fazendo com que ficássemos em segundo lugar geral na classificaccao dos países, seguidos pelos argentinos em terceiro. Estamos enviando arquivos anexos com os 6 problemas da prova e as pontuaccoes de todos os medalhistas. De qualquer forma, as pontuaccoes do Brasil foram: Renan - 50Marcelo - 44Thiago - 43Grazyelly - 36. Os cortes das medalhas foram:Ouro - 50Prata - 33Bronze - 25. Tais cortes se devem a grande saltos entre as pontuaccoes do último de uma faixa e o primeiro da faixa seguinte. Vocês podem ver no arquivo anexo. Levando em conta a inexperiência e as origens de três dos participantes (Barreiras, Varginha e Joinville), achamos que foi um ótimo resultado. Um abracco a todos, Yuri e Samuel.



Pinpoint customers who are looking for what you sell. __._,_.___[IMAGE]Suas configurações de e-mail: E-mail individual |Tradicional Alterar configurações via web (Requer Yahoo! ID) Alterar configurações via e-mail: Alterar recebimento para lista diária de mensagens | Alterar para completo Visite seu grupo | Termos de Uso do Yahoo! Grupos | Descadastrado __,_._,___



Ready for the edge of your seat? Check out tonight's top picks on Yahoo! TV. Instruções para entrar na lista, sair da lista e usar a lista em
http://www.mat.puc-rio.br/~nicolau/olimp/obm-l.html
===

Re: [obm-l] Olímpiada. Nível 2. Fase 3.

2007-05-18 Por tôpico JoaoCarlos_Junior

Prezado Paulo Santa Rita:
 
  Como posso sentir amargura de tuas honestas palavras que buscam auxiliar-me. Tu que deixaste de realizar as atividades de teu interesse para ocupar-te com as minhas. E ainda, foste o único a tal ato.
  Em realidade, quando alguém sente amargor ao ouvir, tal sentimento não deriva diretamente do que ouve, mas de uma interpretação própria e particular do que escuta; essa interpretação, se implica tal sentido, é fruto do orgulho do próprio ouvinte, não está em quem explana. Portanto, obrigado.
  Estou tentando olhar o meu interior, como sugeres. E, com desassombro, exponho-o, pois, de forma distinta, não evoluo. Assim, digo: não tentei fazer demonstração por indução, mas buscar com k=3 e k=4 descobrir uma regra que me pudesse levar a uma demonstração direta para k=n.
  Eu sei que existem muitos casos, como, por exemplo, com k=3, conforme você exarou. No entanto, esses casos são todos semelhantes. Logo, creio que tratando de um, estarei tratando de todos, pois, os demais são repetição. As simbologias: J1, J2, J3 são genéricas, logo, há uma simetria entre elas. 
  Assim, quando concluí que só pode existir J1>J2, J2>J3 e J3J2>J3), sei que também há outros casos, mas esses outros estão inclusos nesse aí. Por exemplo: lógico que poderia ser J1>J2, J2J1 (J3>J1>J2), mas chamando J3 de A1, J1 de A2 e J2 de A3, teríamos: A1>A2>A3, que é a mesma coisa. É isso que estou tentando dizer. Veja que, sempre, posso reorganizar os termos simétricos com outra nomenclatura de forma que os números apresentem-se em seqüência crescente, que não é melhor nem pior que qualquer outra, apenas me facilita.
  Para mim, 1, 2, 3 não são: (um, dois, três), e sim: o primeiro, o segundo, o terceiro, sempre nessa ordem, fixa, independentemente do nome das variáveis. Não fico apegado à linguagem, creio que a ordem é de mais valia, ela implica uma espécie de justiça (simetria) entre os símbolos.
  É razoável essa idéia?
 
Muito grato, com sinceridade, sem fingimento.
João.
 
** 
 

 Ola Joao Carlos e demaiscolegas desta lista ... OBM-L,Releia a sua mensagem inicial ... Nela voce afirma que a UNICAHIPOTESE QUE NAO EXISTE e J1 > J2, J2 > J3 e J3> J1. Nao e verdadeisso. A hipotese adotada e que nao existe ciclos. Assim, tambem naopode existir, por exemplo : J3 > J2, J2 > J1 e J1 > J3. De maneirageral, com 3 jogadores e possivel formar seis ciclos, veja :J1 > J2 > J3 > J1J1 > J3 > J2 > J1J2 > J1 > J3 > J2J2 > J3 > J1 > J2J3 > J1 > J2 > J3J3 > J2 > J1 > J3Ate aqui, com grande generosidade, ainda da pra aceitar. O que eu naocompreendo e a conclusao ... voce diz :-- Logo, so pode existir, J1>J2,  J2 > J3 e J3 <  J1Acima mostrei que a sua suposicao inicial estava errada, pois ha seisciclos possiveis. Agora, afirmo que a sua conclusao tambem estaerrada, pois,  admitindo que A UNICA HIPOTESE QUE NAO EXISTE e  J1 >J2, J2 > J3 e J3> J1 nao se pode inferir que o unico resultadopossivel e J1>J2,  J2 > J3 e J3 <  J1 ... Tambem podem existir, porexemplo :J1 > J2, J2 < J3 e J3 > J1Entao, como voce ve, voce faz uma suposicao errada e tira umaconclusao errada. E nao para por ai ... Para que serviu esta conclusao? Para mostrar que J3 venceu todos os jogos e que J2 perdeu todos,confirmando assim a tese ? Se foi por isso voce deveria enunciarexplicitamente, por exemplo, assim :vemos que no caso de 3 jogadores a nossa tese se confirma.Agora, pergunto : para que analisar o caso com 4 jogadores ? Parareforcar o que ? Se admitimos que voce fez o caso 3 corretamente, onornal e natural seria supor a validade do caso para um N naturalqualquer e provar que isto implica a validade para o caso N+1, talcomo o Claudio Buffara fez. Assim, o caso de 4 jogadores e redundante.Portanto, diante de tantas idiossincrasias eu me senti desestimuladoem analisar mais profundamente a sua solucao ...Eu estava pesquisando semelhancas entre as estrutura das solucoes deequacoes diferencias quando vi a sua mensagem. Percebi nela um sincerodesejo de aprender e resolvi te responder. Se nalgum momento a minharesposta parecer amarga, por favor, desconsidere isso, pois nao e domeu carater. E que realmente eu ando bastante ocupado   e sem tempo,investigando muitas coisas para mim empolgantes ao mesmo tempo tendoque responder  as exigencias burocraticas do meu trabalho.A Matematica nao esta fora mas sim dentro de todos nos. Ela pre-existeaos nossos pensamentos  e fazer Matematica e apenas olhar dentro denos mesmos. Assim, aprenda a olhar o seu interior, a escutá-lo : Issoe Matematica.Um Abraco a TodosPaulo Santa Rita4,0D39,160507Em 16/05/07, [EMAIL PROTECTED]<[EMAIL PROTECTED]>escreveu:>>> Prezados Paulo Santa Rita e Cláudio Buffara:>> Agradeço a ambos pelas respostas.>> Gostaria, se possível, de saber qual parte é que não foi compreendida, pois,> necessito saber se fui pouco claro ou se estou enganando a mim mesmo.>> ATT. João=Instruções para 

Re: [obm-l] Olímpiada. Nível 2. Fase 3.

2007-05-16 Por tôpico JoaoCarlos_Junior
Prezados Paulo Santa Rita e Cláudio Buffara:
Agradeço a ambos pelas respostas.
Gostaria, se possível, de saber qual parte é que não foi compreendida, pois, necessito saber se fui pouco claro ou se estou enganando a mim mesmo.
ATT. João
 
Ola Joao Carlos e demaiscolegas desta lista ... OBM-L,Nao entendi a sua tentativa... outras pessoas ja apresentaramsolucoes, mas nao tem problemas voce ver outras. Segue abaixo osesbocos de duas outras solucoes :1) PRIMEIRA SOLUCAOVou usar a notacao que voce sugere, isto e, X < Y significando que Xperdeu para Y. Sejam A e B dois jogadores. Podemos supor, sem perda degeneralidade, que A < B. Facamos J1=A e J2=B. Assim : J1 < J2.EXPANSAO PARA A DIREITA( Suposicao : NENHUM JOGADOR VENCEU TODOS OS CONFRONTOS  )Como nenhum jogador venceu todos os confrontos, J2 perdeu para alguem.Seja J3 tal que J2 < J3. Logo : J1 < J2 < J3.  J3 perdeu para alguem.Não foi para J1 porque isto implicaria na existencia do ciclo  J1 < J2< J3 < J1 que, por hipotese, não pode existir. Seja portanto J4 talque J3 < J4. Assim : J1 < J2 < J3 < J4.E claro que podemos aplicar o mesmo raciocinio anterior a J4, valedizer, como nenhum jogador venceu todos os confrontos entao J4 perdeupara alguem e este alguem não pode ser J1 ou J2 porque isto implicariana existencia de ciclos. Assim, deve existir J5 tal que J4 < J5 eassim sucessivamente, o que e um absurdo, pois temos um numero finitode jogadores e este processo obviamente se estende infinitamente ...Logo, somos obrigados a admitir que algum jogador venceu todos osconfrontosEXPANSAO PARA A ESQUERDA(suposicao : NENHUM JOGADOR PERDEU TODOS OS CONFRONTOS )Como nenhum jogador perdeu  todos os confrontos, J1 venceu alguem.Seja J3 tal que  J3 < J1. Logo : J3 < J1 < J2 .  J3 venceu alguem. Nãofoi  J2 porque isto implicaria na existencia do ciclo  J2 < J3 < J1 escreveu:> Solicito, por gentileza, correção da resolução (ou tentativa de resolução)> da questão que segue. PROBLEMA 6>> Em um torneio de tênis de mesa (no qual nenhum jogo termina empatado), cada> um dos n participantes jogou uma única vez contra cada um dos outros.> Sabe-se que, para todo k > 2, não existem k jogadores J1, J2, ?, Jk tais que> J1 ganhou de J2, J2 ganhou de J3, J3 ganhou de J4, ?, Jk ? 1 ganhou de Jk,> Jk ganhou de J1. Prove que existe um jogador que ganhou de todos os outros e> existe um jogador que perdeu de todos os outros.>>> TENTATIVA DE RESOLUÇÃO             As hipóteses:>> 1)     Não há empate.>> 2)     Cada jogador joga uma e só uma vez com cada um dos outros.>> 3)     Sabe-se que, para todo k > 2, não existem k jogadores J1, J2, ?, Jk> tais que J1 ganhou de J2, J2 ganhou de J3, J3 ganhou de J4, ?, Jk ? 1 ganhou> de Jk, Jk ganhou de J1.>> A tese: Existe um jogador que ganhou de todos e um que perdeu de todos.>>             Bem, com três jogadores: J1, J2, J3. É sabido que a única> hipótese que não existe é: J1>J2, J2>J3 e J3> J1. Logo, só pode existir, sem> perda de generalidade: J1>J2, J2>J3 e J3< J1, ou seja, J1>J2>J3. Cabe> explicar que o símbolo ?>? utilizado significa, por exemplo: ?Jogador 1> ganhou do Jogador 2?.>>             Com quatro jogadores, não temos: J1>J2, J2>J3, J3> J4 e J4>J1.> Assim, podemos trocar um ou dois desses sinais de > para <.  Então, temos:> (>, >, >, <) ou (>, 

Re: [obm-l] Re:[obm-l] Olímpiada. Nível 2. Fase 3.

2007-05-14 Por tôpico JoaoCarlos_Junior
Prezado Cláudio: 
Obrigado pela dica, e, em realidade, pela aula. 
Por gentileza, se possível, aquela solução que eu dei é então particular e de pouca possibilidade de generalização para problemas desse tipo? É isso?
Fraternalmente, João.
Uma idéia é usar teoria (elementar) dos grafos e demonstrar a proposição por indução no número de vértices. Essa é uma técnica muito utilizada (em teoria dos grafos) e, portanto, vale a pena tê-la em mente na hora de uma prova (especialmente de olimpíada). Além disso, a linguagem de grafos é muito útil na hora de visualizar o problema (afinal, o que pode ser mais básico do que vértices e arestas, ou seja, pontos e linhas?) Se existem n jogadores, você pode representar o torneio por um grafo orientado com n vértices (numerados de 1 a n) e tal que, dados quaisquer i <> j, se o jogador i venceu o jogador j então existe uma seta (aresta orientada) indo do vértice i ao vértice j. A condição do enunciado implica que o grafo não possui ciclos orientados, ou seja, vértices distintos i_1, i_2, ..., i_k (k>=3) com setas indo de i_1 para i_2, i_2 para i_3, ..., i_(k-1) para i_k e i_k para i_1. O problema é provar que existe um vértice de onde partem n-1 setas (uma fonte) e um vértice onde chegam n-1 setas (um dreno). Tomemos inicialmente n = 3.Se não existir uma fonte, então cada vértice tem pelo menos uma seta chegando. Se algum vértice tiver duas setas chegando, este será um dreno.Mas, nesse caso, a terceira seta do grafo, que liga os outros dois vértices, irá (obviamente) partir de um deles. Este será a fonte. Por outro lado, se cada vértice tiver uma seta partindo e uma chegando, então teremos um ciclo, o que é proibido pelo enunciado. Logo, o caso n = 3 está provado. Tomemos agora n >= 3 e suponhamos (hipótese de indução) que o resultado seja verdadeiro para grafos com até n vértices. Considere um grafo com n+1 vértices.Suponhamos que nenhum vértice seja uma fonte ou um dreno.Retire temporariamente um vértice qualquer e as n setas que chegam a ele ou partem dele, e considere o sub-grafo de n vértices resultante.Pela hipótese de indução, este grafo possui uma fonte F e um dreno D. Recoloque agora o vértice V que você retirou.Se ele recebe uma seta de F e manda uma seta para D, acabou: F e D são a fonte e o dreno do grafo maior (com os n+1 vértices). Caso contrário, temos três alternativas a considerar:1) V manda uma seta para F e recebe uma seta de D:Nesse caso, como F manda uma seta para D, FDVF é um ciclo.Mas isso contraria o enunciado. Logo, esse caso não ocorre; 2) V manda setas para F e para D:Nesse caso, D é o dreno do grafo maior.Se V receber alguma seta de algum vértice W, então WVFW é um ciclo, pois F é a fonte do subgrafo e, portanto, manda uma seta para W.Esta contradição mostra que este caso também não ocorre. 3) V recebe setas de F e de D:Esse caso é análogo ao anterior. Basta inverter o sentido das setas. Assim, vemos que a única possibilidade é que a fonte e o dreno do subgrafo sejam a fonte e o dreno do grafo maior. Logo, pelo princípio da indução, o resultado vale para qualquer grafo. Ou seja, num torneio onde todo mundo joga com todo mundo, se vale a "lógica" (ou seja, se não existem ciclos - situações onde A vence B, que vence C, que vence A), então tem um jogador que vence todo mundo e outro que perde pra todo mundo. []s,Claudio.  



De:
[EMAIL PROTECTED]



Para:
obm-l@mat.puc-rio.br



Cópia:




Data:
Fri, 11 May 2007 08:08:26 -0400



Assunto:
[obm-l] Olímpiada. Nível 2. Fase 3.
Solicito, por gentileza, correção da resolução (ou tentativa de resolução) da questão que segue. PROBLEMA 6Em um torneio de tênis de mesa (no qual nenhum jogo termina empatado), cada um dos n participantes jogou uma única vez contra cada um dos outros. Sabe-se que, para todo k > 2, não existem k jogadores J1, J2, ?, Jk tais que J1 ganhou de J2, J2 ganhou de J3, J3 ganhou de J4, ?, Jk ? 1 ganhou de Jk, Jk ganhou de J1. Prove que existe um jogador que ganhou de todos os outros e existe um jogador que perdeu de todos os outros. TENTATIVA DE RESOLUÇÃO As hipóteses:1) Não há empate.2) Cada jogador joga uma e só uma vez com cada um dos outros.3) Sabe-se que, para todo k > 2, não existem k jogadores J1, J2, ?, Jk tais que J1 ganhou de J2, J2 ganhou de J3, J3 ganhou de J4, ?, Jk ? 1 ganhou de Jk, Jk ganhou de J1.A tese: Existe um jogador que ganhou de todos e um que perdeu de todos.    Bem, com três jogadores: J1, J2, J3. É sabido que a única hipótese que não existe é: J1>J2, J2>J3 e J3> J1. Logo, só pode existir, sem perda de generalidade: J1>J2, J2>J3 e J3< J1, ou seja, J1>J2>J3. Cabe explicar que o símbolo ?>? utilizado significa, por exemplo: ?Jogador 1 ganhou do Jogador 2?.    Com quatro jogadores, não temos: J1>J2, J2>J3, J3> J4 e J4>J1. Assim, podemos trocar um ou dois desses sinais de > para <.  Então, temos: (>, >, >, <) ou (>, >, <, <). Assim, no primeiro caso, J4 será o que perdeu de todos; no segundo, será J3; e em ambos, J1 ganhou de t

[obm-l] Olímpiada. Nível 2. Fase 3.

2007-05-11 Por tôpico JoaoCarlos_Junior
Solicito, por gentileza, correção da resolução (ou tentativa de resolução) da questão que segue. PROBLEMA 6Em um torneio de tênis de mesa (no qual nenhum jogo termina empatado), cada um dos n participantes jogou uma única vez contra cada um dos outros. Sabe-se que, para todo k > 2, não existem k jogadores J1, J2, ?, Jk tais que J1 ganhou de J2, J2 ganhou de J3, J3 ganhou de J4, ?, Jk ? 1 ganhou de Jk, Jk ganhou de J1. Prove que existe um jogador que ganhou de todos os outros e existe um jogador que perdeu de todos os outros. TENTATIVA DE RESOLUÇÃO As hipóteses:1) Não há empate.2) Cada jogador joga uma e só uma vez com cada um dos outros.3) Sabe-se que, para todo k > 2, não existem k jogadores J1, J2, ?, Jk tais que J1 ganhou de J2, J2 ganhou de J3, J3 ganhou de J4, ?, Jk ? 1 ganhou de Jk, Jk ganhou de J1.A tese: Existe um jogador que ganhou de todos e um que perdeu de todos.    Bem, com três jogadores: J1, J2, J3. É sabido que a única hipótese que não existe é: J1>J2, J2>J3 e J3> J1. Logo, só pode existir, sem perda de generalidade: J1>J2, J2>J3 e J3< J1, ou seja, J1>J2>J3. Cabe explicar que o símbolo ?>? utilizado significa, por exemplo: ?Jogador 1 ganhou do Jogador 2?.    Com quatro jogadores, não temos: J1>J2, J2>J3, J3> J4 e J4>J1. Assim, podemos trocar um ou dois desses sinais de > para <.  Então, temos: (>, >, >, <) ou (>, >, <, <). Assim, no primeiro caso, J4 será o que perdeu de todos; no segundo, será J3; e em ambos, J1 ganhou de todos.Com n jogadores: É sabido que não temos: (>,>,>, ... , >). Entre esses parêntesis, há n ?>?. Com n par, podemos trocar ?>? para ?   Instruções para entrar na lista, sair da lista e usar a lista em
http://www.mat.puc-rio.br/~nicolau/olimp/obm-l.html
===

Re: [obm-l] SOMA

2006-12-29 Por tôpico JoaoCarlos_Junior
Na realidade, desmembrando estes termos, podemos escrevê-los todos em forma de um triângulo retângulo, em que cada linha é uma PG de razão 2, e cuja primeira linha tem 100 termos, e as seguintes tem um termo a menos que a imediatamente anterior. Bem, é melhor desenhar o triângulo (sorriso), então:
 
1  2^1  2^2 ... 2^99
   2^1  2^2 ... 2^99
    2^2 ... 2^99
    ...
    2^99
 
  Ora, se nós preenchermos os termos restantes dessa matriz, então teremos 100 PG´s de soma idêntica, e para encontramos o resultado desejado, basta dividir por dois a soma total dos termos dessa matriz. Assim, a resposta é 1/2*100*(2^100-1)/2 = 25*(2^100-1), salvo melhor juízo.Alguem pode me ajudar nessa?Achar a soma S= 1 + 2.2 + 3.2^2 + 4.2^3 + 5.2^4 + ... + 100. 2^99=Instruções para entrar na lista, sair da lista e usar a lista emhttp://www.mat.puc-rio.br/~nicolau/olimp/obm-l.html=Instruções para entrar na lista, sair da lista e usar a lista em
http://www.mat.puc-rio.br/~nicolau/olimp/obm-l.html
===

[obm-l] (OFF) Conjunto onde vale o Teorema do Valor Intermediário

2006-09-28 Por tôpico JoaoCarlos_Junior





"Ralph Teixeira" <[EMAIL PROTECTED]>Enviado Por: [EMAIL PROTECTED] 
26/09/2006 07:38 PM

Favor responder a obm-l@mat.puc-rio.br



Para   



cc   



cco   



Assunto   
[obm-l] RES: [obm-l] RES: [obm-l] Conjunto onde vale o Teorema do Valor Intermediário



Somente os mais belos professores assumem tal grau de humildade...
A humildade é tão ou mais valiosa que o próprio inletecto, além de implicar liberdade.
A afirmação direta do professor Ralph, de quem fui aluno: cuja resposta eu ainda não sei, demonstra conjugação de ambas as virtudes supracitadas e, por corolário, elevação moral.
Parabéns ao sempre querido professor, de quem jamais esquecer-me-ei. João
Não... Tome f(x)=1.96 para x<1.4 e f(x)=x^2 para x>1.4. Então f é contínua em X, mas o TVI não vale para f(1.4)<2Acho que eu topo mudar meu TVI para "se f:X em R é contínua". Fica um pouco mais fácil de mexer com as coisas pois o contradomínio não está limitado a X (por exemplo, agora dá para ver mais fácil que qualquer número algébrico está em X) mas mesmo assim não sei se X=R.Abraço,Ralph-Mensagem original-De: [EMAIL PROTECTED] [mailto:[EMAIL PROTECTED]]Emnome de Artur Costa SteinerEnviada em: terça-feira, 26 de setembro de 2006 13:46Para: obm-l@mat.puc-rio.brAssunto: [obm-l] RES: [obm-l] Conjunto onde vale o Teorema do ValorIntermediárioOlhei rapidinho, mas acho que nao precisa ser X = R nao. Se I for umconjunto finito de irracionais, entao X = R - I satisfaz aa sua condicao,certo? Por exemplo X = (-oo , raiz(2)) U (raiz(2) , oo)Artur-Mensagem original-De: [EMAIL PROTECTED] [mailto:[EMAIL PROTECTED]]Emnome de Ralph TeixeiraEnviada em: terça-feira, 26 de setembro de 2006 12:05Para: obm-l@mat.puc-rio.brAssunto: [obm-l] Conjunto onde vale o Teorema do Valor IntermediárioUma aqui para vocês (cuja resposta eu ainda não sei).Seja X um conjunto contendo os racionais e contido em R. Suponha quevale o TVI em X, isto eh, se f:X em X é contínua e f(a)em X) então existe x em (a,b) (e em X) tal que f(x)=c. Aposto que X=R... mascomo provar isto?Abraço,Ralph=Instruções para entrar na lista, sair da lista e usar a lista emhttp://www.mat.puc-rio.br/~nicolau/olimp/obm-l.html==Instruções para entrar na lista, sair da lista e usar a lista emhttp://www.mat.puc-rio.br/~nicolau/olimp/obm-l.html==Instruções para entrar na lista, sair da lista e usar a lista emhttp://www.mat.puc-rio.br/~nicolau/olimp/obm-l.html=Instruções para entrar na lista, sair da lista e usar a lista em
http://www.mat.puc-rio.br/~nicolau/olimp/obm-l.html
===

[obm-l] (OFF) Olímpicos Não Residentes em Capitais

2006-07-29 Por tôpico JoaoCarlos_Junior
Fico de certo modo entristecido e pensativo, ao observa que muitos dos nossos talentosos meninos e meninas que residem fora das capitais e de outras cidades beneficiadas com a presença de grandes mestres conseguem ir muitíssimo bem até o segundo nível das olimpíadas, sétima e oitava séries.Ao chegar no terceiro nível dessa competição, relativa ao ensino médio, aqueles talentos inatos, distantes da presença de um grande mestre, não conseguem progredir como outros, que, anteriormente, revelaram-se não tão bons quanto esses primeiros.Não digo isso para inculpar alguém ou algum professor, mas sim por iniciar uma tentativa de se pensar em uma forma de fazer com que esses também possam ser assistidos de alguma forma, e, portanto, ter suas qualidades melhor desenvolvidas, em prol deles próprios, de todos nós e do próprio país. Fraternalmente, João. Instruções para entrar na lista, sair da lista e usar a lista em
http://www.mat.puc-rio.br/~nicolau/olimp/obm-l.html
===

Res:[obm-l] RE: [obm-l] Segunda Fase, Nível 1, Parte B da XXVII OBM

2006-07-28 Por tôpico JoaoCarlos_Junior
Agradeço sinceramente a ambos os professores:  Shine e Pedro. João. [EMAIL PROTECTED] escreveu: -Para: obm-l@mat.puc-rio.brDe: "Pedro Cardoso" <[EMAIL PROTECTED]>Enviado por: [EMAIL PROTECTED]Data: 27/07/2006 12:51Assunto: [obm-l] RE: [obm-l] Segunda Fase, Nível 1, Parte B da XXVII OBMProblema: Considere tres numeros inteiros positivos consecutivos de tres algarismos tais que o menor e multiplo de 7, o seguinte e multiplo de 9 e o maior de 11. Escreva todas as sequencias de numeros que satisfazem essas propriedades. É bom adiantar que a solução do Carlos Shine é bem mais eficiente por ser mais curta, mas eu mesmo assim mando a minha.Sejam n, n+1 e n+2 os números dessa sequência.n = 7a; n+1 = 9b; n+2 = 11c7a+1 = 9b => 9b - 7a = 1.  Um par que é solução dessa equação é {4;5}.Genericamente, a = 5 + 9k; b = 4 + 7k.(qualquer valor de 'a' gerado por 5+9k gera um múltiplo de 7 cujo sucessor é múltiplo de 9)7a+2 = 11c => 11c -7a = 2. Um par que é solução dessa equação é {4;6}.Genericamente, a = 6 + 11m; b = 4 + 7m.(qualquer valor de 'a' gerado por 6+11m gera um múltiplo de 7 cujo sucessor do sucessor (7a+2) é múltiplo de 11)Daí tiramos que a = 5+9k = 6+11m => 9k - 11m = 1. O par {5;4} é solução, e, genericamente...k = 5 + 11j; m = 4 + 7j.Substituindo k em a = 5+9k, temos a = 5 + 9(5+11j) = 50 + 99j.Como n = 7a, n = 350 + 693j. A única solução entre 100 e 1000 (ou seja, com números de três algarismos) é n = 350, pois, para c = 1, temos n = 1043 (e a sequência seria 1043, 1044, 1045). Então, a única sequência que satisfaz essa propriedade é 350, 351, 352.Pedro Lazéra Cardoso_Acompanhe os desfiles do evento São Paulo Fashion Week. !=Instruções para entrar na lista, sair da lista e usar a lista emhttp://www.mat.puc-rio.br/~nicolau/olimp/obm-l.html=Instruções para entrar na lista, sair da lista e usar a lista em
http://www.mat.puc-rio.br/~nicolau/olimp/obm-l.html
===

[obm-l] Segunda Fase, Nível 1, Parte B da XXVII OBM

2006-07-25 Por tôpico JoaoCarlos_Junior
Pergunta: eh possivel continuar essa tentativa de solucao, sem sair no braco? Problema: Considere tres numeros inteiros positivos consecutivos de tres algarismos tais que o menor e multiplo de 7, o seguinte e multiplo de 9 e o maior de 11. Escreva todas as sequencias de numeros que satisfazem essas propriedades. Tentativa de resolucao Bem, os primeiros numeros maiores que 100, multiplos de 7, 9 e 11 sao, respectivamente, 105, 108 e 110.Ora, as diferencas (distancia) entre os multiplos de 9 e 7 sao, desde a origem: 3,5,0,2,4,6,1, as quais se repetem nessa mesma ordem recursivamente.Ja para 11 e 7, as diferencas (distancias), tambem desde a origem, sao: 5,2,6,3,0,4,1.Ora colocando as diferencas uma em baixo da outra, temos:5,2,6,3,0,4,1.3,5,0,2,4,6,1.O 1 em baixo do 1, isso ajuda? Mas podem nao estar numa sequencia consecutiva. Como continuar sem sair no braco?Instruções para entrar na lista, sair da lista e usar a lista em
http://www.mat.puc-rio.br/~nicolau/olimp/obm-l.html
===

Res:Re: [obm-l] Resultado da IMO 2006

2006-07-18 Por tôpico JoaoCarlos_Junior
Prezado professor Shine ou outros: Na foto dos nossos representantes, por gentileza, identifique-me quem é quem. ATT. João[EMAIL PROTECTED] escreveu: -Para: obm-l@mat.puc-rio.brDe: Carlos Yuzo Shine <[EMAIL PROTECTED]>Enviado por: [EMAIL PROTECTED]Data: 16/07/2006 15:52Assunto: Re: [obm-l] Resultado da IMO 2006Bom, eu já tinha enviado para a lista, mas no siteoficial tem o PDF com as provas assim como os alunos areceberam:Primeiro dia: http://imo2006.dmfa.si/day1/por.pdfSegundo dia: http://imo2006.dmfa.si/day2/por.pdfQuem quiser se aventurar em outros idiomas:http://imo2006.dmfa.si/problems.htmlResultados: http://imo2006.dmfa.si/results.htmlO site é muito bom, vale a pena ver.Ah, a nossa equipe aparece em uma das fotos:http://imo2006.dmfa.si/images/20060713/slika06.jpg(a legenda é "Cheerful Brazilian Team" :) )[]'sShine__Do You Yahoo!?Tired of spam?  Yahoo! Mail has the best spam protection around http://mail.yahoo.com =Instruções para entrar na lista, sair da lista e usar a lista emhttp://www.mat.puc-rio.br/~nicolau/olimp/obm-l.html=Instruções para entrar na lista, sair da lista e usar a lista em
http://www.mat.puc-rio.br/~nicolau/olimp/obm-l.html
===

[obm-l] Re: PELO SIM, PELO NÃO! (OFF)

2005-09-21 Por tôpico JoaoCarlos_Junior


Ponce:

Não nos incomodas!
Tens equivocada presunção sobre isso.

Teus escritos são auxílio.
Não receies.
João.






Rogerio Ponce <[EMAIL PROTECTED]>
Enviado Por: [EMAIL PROTECTED]
20/09/2005 17:35
Favor responder a obm-l
        
        Para:
       obm-l@mat.puc-rio.br
        cc:
       
        Assunto:
       Re: [obm-l] PELO SIM, PELO NÃO!


Olá pessoal,
quase não aguento mais esse problema, mas não dá para
ignorar que minhas perguntas podem ser simplificadas,
da seguinte forma:

Pergunte a "A":
- Se daqui a duas perguntas, eu lhe perguntar
     "Existe um honesto entre tais fulanos?"
  você me responderá um SIM?

A resposta obtida sempre será a indicação correta, e a
pesquisa binária pode ser feita sem dificuldade
alguma.

Portanto, com 3 perguntas assim, é possível localizar
o honesto dentre 8 participantes.

[]'s
Rogerio Ponce

PS: Juro que não escrevo mais nada a respeito desta
questão.


-

--- Rogerio Ponce escreveu:

> Olá Nicolau,
> na verdade, dá para superpor duas vezes (em cada
> pergunta) a política que eu sugeri, de modo a sempre
> obter a verdade.
> 
> Em outras palavras, se com 2 perguntas aninhadas, a
> gente consegue um "inversor", com 4 perguntas
> aninhadas, a gente sempre obtém a verdade.
> 
> E então, mesmo sem a auto-referência, a gente
> consegue
>  distinguir o honesto entre 8 pessoas, fazendo uma
> pesquisa binária desde o início.
> 
> O exemplo é um "pouquinho" enrolado, mas acho que
> funciona:
> 
> Pergunte a "A":
> - Se minha próxima pergunta a você for
>      "Se minha próxima pergunta a você for
>         ""Se minha próxima pergunta
a você for
>           """Existe um honesto
entre tais
> fulanos?"""
>         você me responderá um SIM?""
>      você me responderá um SIM?"
>   você me responderá um SIM?
> 
> 
> []s,
> Rogerio Ponce
> 
> 
> 
> --- Rogerio Ponce <[EMAIL PROTECTED]>
> escreveu:
> 
> > Olá Nicolau,
> > esse solução (resolvendo para 8) também é
> > interessante
> > - aliás, é A MAIS INTERESSANTE -, apesar de eu
> > também
> > achar um pouco "apelativa" pela auto-referência.
> > 
> > 
> > O que imaginei anteriormente, resolveria apenas
> para
> > 5
> > participantes (A,B,C,D,E), da seguinte forma:
> > 
> > Pergunte a "A":
> >  - Se minha próxima pergunta a você for "Existe
> > apenas
> > 1 honesto entre vocês?" , você me responderá um
> > "SIM"?
> > 
> > O honesto responderá SIM, e um desonesto
> responderá
> > NÃO. Supondo que "A" seja desonesto, agora você
> faz
> > a
> > seguinte pergunta a "A":
> > 
> > - Se minha próxima pergunta a você for "O honesto
> se
> > encontra entre B e C?" , você me responderá um
> > "SIM"?
> > 
> > Se a resposta for "NÃO" , então o honesto é B ou
> C.
> > Caso contrário, o honesto é D ou E.
> > Supondo que tenha respondido "NÃO", agora você
> > pergunta a "A":
> > 
> > - Se minha próxima pergunta a você for "O honesto
> é
> > B?", você me responderá um "SIM"?
> > 
> > Se a resposta for "NÃO" , o honesto é B, caso
> > contrário é C.
> > 
> > As outras derivações se resolvem do mesma modo,
> > sempre
> > usando a "dupla filtragem pelo desonesto", de
> forma
> > a
> > sempre obter a resposta invertida.
> > 
> > 
> > Mas como falei, essa minha solução ficou "na
> > poeira",
> > pois só consegue resolver para 5 pessoas...
> > 
> > []s,
> > Rogerio Ponce.
> > 
> > 
> > --- "Nicolau C. Saldanha" <[EMAIL PROTECTED]>
> > escreveu:
> > 
> > > On Wed, Sep 14, 2005 at 09:59:01PM -0300,
> Rogerio
> > > Ponce wrote:
> > > > Olá Nicolau,
> > > > sua solução é bonita porque resolve para
> > qualquer
> > > número de pessoas.
> > > > Mas, e se todos (como sugeriu o Chicão) só
> > puderem
> > > responder "sim" ou "não" a
> > > > qualquer questão?
> > > >  
> > > > Parece-me que - neste caso de apenas 5
> > > participantes - ainda é possível
> > > > resolver com apenas 3 perguntas.
> > > 
> > > Acho que dá até com 8 participantes, mas só com
> um
> > > pouco de apelação.
> > > Digamos que os participantes se chamam
> > > 000, 001, 010, 011, 100, 101, 110, 111.
> > > As perguntas seriam:
> > > 
> > > "Considere a seguinte afirmação:
> > > 'A sua resposta para esta pergunta será
> verdadeira
> > > se e somente se
> > > o primeiro algarismo do nome do honesto é 1.';
> > > a afirmação é verdadeira?"
> > > 
> > > É fácil verificar que se a resposta for SIM
> (resp.
> > > NÃO)
> > > então o primeiro algarismo do nome do honesto é
> 1
> > > (resp. 0),
> > > independentemente da resposta ser verdadeira ou
> > > falsa.
> > > Isto é parecido com o truque apresentado pelo
> Gugu
> > > mas um pouco diferente
> > > (e eu acho que agora correto). Note que a
> pergunta
> > é
> > > duplamente
> > > problemática: é auto-referente e pergunta sobre
> o
> > > futuro.
> > > É muito fácil com este tipo de 'golpe baixo'
> > > produzir perguntas
> > > irrespondíveis, como
> > > "Considere a seguinte afirmação:
> > > 'A sua resposta para esta pergunta será
> verdadeira
> > > se e somente se
> > > a sua resposta será NÃO.';
> > > e afi

Re: [obm-l] PELO SIM, PELO NÃO!

2005-09-19 Por tôpico JoaoCarlos_Junior


Professor Nicolau: 
Em continuidade, percebi que o segundo
e-mail enviado pelo senhor sobre o assunto é a resposta ao meu questionamento.
De qualquer forma, obrigado. João.







"Nicolau C. Saldanha" <[EMAIL PROTECTED]>
Enviado Por: [EMAIL PROTECTED]
18/09/2005 07:56
Favor responder a obm-l
        
        Para:
       obm-l@mat.puc-rio.br
        cc:
       
        Assunto:
       Re: [obm-l] PELO SIM, PELO NÃO!


On Fri, Sep 16, 2005 at 10:50:57AM -0400, [EMAIL PROTECTED]
wrote:
>         O não-entendimento é referente ao trecho
em azul, pois, creio que 
> o primeiro parágrafo é suficiente a refutação.

Em azul para você. Não suponha que todo mundo veja as mensagens com as
mesmas
cores que você: isto é falso para mim e acho que muito longe de ser verdade
para a maioria.

Ou seja: sinto muito, não entendi nada do que você tentou perguntar.

[]s, N.
=
Instruções para entrar na lista, sair da lista e usar a lista em
http://www.mat.puc-rio.br/~nicolau/olimp/obm-l.html
=



Re: [obm-l] PELO SIM, PELO NÃO!

2005-09-16 Por tôpico JoaoCarlos_Junior


Professor Nicolau ou professor Gugu:

        Estou
reestudando a questão desde o princípio, e já surgiu-me um não-entendimento,
o qual transcrevo a seguir.
        Na
primeira mensagem do Professor Nicolau, este colocou:
        “Eu
discordo desta interpretação. Digamos que os candidatos estejam arrumados
assim: d,d,h,d,d (onde h é o honesto e d não) e que você se faça esta pergunta.
Ao primeiro da fila. Mesmo se interpretarmos que ele já decidiu que é hora
de mentir e que perguntado diretamente ele responderá h,h,d,h,h, ele pode
responder, por exemplo, “h,d,h,h,h”: ele estará mentindo e você não descobriu
nada (ou tira conclusão errada).
Acho que esta solução se aplica
a perguntas com resposta “sim ou “não” e mesmo assim não tenho certeza
se se aplica a este problema. Não entendo que o enunciado deixe claro que
exista uma “hora de mentir” predeterminada ante de você formular a primeira
pergunta. Ou seja,
os desonestos podem decidir se vão mentir ou não na primeira pergunta em
função da pergunta, arruinando este truque.”
        
        O
não-entendimento é referente ao trecho em azul, pois, creio que o primeiro
parágrafo é suficiente a refutação. Já o trecho em azul não se assemelha
a esse primeiro parágrafo e, assim, não faz parte da refutação. De outra
forma, creio eu um pouco confuso: o momento de decidir se vão mentir ou
não na primeira pergunta é assunto distinto da forma como podem mentir.
Estou certo nisso?

Desde já, muito grato,
João.


Re: [obm-l] PELO SIM, PELO

2005-09-16 Por tôpico JoaoCarlos_Junior


A discussão entre os professores foi-me
por demais positiva.
Obrigado a ambos. É assim que progredimos,
opiniões expostas com franqueza, sem sentimentalismo ou pusilanimidade
nos que conversam naturalmente.
Reitero meu agradecimento a ambos.

ATT.
João.







"Nicolau C. Saldanha" <[EMAIL PROTECTED]>
Enviado Por: [EMAIL PROTECTED]
15/09/2005 16:11
Favor responder a obm-l
        
        Para:
       obm-l@mat.puc-rio.br
        cc:
       
        Assunto:
       Re: [obm-l] PELO SIM, PELO


On Thu, Sep 15, 2005 at 03:33:28PM -0300, [EMAIL PROTECTED]
wrote:
>    Oi Nicolau,
> Você está sendo coerente, mas é possível interpretar de outra maneira
- se a
> resposta de um mentiroso compulsório consta de várias afirmações,
todas elas
> devem ser falsas. Na verdade eu acho esse problema meio mal formulado
por isso
> mesmo. As perguntas (e as respostas) podem ser arbitrariamente complexas
?

Eu acho que finalmente entendi o seu ponto de vista, mas continuo achando
que esta não é a noção usual de mentira. Quando você faz *uma* pergunta,
qualquer coisa diferente da verdade é uma mentira e esta sua idéia de
negar cada bit em geral nem faz sentido. Assim, se a pergunta for
"Quais destes senhores (A,B,C,D,E) são honestos?" e se de fato
só o B for
honesto a resposta "Só o C é honesto." é uma mentira. Pelo seu
ponto
de vista, é uma 3/5-verdade pq A, D, e E são de fato desonestos e
a resposta também indica que eles são desonestos. Acho este seu ponto
de vista criticável sob vários aspectos: na vida real não se usa assim
a palavra "mentira", estes problemas de "quantas perguntas"
ficam severamente
alterados pq é possível socar um monte de perguntas em uma só, e o problema
em questão fica trivial, basta fazer a pergunta "Quais destes senhores
(A,B,C,D,E) são honestos?": uma resposta mentirosa no seu sentido
é
imediatamente reconhecível e deve indicar quatro honestos (os desonestos)
e um desonesto (o honesto).

Uma outra forma de argumentar a favor do meu ponto de vista é que *uma*
resposta nunca consta de várias afirmações, ela sempre consta de uma
única afirmação obtida juntando as subafirmações por "e".

Por outro lado a intenção do problema aparentemente era que as perguntas
fossem respondidas por SIM ou NÃO.

[]s, N.
=
Instruções para entrar na lista, sair da lista e usar a lista em
http://www.mat.puc-rio.br/~nicolau/olimp/obm-l.html
=



[obm-l] (OFF TOPIC) Astronomia

2005-09-06 Por tôpico JoaoCarlos_Junior


Prezados Senhores:

        Com
as atuais divulgações das fotos do Hubble, qual o valor de se ter
agora um telescópio, por melhor que ele seja?
        Espero,
justamente, que haja tal valor, daí a seriedade da inquirição. Pretendo
adquirir um, a depender das respostas.

ATT.
João Carlos.


Re: [obm-l] Reportagem na TVE

2005-08-04 Por tôpico JoaoCarlos_Junior




Essa reportagem passará novamente? Ou outra assemelhada? Se sim, em que dia
será?
ATT. João.


=
Instruções para entrar na lista, sair da lista e usar a lista em
http://www.mat.puc-rio.br/~nicolau/olimp/obm-l.html
=


[obm-l] OFF TOPIC Resultado da China

2005-07-20 Por tôpico JoaoCarlos_Junior

Por que a China obteve resultado com
tantos ouros?
Que política educativa vinculada à matemática
existe por lá?

Re: [obm-l] Resultado da Equipe da IMO

2005-07-20 Por tôpico JoaoCarlos_Junior


Brasileiros que nos levantam ante o
mundo, adquiriremos respeito e consideração em função de vossas atitudes.
Parabéns aos alunos que com galhardia
erguem-nos!
Parabéns aos professores que os ensinam
e elevam o nível o intelectual da nação brasileira!








Olimpiada Brasileira de Matematica
<[EMAIL PROTECTED]>
Enviado Por: [EMAIL PROTECTED]
19/07/2005 13:56
Favor responder a obm-l
        
        Para:
       obm-l@mat.puc-rio.br
        cc:
       
        Assunto:
       [obm-l] Resultado da Equipe da IMO


O Resultado da equipe na IMO
foi a seguinte:

Gabriel Tavares Bujokas          Medalha de Ouro
  37 pontos
Edson Augusto Bezerra Lopes Sem premiação        6
pontos
Leandro Farias Maia                M.
Honrosa            9 pontos
Levi Máximo Viana                
 Sem premiação       6 pontos
Thomás Yoiti Sasaki Hoshina   Medalha de Bronze 14 pontos
Guilherme Nogueira de Souza  M. Honrosa        
   10 pontos

Abraços, Nelly.
=
Instruções para entrar na lista, sair da lista e usar a lista em
http://www.mat.puc-rio.br/~nicolau/olimp/obm-l.html
=



Re: [obm-l] geometria

2005-07-06 Por tôpico JoaoCarlos_Junior

Isso, tens razão. 
Não sei se digitei errado ou a causa
do equívoco é outra. 
No papel que utilizei realmente achei
7 e 1.
Vc está certo, obrigado.







Carlos <[EMAIL PROTECTED]>
Enviado Por: [EMAIL PROTECTED]
06/07/2005 12:06
Favor responder a obm-l
        
        Para:
       obm-l@mat.puc-rio.br
        cc:
       
        Assunto:
       Re: [obm-l] geometria


Oi, acho que R=7 e r=1.
Abraços,
Carlos
[EMAIL PROTECTED] wrote:

>
> Prezados senhores:
>
>         Corrijam-me se eu estiver errado.
>         Brunno, não vou escrever a solução com
precisão. Observe dois 
> triângulos retângulos com vértices nos centros das circunferências
e 
> nos quais um dos catetos (em cada triângulo) é paralelo as tangentes

> internas e externas.
>         Desses triângulos, você perceberá que
R+r=8 e R-r=6. Portanto, 
> R=8 e r=1.
>
>
>
>
>                  *"Brunno
Fernandes" <[EMAIL PROTECTED]>*
> Enviado Por: [EMAIL PROTECTED]
>
> 05/07/2005 21:04
> Favor responder a obm-l
>
>                  
      
>         Para:        
>         cc:        
>         Assunto:        [obm-l]
geometria
>
>
>
>
> Ola pessoal, poderiam me ajudar com essa questao de geometria
>
> As tangentes comuns externas e internas a duas circunferencias medem
8 
> cm e
> 6 cm, respectivamente. Achar os raios das duas circunferencias sabendo

> que a
> distancia entre os seus centros tem 10 cm
>
> Um abraco
>
> =
> Instruções para entrar na lista, sair da lista e usar a lista em
> http://www.mat.puc-rio.br/~nicolau/olimp/obm-l.html
> =
>



                

                

                
                 
___ 
Yahoo! Acesso Grátis - Internet rápida e grátis. 
Instale o discador agora! http://br.acesso.yahoo.com/

=
Instruções para entrar na lista, sair da lista e usar a lista em
http://www.mat.puc-rio.br/~nicolau/olimp/obm-l.html
=



Re: [obm-l] geometria

2005-07-06 Por tôpico JoaoCarlos_Junior


Prezados senhores:

        Corrijam-me
se eu estiver errado.
        Brunno,
não vou escrever a solução com precisão. Observe dois triângulos retângulos
com vértices nos centros das circunferências e nos quais um dos catetos
(em cada triângulo) é paralelo as tangentes internas e externas.
        Desses
triângulos, você perceberá que R+r=8 e R-r=6. Portanto, R=8 e r=1.







"Brunno Fernandes" <[EMAIL PROTECTED]>
Enviado Por: [EMAIL PROTECTED]
05/07/2005 21:04
Favor responder a obm-l
        
        Para:
       
        cc:
       
        Assunto:
       [obm-l] geometria


Ola pessoal, poderiam me ajudar com essa questao de
geometria

As tangentes comuns externas e internas a duas circunferencias medem 8
cm e
6 cm, respectivamente. Achar os raios das duas circunferencias sabendo
que a
distancia entre os seus centros tem 10 cm

Um abraco

=
Instruções para entrar na lista, sair da lista e usar a lista em
http://www.mat.puc-rio.br/~nicolau/olimp/obm-l.html
=



Res:Re: [obm-l] Problema 1 da XXV OBM - Nível 1, fase 3

2005-04-29 Por tôpico JoaoCarlos_Junior
Wilner, você tem razão. Li o problema errado, e compliquei a solução. X, y e z podem ser os algarismos do próprio número menor que 1000, o que dá uma solução de uma linha. Não há a menor necessidade de se falar em resto, a não ser que se leia errado o problema como eu: li divisível por 7, ao invés de igual a 7. O fato é que ambos os problemas tem a mesma resposta 36.[EMAIL PROTECTED] escreveu: -Para: obm-l@mat.puc-rio.brDe: Eduardo Wilner <[EMAIL PROTECTED]>Enviado por: [EMAIL PROTECTED]Data: 29/04/2005 12:05Assunto: Re: [obm-l] Problema 1 da XXV OBM - Nível 1, fase 3  Prezado João Carlos  Poderia explicar melhor tua solução?  Parece que vc. chega a C(7,9)! De onde?  Porque os algarismos resultam como restos da divisãopor 7?  Eu encontrei 42 ...!  Abraço  Wilner --- [EMAIL PROTECTED] wrote:> Como sempre gentil, obrigado: amigo Buffara.> > > > > Claudio Buffara <[EMAIL PROTECTED]>> Enviado Por: [EMAIL PROTECTED]> 28/04/2005 21:22> Favor responder a obm-l>  >         Para:   >         cc: >         Assunto:        Re: [obm-l] Problema 1 da> XXV OBM - Nível 1, fase > 3> > > Sim.> > on 28.04.05 19:46, [EMAIL PROTECTED]> at > [EMAIL PROTECTED] wrote:> > Quantos inteiros positivos menores que 1.000 têm> soma de seus algarismos > igual a 7?> Pergunta: essa solução que segue abaixo faz sentido?> Solução: esse problema é equivalente a encontrar o> número de soluções > inteiras para a equação: x+y+z=7, na qual x, y e z> são os restos da > divisão da centena, dezena e unidade do inteiro> (menor que 1000) por 7, ou > seja, 9!/(7!2!)=36.> 				Yahoo! Acesso Grátis - Internet rápida e grátis. Instale o discador agora! http://br.acesso.yahoo.com/=Instruções para entrar na lista, sair da lista e usar a lista emhttp://www.mat.puc-rio.br/~nicolau/olimp/obm-l.html=Instruções para entrar na lista, sair da lista e usar a lista em
http://www.mat.puc-rio.br/~nicolau/olimp/obm-l.html


Re: [obm-l] Problema 1 da XXV OBM - Nível 1, fase 3

2005-04-29 Por tôpico JoaoCarlos_Junior

Como sempre gentil, obrigado: amigo
Buffara.






Claudio Buffara <[EMAIL PROTECTED]>
Enviado Por: [EMAIL PROTECTED]
28/04/2005 21:22
Favor responder a obm-l
        
        Para:
       
        cc:
       
        Assunto:
       Re: [obm-l] Problema 1 da XXV OBM -
Nível 1,        fase 3


Sim.

on 28.04.05 19:46, [EMAIL PROTECTED] at [EMAIL PROTECTED]
wrote:

Quantos inteiros positivos menores
que 1.000 têm soma de seus algarismos igual a 7?
Pergunta: essa solução que segue abaixo faz sentido?
Solução: esse problema é equivalente a encontrar o número de soluções inteiras
para a equação: x+y+z=7, na qual x, y e z são os restos da divisão da centena,
dezena e unidade do inteiro (menor que 1000) por 7, ou seja, 9!/(7!2!)=36.


[obm-l] Problema 1 da XXV OBM - Nível 1, fase 3

2005-04-28 Por tôpico JoaoCarlos_Junior
Quantos inteiros positivos menores que 1.000 têm soma de seus algarismos igual a 7?Pergunta: essa solução que segue abaixo faz sentido?Solução: esse problema é equivalente a encontrar o número de soluções inteiras para a equação: x+y+z=7, na qual x, y e z são os restos da divisão da centena, dezena e unidade do inteiro (menor que 1000) por 7, ou seja, 9!/(7!2!)=36.Instruções para entrar na lista, sair da lista e usar a lista em
http://www.mat.puc-rio.br/~nicolau/olimp/obm-l.html


[obm-l] creio OFF TOPIC: Eureka 21 no site da SBM

2005-03-23 Por tôpico JoaoCarlos_Junior


Por obsequio, respondam-me os que sabem:
qual a data prevista para inclusao da revista eureka 21 no site da SBM?

ATT. Joao Carlos

Re: [obm-l] Dado interessante sobre a IMO 2004

2004-07-20 Por tôpico JoaoCarlos_Junior




E o Ralph e o professor Gugu? Quais as notas de cada, quando aumentaram a
coleção de prêmios olímpicos do país com três medalhas douradas?
E  ainda,  no  ano  deles, quantos atingiram os 42 pontos? E De que
  países eram?
Essas duas últimas perguntas valem também para o professor Nicolau.

  Um abraço, João.



   

  Olimpiada

  Brasileira dePara: [EMAIL PROTECTED] 
 
  Matematica   cc: 

  <[EMAIL PROTECTED]>Assunto:  Re: [obm-l] Dado 
interessante sobre a IMO 2004
  Enviado Por: 

  [EMAIL PROTECTED]

  uc-rio.br

   

   

  20/07/2004 13:34 

  Favor responder a

  obm-l

   

   





At 01:09 PM 7/20/04 -0300, you wrote:
>Olá pessoal, gostaria de parabenizar a equipe pela conquista e tirar umas
>dúvidas que eu tenho...

>1-) Em alguma IMO o Brasil já fez 42 pontos?
Sim, foi o Prof. Nicolau Saldanha. :) :)

>2-) Existe IMO no nível universitário?
Sim, a IMC. (nossa equipe vai na quinta-feira)

>3-) Ouvi dizer que temos um rapaz de 19 anos que terminou o doutorado no
>IMPA.
Verdade.

>Gostaria de saber se ele já participou de alguma IMO?
Não.

>Se sim, como foi o desempenho, se não, por quê?
Ele não é olímpico.

>Grato!
>Um abraço
>Alan

Abração, Nelita.


=
Instruções para entrar na lista, sair da lista e usar a lista em
http://www.mat.puc-rio.br/~nicolau/olimp/obm-l.html
=




=
Instruções para entrar na lista, sair da lista e usar a lista em
http://www.mat.puc-rio.br/~nicolau/olimp/obm-l.html
=


[obm-l] Cone Sul Q 2

2004-05-28 Por tôpico JoaoCarlos_Junior
Professor Márcio Cohen, outros professores, alunos, amigos,

  A resolução que segue é satisfatória?

  Desenhe-se  a  figura integralmente. A mesma é simétrica em relação à
  reta  PO, digo: com a movimentação de Q. Logo, é razoável pensar que esse
  ponto  fixo  é  a  intersecção de MN com PO, seja R tal ponto. Sim, O é o
  centro do círculo dado.
  Também por simetria, é razoável pensar que R é médio de MN. A questão
  então se resume a amarrar R às partes fixas (hipóteses do problema).
  Ora,  pontos  médios  de  segmentos (não de arcos) lembram, em regra,
  paralelogramos.   Se   provarmos  então  que  PMTN  é  paralelogramo,  (T
  intersecção de AB com PO), está resolvido o problema.
  Para  demonstrar  que  PMTN  é paralelogramo, muitas maneiras há, com
  igualdade de segmentos, de ângulos, o que parece mais fácil é esse último
  caso: igualdade de ângulos. Assim, tentemos demonstrar que NPT= PTM e que
  TPM = NTP (ângulos).
  PAM  =  PTM  (PMTA  é inscritível) e, tais ângulos são iguais ao arco
  menor QA/2 (PA é tangente ao círculo dado). Mas, XBT (X intersecção de BN
  com  PT)  tem essa mesma medida e é igual a XPN, pois os triângulos NPX e
  XTB  são semelhantes, o que se vê facilmente. Enfim, NPT = PTM (ângulos).
  Analogamente,  prova-se  que TPM = PTN. Logo, PMTN é paralelogramo, o que
  demonstra as suspeitas oriundas da simetria. (FIM).

  Na  realidade,  acredito  que  o  foco de minha dúvida restringe-se a
  saber  se  a  simetria,  conforme mencionada acima, efetivamente prova ou
  apenas levanta suspeita. E se assim, pode ser utilizada.

  ATT. João.




=
Instruções para entrar na lista, sair da lista e usar a lista em
http://www.mat.puc-rio.br/~nicolau/olimp/obm-l.html
=


Re: [obm-l] Equipe IMO-2004

2004-05-28 Por tôpico JoaoCarlos_Junior

  Bons preparativos a todos os representantes brasileiros. Boa sorte na
prova, quando chegar à época!
  Em especial, ao Fabio Dias Moreira, que muito contribui nesta lista.

  Em  especial,  ao  Gabriel  Bujokas, pelo belo resultado alcançado na

  Olimpíada da Cone Sul deste ano.

  Aos  outros,t  que  possam  nos  surpreender  e  a  si  próprios  com

  resultados mui satisfatórios.



  Um abraço, João.




   

  Olimpiada

  Brasileira dePara: [EMAIL PROTECTED] 
 
  Matematica   cc: 

  <[EMAIL PROTECTED]>Assunto:  [obm-l] Equipe IMO-2004 
  
  Enviado Por: 

  [EMAIL PROTECTED]

  uc-rio.br

   

   

  28/05/2004 09:37 

  Favor responder a

  obm-l

   

   





Caros(as) amigos(as) da lista:

Definida a equipe da IMO-2004.

Líder: Carlos Gustavo T. de A. Moreira (Rio de Janeiro - RJ)
Vice-Líder: Carlos Yuzo Shine (São Paulo - SP)

Equipe:
BRA1: Fábio Dias Moreira (Rio de Janeiro - RJ)
BRA2: Gabriel Tavares Bujokas (São Paulo - SP)
BRA3: Henry Wei Cheng Hsu (São Paulo - SP)
BRA4: Rafael Daigo Hirama (Campinas - SP)
BRA5: Rafael Marini Silva (Rio de Janeiro - RJ) ex (Vila Velha - ES)
BRA6: Thiago Costa Leite Santos (São Paulo - SP)

Abraços, Nelly.










=
Instruções para entrar na lista, sair da lista e usar a lista em
http://www.mat.puc-rio.br/~nicolau/olimp/obm-l.html
=


RE: [obm-l] Cone Sul - Problema 2

2004-05-27 Por tôpico JoaoCarlos_Junior

Professor Márcio Cohen, outros professores, alunos, amigos,

  A resolução que segue é satisfatória?

  Desenhe-se  a  figura integralmente. A mesma é simétrica em relação à
  reta  PO, digo: com a movimentação de Q. Logo, é razoável pensar que esse
  ponto  fixo  é  a  intersecção de MN com PO, seja R tal ponto. Sim, O é o
  centro do círculo dado.
  Também por simetria, é razoável pensar que R é médio de MN. A questão
  então se resume a amarrar R às partes fixas (hipóteses do problema).
  Ora,  pontos  médios  de  segmentos (não de arcos) lembram, em regra,
  paralelogramos.   Se   provarmos  então  que  PMTN  é  paralelogramo,  (T
  intersecção de AB com PO), está resolvido o problema.
  Para  demonstrar  que  PMTN  é paralelogramo, muitas maneiras há, com
  igualdade de segmentos, de ângulos, o que parece mais fácil é esse último
  caso: igualdade de ângulos. Assim, tentemos demonstrar que NPT= PTM e que
  TPM = NTP (ângulos).
  PAM  =  PTM  (PMTA  é inscritível) e, tais ângulos são iguais ao arco
  menor QA/2 (PA é tangente ao círculo dado). Mas, XBT (X intersecção de BN
  com  PT)  tem essa mesma medida e é igual a XPN, pois os triângulos NPX e
  XTB  são semelhantes, o que se vê facilmente. Enfim, NPT = PTM (ângulos).
  Analogamente,  prova-se  que TPM = PTN. Logo, PMTN é paralelogramo, o que
  demonstra as suspeitas oriundas da simetria. (FIM).

  Na  realidade,  acredito  que  o  foco de minha dúvida restringe-se a
  saber  se  a  simetria,  conforme mencionada acima, efetivamente prova ou
  apenas levanta suspeita. E se assim, pode ser utilizada.

  ATT. João.



   

  André Araújo 

  <[EMAIL PROTECTED]Para: [EMAIL PROTECTED]
  
  l.com>   cc: 

  Enviado Por: Assunto:  RE: [obm-l] Cone Sul - 
Problema 2 
  [EMAIL PROTECTED]

  uc-rio.br

   

   

  25/05/2004 13:17 

  Favor responder a

  obm-l

   

   





Abaixo uma outra solucao p/ o problema 2 da Cone Sul.

segunda solucao:

Seja S a intersecao de AB com a reta PO, onde O eh o centro de C. Eh facil
ver q AB eh perpendicular a PS. Dai conclui-se:

i) quadrilatero PMSA eh inscritivel (ang PSA = ang PMA = 90);
ii) quadrilatero PNSB eh inscritivel (ang PSB = ang PNB = 90);

Como PA e PB sao tangentes a C, tem-se:

iii) ang ABQ = ang PAM = arco menor AQ/2;
iv) ang PBQ = ang BAQ = arco menor BQ/2;

De i), ii), iii) e iv) tem-se:

v) ang MSP = ang PAM = ang ABQ = ang NPS;
vi) ang NSP = ang PBQ = ang BAQ = ang MPS;

De v) e vi) conclui-se que os triangulos PMS e PNS sao congruentes, caso
A.L.A. Ou seja, PMSN eh paralelogramo. Logo a reta MN corta o ponto medio
PS
(fixo).

[ ]'s

AA.



>Cone Sul - Problema 2
>
>"Dada uma circunferencia C e um ponto P exterior a ela, tracam-se por P as
>duas tangentes aa circunferencia, sendo A e B os pontos de tangencia.
>Toma-se um ponto Q sobre o menor arco AB de C. Seja M a intersecao da reta
>AQ com a perpendicular a AQ tracada por P e seja N a intersecao da reta BQ
>com a perpendicular a BQ tracada por P. Demonstre que, ao variar Q no arco
>AB, todas as retas MN passam por um mesmo ponto.
>
>
>Solucao:
>
>Sejam:
>H o pe da perpendicular de P a AB
>R e S as projecoes de N e M, respectivamente, a PH
>Q e T as projecoes de N e M, respectivamente, a AB
>
>No triangulo PNM:
>PN = PB.sen(
>QH = NR = PN.sen( (por
I)
>
>QH = PB.sen(
>Da mesma forma encontramos:
>
>TH = PA.sen(
>Como PA = PB, 
>Logo, a intersecao de MN com a altura PH se da no ponto medio de MN, que
>chamamos de L, e LH eh base media do trapezio QNMT com bases NQ e MT.
>Entao LH = (NQ + MT)/2
>
>Mas NQ = PH - PR = PH - PN.cos(
>Da mesma forma:
>
>MT = PH - PA.sen(
>e
>
>NQ + MT = 2PH - PA.(sen(
>= 2PH - PA.sen(
>e LH = (NQ + MT)/2 = PH/2
>
>
>Ou seja, todas as retas MN p

Re: [obm-l] Geometria Plana

2004-05-14 Por tôpico JoaoCarlos_Junior

Bem, antes de tudo, obrigado Buffara (sempre solícito) e Boromir pelo
esclarecimento.

Boromir,

Com relação à segunda questão, esclareço-te:
1)  O  enunciado  deve  ser  interpretado para que seja resolvida a
  questão a ele referente. Em específico, a parte que cabe interpretação é:
  Ora, se os círculos interiores são tangentes ao círculo dado C, e cada um
  deles  contém respectivamente as cordas AD e BD, e A e B também pertencem
  ao círculo dado C, então, conclui-se que...
As respostas às tuas perguntas:
1)  A corda móvel não tem comprimento fixo. Ela passa sempre por um
  ponto fixo D, tal que AD é diferente de DB.
2) D está sobre a corda AB e não sobre o arco AB.

  ATT. João




   

  "boromir"

  <[EMAIL PROTECTED]Para: <[EMAIL PROTECTED]>  
  
  m.br>cc: 

  Enviado Por: Assunto:  Re: [obm-l] Geometria Plana   

  [EMAIL PROTECTED]

  uc-rio.br

   

   

  13/05/2004 01:49 

  Favor responder a

  obm-l

   

   









=
>De:[EMAIL PROTECTED]
>Para:[EMAIL PROTECTED]
>Assunto:[obm-l] Geometria Plana
>
>Caros participantes,
>
> Três exercícios de Geometria Plana
>seguem abaixo.
>
> Minha dúvida é somente com relação à
>letra (b) do primeiro, a qual:
>  Essa  figura  (lugar geométrico) é tal chamada
>cardióide? Sua equação é Y
>  =  (sent  +   cost),com  ?90vértice de tal ângulo t é o
>  simétrico  de  B em relação a O, centro do
>círculo? O próprio ponto B não
>  pertence a esse LG, correto?
>
> Os   demais   exercícios  foram
>lançados,  pois  que, pode  haver
>  interessados nos mesmos.
>
>  1) Dado um círculo de centro O, seja BC uma
>corda fixa desse círculo, tal
>  que BC  =  90  graus.  Para cada A
>pertencente ao círculo, constrói-se o
>  quadrado ABMN, exterior ao triângulo ABC.
>  a)  Mostre que a reta AN passa por um ponto
>fixo.
AN é perpendicular a AB e portanto a reta AN intersecta o círculo no ponto
B´ diametralmente oposto a B. Como B é fixo B' também é fixo.


>  b)  Determine o lugar geométrico de N.
Tome o ponto C' diametralmente oposto a C. Os triângulos C'AB e C'AN são
congruentes (caso LAL no ângulo A) e, portanto C'B = C'N. Como C'B é fixo
igual ao raio do círculo original vezes metade de sqrt(2), o LG de N é um
círculo de centro C' e raio igual ao raio docírculo dado vezes sqrt(2).



>
>  2)  AB  é uma corda móvel de um círculo dado
>C, e D é um ponto fixo sobre
>  AB  (DA diferente de DB). Dois círculos
>variáveis tangentes interiormente
>  ao  círculo C e contendo respectivamente as
>cordas AD e DB se interceptam
>  em Q. Determinar o lugar geométrico do ponto
>Q.
>
Eu não entendi muito bem o enunciado...
1) A corda móvel tem comprimento fixo?
2) D está sobre a corda AB ou sobre o arco AB?

>  3)  Dois  círculos  de  centro  O e O´e de
>raios R e R´se interceptam nos
>  pontos  A  e  B.  Uma  reta r contendo A
>intercepta o círculo O em C e o
>  círculo O´em D.
>  a)  Provar que a soma dos arcos CA + AD
>permanece constante quando a reta
>  r varia;
O ângulo OAO' nunca se altera. OAO'+ OAC + O'AD = 180 graus. OAC é metade
do suplemento do arco AC e O'AD é metade do suplemento do arco AD,portanto:
OAO' + 90 - 0,5*arco(AC) + 90 - 0,5*Arco AD = 180
0,5*[arco(AC) + arco(AD)] = OAO'
Arco (AC) + Arco(AD)= 2*OAO' que é constante.

>  b)  Considerar uma segunda posição da reta r
>que determine no círculo de
>  centro O um ponto E, e no círculo de centro
>O´um ponto F, tais que P seja
>  a interseção de EC e DF. Prove que os ângulos
>CPD e CBD são
>  suplementares.


São suplementares e alem disso são constantes.
NOte que o ângulo PEF mede metade do arco AC; O ângulo AFD

[obm-l] Geometria Plana

2004-05-12 Por tôpico JoaoCarlos_Junior
Caros participantes,

Três exercícios de Geometria Plana seguem abaixo.

Minha dúvida é somente com relação à letra (b) do primeiro, a qual:
  Essa  figura  (lugar geométrico) é tal chamada cardióide? Sua equação é Y
  =  (sent  +  cost),  com  ?90http://www.mat.puc-rio.br/~nicolau/olimp/obm-l.html
=


[obm-l] Logaritmos

2004-04-02 Por tôpico JoaoCarlos_Junior
Prove que, para todo inteiro n maior que 1 e para todo x diferente de zero,
com x maior que ?1, tem-se:

 (1+x)^n > (1+nx)

Fonte: Logaritmos ? Elon Lages Lima, p. 11.



=
Instruções para entrar na lista, sair da lista e usar a lista em
http://www.mat.puc-rio.br/~nicolau/olimp/obm-l.html
=


[obm-l] Re: Geometria Plana

2004-02-26 Por tôpico JoaoCarlos_Junior
Alguém colocara na lista o exercício que abaixo segue, porém, cometi o
equívoco de apagá-lo:

  Dado  um  triângulo  ABC,  as tangentes ao círculo circunscrito a tal
  triângulo,  pelos  vértices  dos  mesmos, interceptam os lados opostos em
  três pontos distintos. Provar que tais pontos são colineares.

  A  solução que segue é simples, não no sentido de bela, mas devido ao
  uso  de  parcos  conhecimentos  para  inferi-la. Dirigindo-me ao original
  interessado na resolução, digo:

  Desenhe  a  figura  ou parte dela. Sejam M, N e P esses pontos: M é a
  intersecção de AC com a tangente ao círculo por B; N é a interseção de BC
  com a tangente por A, e P é o outro, construído de forma semelhante. Seja
  X  a  interseção  das  tangentes por A e B. Ainda com linguagem, sejam os
  ângulos: NMB = teta, MNA = alfa, BNP = beta e BPN = delta. Assim:

Triângulo MNX, teta + alfa = 2C;
Triângulo PNB, beta + delta = 180 ? B e
Triângulo MPB, teta + delta = C.

  Logo,  alfa + beta = 180 + C ? B. Como ANB = B ? C, tem-se que alfa +
  beta + ANB = 180. Logo, M, N e P são colineares.

  Essa  questão  suscitou-me  outras.  Assim,  inquiro ao professores e
  interessados: quais são todas as formas, em geometria plana, de se provar
  colinearidade de três pontos? Lembro-me de Simpson. Quais as outras?

  ATT. João Carlos.





=
Instruções para entrar na lista, sair da lista e usar a lista em
http://www.mat.puc-rio.br/~nicolau/olimp/obm-l.html
=


Re: [obm-l] Re: [obm-l] Números Pitagóricos

2004-02-13 Por tôpico JoaoCarlos_Junior

Este libro é vendido pela própria SBM. O site é http://www.sbm.org.br.








"Daniel Melo Wanzeller" <[EMAIL PROTECTED]>
Enviado Por: [EMAIL PROTECTED]
13/02/2004 11:48
Favor responder a obm-l

        
        Para:        <[EMAIL PROTECTED]>
        cc:        
        Assunto:        [obm-l] Re: [obm-l] Números Pitagóricos


Voce sabe onde encontrar este livro???
- Original Message -
From: <[EMAIL PROTECTED]>
To: <[EMAIL PROTECTED]>
Sent: Friday, December 19, 2003 4:22 PM
Subject: [obm-l] Números Pitagóricos


> No livro: Episódios da História Antiga da Matemática, de Asger Aaboe,
> traduzido por João Pitomberia de Carvalho, SBM, há em sua pág.32 o
seguinte
> teorema:
>         Se  p  e q tomam todos os valores inteiros, restritos somente
pelas
>
>   seguintes condições:
>
>   1)  p > q > 0;
>   2)  p e q não possuem divisor comum (distinto de 1) e
>   3)  p e q não são ambos ímpares.
>
>
>         Então  as  expressões:  x=p^2 ? q^2; y=2pq e z=p^2 + q^2
fornecerão
>
>   todos os ternos pitagóricos reduzidos, e cada terno somente uma vez.
>
>         Pergunto: Como demonstrar tal teorema?
>
>         Nas  notas  de  rodapé,  há afirmação que uma demonstração para
tal
>
>   teorema  está  em  H.Rademacher e O.Toeplitz, secção 14, p.88, porém,
não
>
>   tenho tal livro.
>
>         Assim, solicito, por obséquio, uma demonstração.
>
>   ATT. João Carlos
>
>
>
>
> =
> Instruções para entrar na lista, sair da lista e usar a lista em
> http://www.mat.puc-rio.br/~nicolau/olimp/obm-l.html
> =

=
Instruções para entrar na lista, sair da lista e usar a lista em
http://www.mat.puc-rio.br/~nicolau/olimp/obm-l.html
=




[obm-l] Números Pitagóricos

2003-12-19 Por tôpico JoaoCarlos_Junior
No livro: Episódios da História Antiga da Matemática, de Asger Aaboe,
traduzido por João Pitomberia de Carvalho, SBM, há em sua pág.32 o seguinte
teorema:
Se  p  e q tomam todos os valores inteiros, restritos somente pelas

  seguintes condições:

  1)  p > q > 0;
  2)  p e q não possuem divisor comum (distinto de 1) e
  3)  p e q não são ambos ímpares.


Então  as  expressões:  x=p^2 ? q^2; y=2pq e z=p^2 + q^2 fornecerão

  todos os ternos pitagóricos reduzidos, e cada terno somente uma vez.

Pergunto: Como demonstrar tal teorema?

Nas  notas  de  rodapé,  há afirmação que uma demonstração para tal

  teorema  está  em  H.Rademacher e O.Toeplitz, secção 14, p.88, porém, não

  tenho tal livro.

Assim, solicito, por obséquio, uma demonstração.

  ATT. João Carlos




=
Instruções para entrar na lista, sair da lista e usar a lista em
http://www.mat.puc-rio.br/~nicolau/olimp/obm-l.html
=


Re: [obm-l] Livraria no Brasil para compra de livros de matem ática

2003-11-13 Por tôpico JoaoCarlos_Junior

Cláudio,

  Testei  alguns  títulos  no www.amazom.com e encontrei-os. Muitíssimo
obrigado!
  Com  relação  ao  livros da MIR, não sei se eles se extinguiram junto
com  a  União  Soviética,  (risos).  Porém,  no  mesmo  livro  que eu havia
mencionado, o de Olimpíadas de 1ª a 8ª, há o que segue:

  As  traduções  para  o espanhol ou para o inglês dos livros russos da
Editora  Mir  podem  ser  comprados  por  reembolso  postal.  Endereço para
informações e pedidos:
  Casa da Cultura da Livraria Tecno-Científica Ltda.
  R. Francisca Miquelina, 66
  01316 ? São Paulo, SP
  Telefone: (011) 34-1529.

  Não  sei  se  em  tal  casa,  há  ainda os livros da MIR. O número do
telefone parece-me desatualizado, não sei as outras informações.
  Digitei no "google": Casa da Cultura da Livraria Tecno-Científica, só
Casa  da  Cultura, só Livraria Tecno-Científica, porém, as respostas não me
satisfizeram. Boa Sorte e obrigado!

Um abraço, João.






   
 
  Claudio Buffara  
 
  <[EMAIL PROTECTED]Para: <[EMAIL PROTECTED]>  
  
  .com.br>  cc:
 
  Enviado Por:  Assunto:  Re: [obm-l] Livraria no 
Brasil para compra de 
  [EMAIL PROTECTED] livros de matem  ática 

  .puc-rio.br  
 
   
 
   
 
  13/11/2003 09:04 
 
  Favor responder a
 
  obm-l
 
   
 
   
 




on 13.11.03 11:00, [EMAIL PROTECTED] at
[EMAIL PROTECTED] wrote:

> Na última página do Livro: Olimpíadas de Matemática 1ª a 8ª, há indicação
> de duas livrarias para compra dos livros indicados na bibliografia.
Porém,
> nenhuma dela, agora, vende tais livros. Alguém pode indicar, por
gentileza,
> outra livraria no Brasil, onde tais livros possam ser adquiridos.
>
>
> ATT. João Carlos
>
>
Bom, esse livro eh de 1988 - era pre-internet...

Eu procuraria nos sites:

http://www.livrariacultura.com.br/

http://www.amazon.com/

http://www.barnesandnoble.com/

https://enterprise.maa.org/ecomtpro/timssnet/common/tnt_frontpage.cfm

http://www.ams.org/bookstore

Infelizmente, eu nunca consegui encontrar uma referencia pros livros da
editora Mir. Acho que ela acabou junto com a Uniao Sovietica.

Um abraco,
Claudio.

=
Instruções para entrar na lista, sair da lista e usar a lista em
http://www.mat.puc-rio.br/~nicolau/olimp/obm-l.html
=





=
Instruções para entrar na lista, sair da lista e usar a lista em
http://www.mat.puc-rio.br/~nicolau/olimp/obm-l.html
=


[obm-l] Livraria no Brasil para compra de livros de matemática

2003-11-13 Por tôpico JoaoCarlos_Junior
Na última página do Livro: Olimpíadas de Matemática 1ª a 8ª, há indicação
de duas livrarias para compra dos livros indicados na bibliografia. Porém,
nenhuma dela, agora, vende tais livros. Alguém pode indicar, por gentileza,
outra livraria no Brasil, onde tais livros possam ser adquiridos.


  ATT. João Carlos



=
Instruções para entrar na lista, sair da lista e usar a lista em
http://www.mat.puc-rio.br/~nicolau/olimp/obm-l.html
=


Re:[obm-l] Olímpiada 2003, nível 3 - Primeira fase

2003-11-07 Por tôpico JoaoCarlos_Junior

  Acredito que sua solução é melhor que a minha, Buffara! Pois, é mais
curta e com mais simetrias. Não é fácil enxergar a todas essas, assim como
fizestes.

  Obrigado, João Carlos.



   
 
  "claudio.buffara"
 
  <[EMAIL PROTECTED]Para: "obm-l" <[EMAIL PROTECTED]>  
  
  .com.br>  cc:
 
  Enviado Por:  Assunto:  Re:[obm-l] Olímpiada 
2003, nível 3 - Primeira 
  [EMAIL PROTECTED] fase   

  .puc-rio.br  
 
   
 
   
 
  07/11/2003 11:51 
 
  Favor responder a
 
  obm-l
 
   
 
   
 




Nao sei se voce vai achar mais simples, mas a minha solucao seria a
seguinte:

O numero total de triminos distintos eh:
Tipo ABC: Binom(7,3) = 35
Tipo AAB: 7*6 = 42
Tipo AAA: 7

Total = 35+42+7 = 84

Como cada trimino tem 3 numeros, temos um total de 3*84 = 252 numeros
escritos em todos eles.

Por simetria, cada numero aparece o mesmo numero de vezes. Logo, cada
numero aparece 252/7 = 36 vezes.

Logo, a soma dos numeros serah 36*(0+1+2+3+4+5+6) = 36*21 = 756.

Um abraco,
Claudio.

 
 De:  [EMAIL PROTECTED]  
 

 
 Para:[EMAIL PROTECTED]   
 


 
 Cópia:  
 


 
 Data:Fri, 7 Nov 2003 10:02:10 -0400 
 


 
 Assunto: [obm-l] Olímpiada 2003, nível 3 - Primeira fase
 


 
 
 


> Na EUREKA 16, pág.13, há o exercício que segue da primeira fase da
> Olimpíada, nível 03, deste ano:
>
>
> No triminó marciano, as peças têm 3 números cada (diferente do
>
> dominó da Terra, onde cada peça tem apenas 2 números). Os números no
>
> triminó marciano também variam de 0 a 6, e para cada escolha de 3 números
>
> (não necessariamente distintos) existe uma e somente uma peça que contém
>
> esses 3 números. Qual é a soma dos números de todas as peças do triminó
>
> marciano?
>
>
>
> Apesar de resolvê-lo sem uso de lápis ou papel, conforme reprodução
>
> a seguir, pergunto: há uma forma mais simples?
>
>
>
> RESPOSTA
>
>
>
> Pense-se no triminó sem a parte central. Assim, para cada valor da
>
> parte superior do, agora, dominó, a parte inferior assume os valores de 0
>
> a 6. Logo, para cada um daqueles valores, se tem a soma 21. Ou seja, a
>
> soma parcial do dominó é 7x21 + 21 = 8x21, sendo este último 21 da soma
>
> dos valores da parte superior.
>
> Se se vai incluir no dominó a parte central para formar o triminó,
>
> parte da soma acima estará repetida e parte não, devido à simetria, no
>
> triminó, por eixo mediano horizontal ao mesmo. Ou seja, a soma encontrada
>
> é um número da forma 2X+Y.
>
> Y é (1+1)+(2+2)+(3+3)+(4+4)+(5+5)+(6+6). Ora, isto é, logicamente,
>
> igual a 21x2. Assim, 2X=6x21. Então, X=3x21. Logo, para cada valor
>
> central, teremos a soma X+Y=5x21, ou seja, 7x5x21 + 21, este último 21 da
>
> própria soma da parte central. Portanto, a resposta é 36x21 = 720+36 =
>
> 756.
>
>
>
> ===

[obm-l] Olímpiada 2003, nível 3 - Primeira fase

2003-11-07 Por tôpico JoaoCarlos_Junior
Na EUREKA 16, pág.13, há o exercício que segue da primeira fase da
Olimpíada, nível 03, deste ano:


No  triminó  marciano,  as  peças  têm 3 números cada (diferente do

  dominó  da  Terra,  onde  cada  peça tem apenas 2 números). Os números no

  triminó marciano também variam de 0 a 6, e para cada escolha de 3 números

  (não  necessariamente distintos) existe uma e somente uma peça que contém

  esses  3  números. Qual é a soma dos números de todas as peças do triminó

  marciano?



Apesar de resolvê-lo sem uso de lápis ou papel, conforme reprodução

  a seguir, pergunto: há uma forma mais simples?



  RESPOSTA



Pense-se  no triminó sem a parte central. Assim, para cada valor da

  parte superior do, agora, dominó, a parte inferior assume os valores de 0

  a  6.  Logo,  para cada um daqueles valores, se tem a soma 21. Ou seja, a

  soma  parcial  do dominó é 7x21 + 21 = 8x21, sendo este último 21 da soma

  dos valores da parte superior.

Se  se vai incluir no dominó a parte central para formar o triminó,

  parte  da  soma  acima estará repetida e parte não, devido à simetria, no

  triminó, por eixo mediano horizontal ao mesmo. Ou seja, a soma encontrada

  é um número da forma 2X+Y.

Y  é (1+1)+(2+2)+(3+3)+(4+4)+(5+5)+(6+6). Ora, isto é, logicamente,

  igual  a  21x2.  Assim,  2X=6x21.  Então,  X=3x21.  Logo, para cada valor

  central, teremos a soma X+Y=5x21, ou seja, 7x5x21 + 21, este último 21 da

  própria  soma  da  parte central. Portanto, a resposta é 36x21 = 720+36 =

  756.



=
Instruções para entrar na lista, sair da lista e usar a lista em
http://www.mat.puc-rio.br/~nicolau/olimp/obm-l.html
=


[obm-l] Livrarias indicadas pela SBM

2003-11-07 Por tôpico JoaoCarlos_Junior
Na última página do livro da SBM: Olimpíadas Brasileiras de Matemática: 1ª
a 8ª, há indicação de duas livrarias, nas quais os livros constantes na
bibliografia podem ser adquiridos.
  Porém, nenhuma das duas, agora, trabalha com tais obras. Há, então,
  outra livraria no Brasil, em que se pode adquiri-los?

  ATT. João Carlos


=
Instruções para entrar na lista, sair da lista e usar a lista em
http://www.mat.puc-rio.br/~nicolau/olimp/obm-l.html
=


Re: [obm-l] Prova do IME

2003-11-06 Por tôpico JoaoCarlos_Junior

Uma solução resumida do que enviei é:

  H=4h. S=24s. Logo, a resposta é: 1/(4*24-1) = 1/95.


   
 
  "leonardo mattos"
 
  <[EMAIL PROTECTED]Para: [EMAIL PROTECTED]
  
  om>   cc:
 
  Enviado Por:  Assunto:  Re: [obm-l] Prova do IME 
 
  [EMAIL PROTECTED]

  .puc-rio.br  
 
   
 
   
 
  05/11/2003 17:52 
 
  Favor responder a
 
  obm-l
 
   
 
   
 




Como ele nao especifica quem divide quem a razao tambem poderia ser 95...


>From: [EMAIL PROTECTED]
>Reply-To: [EMAIL PROTECTED]
>To: [EMAIL PROTECTED]
>Subject: Re: [obm-l] Prova do IME
>Date: Wed, 5 Nov 2003 16:58:55 -0400
>
>
>Eu encontrei!
>   A  pirâmide  menor, cuja base é B, o médio de AB e o médio de BC,
>tem
>
>   altura  igual  a h/4. Pois, ela é "levantada" em um quarto de OB.

>A
>
>   área da base dessa pirâmide é 1/4 * área do triângulo ABC.
>
>   A  pirâmide cuja base é o hexágono tem área da base igual a 6*área
>do
>
>   triângulo  ABC.  Pois a área do triângulo DOC é igual a do ABC, já que
>se
>
>   transladando  o  vértice D para E, DE//OC, tem-se triângulo congruente
>ao
>
>   ABC.
>
>   Fazendo-se  a  diferença  entre  os  dois volumes calculados acima,

>e
>
>   posteriormente, dividindo-se tal diferença por aquele, achar-se-á a
>razão
>
>   (1/16)/(6-(1/16))=1/95.
>
>   Como sou café-com-leite, vou tentar outras. Isto, se tiver tempo.
>
>
>
>   ATT. João.
>
>
>
>
>
>
>
>
>   "Paulo Santa Rita"
>   <[EMAIL PROTECTED]>   Para:
>[EMAIL PROTECTED]
>   Enviado Por:  cc:
>   [EMAIL PROTECTED]Assunto:  Re: [obm-l]

>Prova do IME
>   .puc-rio.br
>
>
>   05/11/2003 14:55
>   Favor responder a
>   obm-l
>
>
>
>
>
>
>ola Pessoal !
>
>Alguem encontrou uma forma nao-GPI de fazer a questao tres ?
>
>Nao vou fazer, pois, pela regra que enunciei estou proibido de fazer isso
>(
>e o Claudio tambem ) mas vou falar duas coisas :
>
>PRIMEIRO - Voces, sem duvida, conhecem aquela formula que - sendo dado
tres
>
>pontos nao alinhados no plano cartesiano - nos permite encontrar a area do
>triangulo formado pelos tres pontos. A formula tem uma cara assim :
>
>Area do triangulo = (1/2)*DET, DET e o determinante da matriz formada
pelos
>
>tres pontos que representam os vertices do triangulo. Se voces nao sabem,
>existe o analogo desta formula para a dimentao 3, isto e, sendo dados 4
>pontos em R^3 nao coplanares, existe uma formula ( derivada por Lagrange )
>que calcula o volume da piramide. Essa formula e assim :
>
>Volume = (1/3!)*DET, onde DET e o determinante da matriz formada pelos 4
>pontos que representam a piramide.
>
>Entao, basta colocar a piramide regular no R^3 e determinar as coordenadas
>dos vertices do pequeno solido e, a seguir, aplicar a formula.
>
>
>SEGUNDO : Tem uma regrinha que diz, mais ou menos, o seguinte : INTEGRAL
de
>
>area E VOLUME, isto e, se pudermos exprimir uma area variavel em funcao de
>sua distancia a um determinado ponto, ao integrarmos, obteremos o volume.
>Ora, a area de sucessivas secoes sobre o pequeno solido e facilmente
>calculavel em funcao da distancia ao vertice. A integracao vai dar o
>volume.
>
>Quem faz a questao 4 de forma nao-GPI ?
>
>Bom, e com prazer que participo, mas foi necessario fazer um esforço pra
>estar aqui com voces neste momento um  tanto dificil, pois estou bastante
>atarefado. Eu vou ficar por aqui. O imbecil nao esta pertubando mais ( se
a
>
>nossa lista fosse patrimonio publico, caberia denuncia aqui no MPF e a
>Policia Federal seria acionada pra prende-lo ) e deu pra mostrar que pra
>

Re: [obm-l] Prova do IME

2003-11-05 Por tôpico JoaoCarlos_Junior

Eu encontrei!
  A  pirâmide  menor, cuja base é B, o médio de AB e o médio de BC, tem

  altura  igual  a h/4. Pois, ela é "levantada" em um quarto de OB.   A

  área da base dessa pirâmide é 1/4 * área do triângulo ABC.

  A  pirâmide cuja base é o hexágono tem área da base igual a 6*área do

  triângulo  ABC.  Pois a área do triângulo DOC é igual a do ABC, já que se

  transladando  o  vértice D para E, DE//OC, tem-se triângulo congruente ao

  ABC.

  Fazendo-se  a  diferença  entre  os  dois volumes calculados acima, e

  posteriormente, dividindo-se tal diferença por aquele, achar-se-á a razão

  (1/16)/(6-(1/16))=1/95.

  Como sou café-com-leite, vou tentar outras. Isto, se tiver tempo.



  ATT. João.







   
 
  "Paulo Santa Rita"   
 
  <[EMAIL PROTECTED]>   Para: [EMAIL PROTECTED]
  
  Enviado Por:  cc:
 
  [EMAIL PROTECTED]Assunto:  Re: [obm-l] Prova do IME  

  .puc-rio.br  
 
   
 
   
 
  05/11/2003 14:55 
 
  Favor responder a
 
  obm-l
 
   
 
   
 




ola Pessoal !

Alguem encontrou uma forma nao-GPI de fazer a questao tres ?

Nao vou fazer, pois, pela regra que enunciei estou proibido de fazer isso
(
e o Claudio tambem ) mas vou falar duas coisas :

PRIMEIRO - Voces, sem duvida, conhecem aquela formula que - sendo dado tres

pontos nao alinhados no plano cartesiano - nos permite encontrar a area do
triangulo formado pelos tres pontos. A formula tem uma cara assim :

Area do triangulo = (1/2)*DET, DET e o determinante da matriz formada pelos

tres pontos que representam os vertices do triangulo. Se voces nao sabem,
existe o analogo desta formula para a dimentao 3, isto e, sendo dados 4
pontos em R^3 nao coplanares, existe uma formula ( derivada por Lagrange )
que calcula o volume da piramide. Essa formula e assim :

Volume = (1/3!)*DET, onde DET e o determinante da matriz formada pelos 4
pontos que representam a piramide.

Entao, basta colocar a piramide regular no R^3 e determinar as coordenadas
dos vertices do pequeno solido e, a seguir, aplicar a formula.


SEGUNDO : Tem uma regrinha que diz, mais ou menos, o seguinte : INTEGRAL de

area E VOLUME, isto e, se pudermos exprimir uma area variavel em funcao de
sua distancia a um determinado ponto, ao integrarmos, obteremos o volume.
Ora, a area de sucessivas secoes sobre o pequeno solido e facilmente
calculavel em funcao da distancia ao vertice. A integracao vai dar o
volume.

Quem faz a questao 4 de forma nao-GPI ?

Bom, e com prazer que participo, mas foi necessario fazer um esforço pra
estar aqui com voces neste momento um  tanto dificil, pois estou bastante
atarefado. Eu vou ficar por aqui. O imbecil nao esta pertubando mais ( se a

nossa lista fosse patrimonio publico, caberia denuncia aqui no MPF e a
Policia Federal seria acionada pra prende-lo ) e deu pra mostrar que pra
cada questao IME e possivel encontrar facilmente um montao de maneiras de
faze-las. E so ter serenidade e pensar. As solucoes GPI sao muito boas,
talvez as melhores.

Com os melhores votos de
Paz profunda, sou

Paulo Santa Rita
4,1651,051103

>From: "Paulo Santa Rita" <[EMAIL PROTECTED]>
>Reply-To: [EMAIL PROTECTED]
>To: [EMAIL PROTECTED]
>Subject: Re: [obm-l] Prova do IME
>Date: Wed, 05 Nov 2003 17:30:47 +
>MIME-Version: 1.0
>X-Originating-IP: [200.216.62.82]
>X-Originating-Email: [EMAIL PROTECTED]
>Received: from mc8-f18.hotmail.com ([65.54.253.154]) by mc8-s3.hotmail.com

>with Microsoft SMTPSVC(5.0.2195.5600); Wed, 5 Nov 2003 09:38:24 -0800
>Received: from sucuri.mat.puc-rio.br ([139.82.27.7]) by
mc8-f18.hotmail.com
>with Microsoft SMTPSVC(5.0.2195.5600); Wed, 5 Nov 2003 09:33:01 -0800
>Received: (from [EMAIL PROTECTED])by sucuri.mat.puc-rio.br (8.9.3/8.9.3)
>id PAA10632for obm-l-MTTP; Wed, 5 Nov 2003 15:31:29 -0200
>Received: from hotmail.co

Re: [obm-l] hahahahahahha

2003-11-05 Por tôpico JoaoCarlos_Junior

  Caro amigo,


  Observa o objeto a partir do qual retiras prazer. Julgas-te sadio?

  Corrigi-te rápido, para teu próprio bem.



  ATT. João.



   
 
  "Cláudio\(Prática\)" 
 
  <[EMAIL PROTECTED]Para: <[EMAIL PROTECTED]>  
  
  .com.br>  cc:
 
  Enviado Por:  Assunto:  [obm-l] hahahahahahha
 
  [EMAIL PROTECTED]

  .puc-rio.br  
 
   
 
   
 
  05/11/2003 00:19 
 
  Favor responder a
 
  obm-l
 
   
 
   
 




Essa lista me mata de rir.O Lalau não sabe resolver o problema sozinho
HAHAHAHHAAHAHAHAHAHAHHHAHHAHHA
LALAU IDIOTA,TAPADO
VCS Q ESCREVEM ESSAS BABOSEIRAS NESSA LISTA TB SÃO UNS RETARDADOS




=
Instruções para entrar na lista, sair da lista e usar a lista em
http://www.mat.puc-rio.br/~nicolau/olimp/obm-l.html
=


[obm-l] OFF TOPIC: a matemática fora dos grandes centros

2003-10-22 Por tôpico JoaoCarlos_Junior
Professores e interessados,


Como tornar mais eficaz a busca por novos talentos brasileiros para

  matemática, não residentes em São Paulo, Rio de Janeiro ou Fortaleza?

  Acredito  que a proximidade com os sensacionais professores do IMPA e

  de  outras  organizações,  localizadas nas três cidades já mencionadas, é

  dentre   outras  causas,  de  grande  relevância,  para  essa  descoberta

  circunscrever-se, em regra, a essas localidades.

  Minha  sugestão  é  que  com  a  confecção de filmes das aulas desses

  professores  e  com  um  sistema de estudo à distância, tornar-se-ia mais

  eficaz  essa  busca por novos talentos. Assim, pergunto: o que está sendo

  feito neste sentido?

  Desculpem-me  o  assunto OFF TOPIC, mas essa preocupação é natural de

  quem,  sem muitas condições, está envolvido com matemática em Mato Grosso

  do Sul.



  Um abraço, João.



=
Instruções para entrar na lista, sair da lista e usar a lista em
http://www.mat.puc-rio.br/~nicolau/olimp/obm-l.html
=


Re: [obm-l] Geometria Espacial - Pirâmides (Mr. Crowley)

2003-10-01 Por tôpico JoaoCarlos_Junior

Caro amigo,


  Desenhe  sua  pirâmide.  Trace  por  A  reta  paralela  a  BD.  Não é

  necessário  dizer  que  tal  reta está contida no plano da base, mas pode

  haver  quem  disso não saiba. Seja B´e C´a interseção dessa reta com BC e

  DC, respectivamente.

  Ora,  por Menelaus no triângulo VBC, secante B´QM, em que M é o médio

  de  VC  e Q, a intersecção de B´M com VB, ver-se-á que QB é metade de VQ.

  Por  simetria, ND é metade de NV, em que N é a intersecção de D´M com VD.

  Logo,  NQ,  paralela  a  BD,  tem  por  medida  2/3 de BD, ou seja, (2/3)

  *L*(sqrt2)).

  O  triângulo AVC é eqüilátero. Logo, AM é altura de tal triângulo, e,

  portanto, mede L*sqrt(6)/2.

  Como AM é altura do triângulo isósceles B´D´M e NQ é paralela a B´D´,

  então  AM é perpendicular a NQ. Logo, a área desejada é ½*NQ*AM, ou seja,

  sqrt(3)/3* L*L.

  Salvo  correções necessárias de professores ou outros, que são sempre

  bem-vindas, esta é a resposta.

  Um abraço, João Carlos.





   
  
  "paraisodovestibulando"  
  
  
  bol.com.br>cc:   
  
  Enviado Por:   Assunto:  [obm-l] Geometria 
Espacial - Pirâmides (Mr.   
  [EMAIL PROTECTED] Crowley)   

  puc-rio.br   
  
   
  
   
  
  01/10/2003 00:22 
  
  Favor responder a obm-l  
  
   
  
   
  




Olá Pessoal da Lista,

Gostaria de deixar meus agradecimentos ao Cláudio e ao
Leandro pelas ajudas (valew mesmo).


Me ajudem neste exercício:

Seja uma pirâmide regular de vértice V e base
quadrangular ABCD. O lado da base da pirâmide mede L e
a aresta lateral L.sqrt(2). Corta-se a essa pirâmide
por um plano que contém o vértice A, é paralelo à reta
BD, e contém o ponto médio da aresta VC. Calcule a área
da seção determinada pela interseção do plano com a
pirâmide.



Grato

Mr. Crowley

__
Acabe com aquelas janelinhas que pulam na sua tela.
AntiPop-up UOL - É grátis!
http://antipopup.uol.com.br/


=
Instruções para entrar na lista, sair da lista e usar a lista em
http://www.mat.puc-rio.br/~nicolau/olimp/obm-l.html
=





=
Instruções para entrar na lista, sair da lista e usar a lista em
http://www.mat.puc-rio.br/~nicolau/olimp/obm-l.html
=


[obm-l] teste logico

2003-08-14 Por tôpico JoaoCarlos_Junior

Pediram-me  para  resolver  o exercício que segue, porém, ainda não

  consegui resolvê-lo.

As  regras com relação a mãe e filhos e pai e filhas estão erradas,

  acredito. Parece-me que o correto é:

A  mãe não pode ficar sozinha com os filhos, caso dentre esses haja

  pelo menos 01 menino. Analogamente, para o pai e as filhas.



  Um abraço, João.


Quero ver ser vcs conseguem resolver essa eu consegui!!

Esse é um TESTE de QI que é aplicado em DINÂMICA DE GRUPO em EMPRESAS
JAPONESAS !!

Consiste em VOCÊ atravessar todos para o OUTRO LADO de um RIO, sendo que 2
de CADA VEZ.
As regras são as seguintes:
. Somente O PAI, A MÃE e o POLICIAL sabem GUIAR O BARCO;
. A mãe não pode ficar sozinha com OS FILHOS;
. O pai não pode ficar sozinho com AS FILHAS;
. O criminoso não pode ficar sozinho com NENHUM integrante da FAMÍLIA;
. O barco só pode transportar 2 pessoas por vez.

PARA COMEÇAR clique no CIRCULO AZUL !
E para andar com os BONECOS basta CLICAR em  cima DELES.
Para MOVER o BARCO basta clicar na ALAVANCA !!!


http://smallcampus.net/html/maths_games/2001-05-03/riverIQGame.swf

http://smallcampus.net/html/maths_games/2001-05-03/riverIQGame.swf



=
Instruções para entrar na lista, sair da lista e usar a lista em
http://www.mat.puc-rio.br/~nicolau/olimp/obm-l.html
=


[obm-l] Re: teste logico

2003-08-06 Por tôpico JoaoCarlos_Junior


Não precisa mais, pois que, consegui resolver o exercício. Não é dificil!


Obrigado! João.






   
  
  ANGELA GIOVANNA  
  
  <[EMAIL PROTECTED]Para: [EMAIL PROTECTED]
 
  hoo.com.br>cc:   
  
 Assunto:  teste logico
  
  01/08/2003 16:46 
  
   
  
   
  






Note: forwarded message attached.



Conheça o novo Cadê? - Mais rápido, mais fácil e mais preciso.
Toda a web, 42 milhões de páginas brasileiras e nova busca por imagens!
- Mensagem de "karla.branquinho" <[EMAIL PROTECTED]> em Fri,
1 Aug 2003 16:43:49 -0300 -
   
Para: [EMAIL PROTECTED] 
   
 Assunto: teste logico 
   




__
Acabe com aquelas janelinhas que pulam na sua tela.
AntiPop-up UOL - É grátis!
http://antipopup.uol.com.br/
Quero ver ser vcs conseguem resolver essa eu consegui!!

Esse é um TESTE de QI que é aplicado em DINÂMICA DE GRUPO em EMPRESAS
JAPONESAS !!

Consiste em VOCÊ atravessar todos para o OUTRO LADO de um RIO, sendo que 2
de CADA VEZ.
As regras são as seguintes:
. Somente O PAI, A MÃE e o POLICIAL sabem GUIAR O BARCO;
. A mãe não pode ficar sozinha com OS FILHOS;
. O pai não pode ficar sozinho com AS FILHAS;
. O criminoso não pode ficar sozinho com NENHUM integrante da FAMÍLIA;
. O barco só pode transportar 2 pessoas por vez.

PARA COMEÇAR clique no CIRCULO AZUL !
E para andar com os BONECOS basta CLICAR em  cima DELES.
Para MOVER o BARCO basta clicar na ALAVANCA !!!


http://smallcampus.net/html/maths_games/2001-05-03/riverIQGame.swf
Quero ver ser vcs conseguem resolver essa eu consegui!!

Esse é um TESTE de QI que é aplicado em DINÂMICA DE GRUPO em EMPRESAS
JAPONESAS !!

Consiste em VOCÊ atravessar todos para o OUTRO LADO de um RIO, sendo que 2
de CADA VEZ.


As regras são as seguintes:
. Somente O PAI, A MÃE e o POLICIAL sabem GUIAR O BARCO;
. A mãe não pode ficar sozinha com OS FILHOS;
. O pai não pode ficar sozinho com AS FILHAS;
. O criminoso não pode ficar sozinho com NENHUM integrante da FAMÍLIA;
. O barco só pode transportar 2 pessoas por vez.


PARA COMEÇAR clique no CIRCULO AZUL !
E para andar com os BONECOS basta CLICAR em  cima DELES.
Para MOVER o BARCO basta clicar na ALAVANCA !!!



http://smallcampus.net/html/maths_games/2001-05-03/riverIQGame.swf




=
Instruções para entrar na lista, sair da lista e usar a lista em
http://www.mat.puc-rio.br/~nicolau/olimp/obm-l.html
=


Re: [obm-l] Fotos da IMO

2003-08-01 Por tôpico JoaoCarlos_Junior

Professor Nicolau ou outros,

  Seria possível a identificação dos brasileiros nas fotos a nós
enviadas?

Um abraço, João Carlos.



   
 
  "Nicolau C. Saldanha"
 
  <[EMAIL PROTECTED]Para: [EMAIL PROTECTED],   
 
  c-rio.br>  [EMAIL PROTECTED] 
   
  Enviado Por:  cc:
 
  [EMAIL PROTECTED]Assunto:  [obm-l] Fotos da IMO  

  .puc-rio.br  
 
   
 
   
 
  31/07/2003 19:30 
 
  Favor responder a
 
  obm-l
 
   
 
   
 




O Elio mandou umas fotos da IMO:

http://www.mat.puc-rio.br/~nicolau/olimp/jf/

[]s, N.
=
Instruções para entrar na lista, sair da lista e usar a lista em
http://www.mat.puc-rio.br/~nicolau/olimp/obm-l.html
=





=
Instruções para entrar na lista, sair da lista e usar a lista em
http://www.mat.puc-rio.br/~nicolau/olimp/obm-l.html
=


Re: [obm-l] Re: Como os Matemáticos Complicam II

2003-07-11 Por tôpico JoaoCarlos_Junior

J. Paulo,


  Como  colocas, és quem está a enfrentar o bicho papão. Porém, o bicho

  papão está em ti próprio. Percebes isso? Tua polêmica inteira, inadequada

  nesta  lista, resume-se a: para que serve a matemática? E Principalmente,

  a abstrata?

  Isso  não  é  para  ser  discutido  aqui, por isso, serei breve e não

  escreverei novamente sobre o assunto.

  A  pergunta  acima fiz-me por um período de uns três meses, há alguns

  anos atrás. Era pergunta que me fazia naturalmente e com muita vontade de

  ter  a  resposta.  Cada fibra de meu coração desejava-a. Essa veio ao fim

  desse  prazo.  Foi  a  mais  completa  resposta que já tive, pois além de

  verdadeira,  veio  com  um grau elevado de satisfação como nenhuma outra.

  Completou-me,  satisfez-me,  inteiramente.  A  experiência  em  si  mesma

  exultou-me. Posteriormente, soube de um psicólogo amigo que esta forma de

  questionamento  é  técnica  de  autodescobrimento, cujas respostas vêm do

  subconsciente. Técnica, a qual eu não conhecia. Vivi-a sem conhecê-la. No

  fundo acho que ainda não a conheço. Não tenho consciência de como a mesma

  se processa. Compreende?

  A resposta é a matemática serve para o próprio ser humano em si, isto

  é,  coloca-o  como  centro  de  desenvolvimento, e desenvolve-o natural e

  eficazmente,  pois  que  é  ela  quem  o  auxilia a organizar, discernir,

  separar, concluir, inferir, deduzir, criar, etc., todo e qualquer ramo do

  conhecimento humano.

  Isto não implica que se deva comparar pessoas distintas. O que digo é

  um   médico   seria   melhor  médico,  se  também  estudasse  matemática.

  Comparando-o  consigo próprio, isto é, seu passado, sem estudo da mesma e

  seu futuro, com tal estudo. É claro que esse estudo é longo e contínuo. A

  percepção  da  melhora  é sutil na vida desse médico, e para ele próprio,

  mas há efetivamente evolução.

  Enfim,  a  matemática em si auxilia a evolução intelectual e racional

  do  próprio  homem.  Claro  que  há  outros  ramos nos quais o homem deve

  desenvolver-se,   como  no  amor,  na  humildade,  etc.  Porém,  no  lado

  intelectual, a matemática é imbatível.

  Um forte abraço, João.



=
Instruções para entrar na lista, sair da lista e usar a lista em
http://www.mat.puc-rio.br/~nicolau/olimp/obm-l.html
=


Re: [obm-l] Equipe 44-IMO

2003-06-05 Por tôpico JoaoCarlos_Junior


Desejo a todos os matemáticos brasileiros muito positivismo.

Que seus sonhos sejam conquistados!

Que  a  medalha  dourada  esteja estampada no peito de todos vocês,

  refletindo  em  seus  semblantes  a  pura alegria, vitória e conquista de

  representar um povo tão lindo e livre.

A  você,  Professor Nicolau, percebo seu esforço de elevar o ensino

  da  matemática  em seu próprio país. Isto é missão das grandes almas, não

  tenha  dúvida disso. Sei do retorno emotivo e motivador, da esperança que

  tens  de  vê-los  brilhar  perante  essa  árdua tarefa, porém, gloriosa e

  inebriante  a  que  se empenham tão jovens alunos, que no fundo, poderiam

  ser teus filhos.

  Muita paz e luz ao coração de todos!



  De um brasileiro, desconhecido de vocês, João.





=
Instruções para entrar na lista, sair da lista e usar a lista em
http://www.mat.puc-rio.br/~nicolau/olimp/obm-l.html
=


Re: [obm-l] Treinamento no Rio

2003-03-10 Por tôpico JoaoCarlos_Junior

É com certa tristeza que leio este e-mail, pois que não moro no Rio de
Janeiro, assim não tenho possibilidade de assistir a tão belas aulas de tão
dignos professores.
Será  que  não  há  possibilidade  de que as mesmas sejam filmadas,

  transmitidas   ao   vivo  pela  internet  ou  qualquer  outra  forma  que

  possibilitassem  alunos  de  outros estados embeberem-se das mesmas? Será

  que não há uma forma, mesmo que necessitemos custeá-las de algum meio?



  Um forte abraço a todos, João Carlos.





   
 
  "Igor Castro"
 
  <[EMAIL PROTECTED]>   Para: <[EMAIL PROTECTED]>  
  
  Enviado Por:  cc:
 
  [EMAIL PROTECTED]Assunto:  Re: [obm-l] Treinamento no 
Rio
  .puc-rio.br  
 
   
 
   
 
  09/03/2003 01:38 
 
  Favor responder a
 
  obm-l
 
   
 
   
 




Está definido que a reunião ocorrera somente neste horario e nesse dia
mesmo? Por que creio que muitos alunos interessados, como eu, não poderão
ir
frequentemente devido ao colégio e curso que muitos fazem. Mas também não
vejo um horario/dia que facilite a todos rapidamente. Acho que seria bem
legal mesmo que se formasse um "grupo" de estudo para as olímpiadas no
RJ(como tem no CE), por isso, acho que todos os esforços deveriam ser
feitos
para atrair o maior número possível de pessoas(alunos e profs). Bem, é só
uma colocação do meu ponto de vista, espero que a idéia possa ser
amadurecida e que se encontre o melhor para todos.
Abraços,
Igor...
- Original Message -
From: "Carlos Gustavo Tamm de Araujo Moreira" <[EMAIL PROTECTED]>
To: <[EMAIL PROTECTED]>
Sent: Friday, March 07, 2003 1:25 PM
Subject: Re: [obm-l] Treinamento no Rio


>Caros colegas,
>Estou escrevendo para lembrar da reuniao de segunda...
>Abracos,
> Gugu
>
> >
> >   Caros colegas,
> >   Na primeira segunda-feira depois do carnaval (10/2), no IMPA, as
14:00
> >horas comecam as reunioes semanais de treinamento olimpico abertas ao
> >publico, que visam entre outras coisas treinar para a IMO. Somos
responsaveis
> >por estas reunioes eu e o Luciano, mas deveremos tambem ter aulas de
outros
> >ilustres colegas (oi Nicolau! oi Okakamo! oi Morgado! oi Wagner!). Estao
> >todos convidados (especialmente o pessoal do Rio...)!
> >   Abracos,
> >   Gugu
> >
> >
=
> >Instruções para entrar na lista, sair da lista e usar a lista em
> >http://www.mat.puc-rio.br/~nicolau/olimp/obm-l.html
> >O administrador desta lista é <[EMAIL PROTECTED]>
> >
=
>
> =
> Instruções para entrar na lista, sair da lista e usar a lista em
> http://www.mat.puc-rio.br/~nicolau/olimp/obm-l.html
> O administrador desta lista é <[EMAIL PROTECTED]>
> =
>


=
Instruções para entrar na lista, sair da lista e usar a lista em
http://www.mat.puc-rio.br/~nicolau/olimp/obm-l.html
O administrador desta lista é <[EMAIL PROTECTED]>
=




=
Instruções para entrar na lista, sair da lista e usar a lista em
http://www.mat.puc-rio.br/~nicolau/olimp/obm-l.html
O administrador desta lista é <[EMAIL PROTECTED]>
=


[obm-l] Determinante - MIR

2003-03-06 Por tôpico JoaoCarlos_Junior
Calcular o determinante:

  |   1½   1/3  ...1/n  |
  |   ½ 1/3¼   ...1/(n+1)   |
  | .  |
  |   1/n   1/(n+1)   1/(n+2)   ...   1/(2n-1)|

  Resposta: [1!2! ... (n-1)]^3 / [n!(n+1)! ... 2(n-1)!]

  Fonte:  Álgebra Superior ? D. Faddieev, I. Sominski ? Ed. MIR ? Moscu. p.
  53.

  Um forte abraço a todos, João Carlos.

=
Instruções para entrar na lista, sair da lista e usar a lista em
http://www.mat.puc-rio.br/~nicolau/olimp/obm-l.html
O administrador desta lista é <[EMAIL PROTECTED]>
=


[obm-l] Observar passos para progredir

2003-02-28 Por tôpico JoaoCarlos_Junior
Queridos professores e amigos,


  É  fato  que homens seguem passos de outros homens, que alunos seguem

  os  de  seus mestres até aonde podem, muitas vezes, ir. Raras vezes, pode

  ocorrer  que alunos igualem-se a seus mestres, para os substituir, já que

  a vida flui, neste planeta.

Fui aluno do Ralph, em 1989, na turma IME/ITA do Impacto.  Cursei o

  IME  de  90  a  94.  Fiz  mestrado  no  mesmo  Instituto  de 98 a 99. Sem

  idolatria,  afirmo  admirar  os passos de Pitombeira de Carvalho, Nicolau

  Corsão Saldanha, Augusto César de Oliveira Morgado, Ralph Costa Teixeira,

  João  Bosco, Paulo Cezar Pinto Carvalho, Pedro Fernandez, Eduardo Wagner,

  Élon Lages Lima, João Paulo Q. Carneiro, Carlos Gustavo T. de A. Moreira.

  Graças  a  Deus! A Internet permite-me ser aluno, à distância, dos demais

  citados, os quais não conheci pessoalmente.

  Também,  vale  a  penas destacar as contribuições do melhores alunos,

  como,  por  exemplo,  Cláudio  ([EMAIL PROTECTED]), que mui

  contribui  para desenvolvimento dos alunos mais retardatários, dentro dos

  quais, incluo-me, com naturalidade.

Nesta  seqüência  de  observar passos para o progresso, nós eternos

  alunos  gostaríamos  de  saber,  se  vocês  professores conheceram outros

  mestres   por   vós  admiráveis  e  que  problemas  resolveram  para  vos

  impressionar? Já que a lista é de resolução de problemas.

  OBS:  Provavelmente, esqueci-me de muitos nomes, tanto de professores

  quanto  de  alunos.  Justifico-me  pela  distância  de  anos  luz  em que

  encontro-me  do  ponto onde vocês estão, juntamente, com outros. Também a

  ordem supracitada de professores foi feita por sorteio.



  Um forte abraço a todos, João Carlos.

=
Instruções para entrar na lista, sair da lista e usar a lista em
http://www.mat.puc-rio.br/~nicolau/olimp/obm-l.html
O administrador desta lista é <[EMAIL PROTECTED]>
=


[obm-l] Determinantes

2003-02-06 Por tôpico JoaoCarlos_Junior
Queridos amigos, como resolver as questões que seguem abaixo?

  1)  F(x) = x(x-1)(x-2)...(x-n+1). Calcular os determinantes:
  a) |F(0)  F(1)  F(2) ...  F(n)  |
   |F(1)  F(2)  F(3) ...  F(n+1)|
   |..  |
   |F(n) F(n+1) F(n+2)... F(2n) |

  b)|F(a) F´(a)  F"(a) ... F^(n)(a)  |
   |F´(a)F"(a)  F´´´(a) ...   F^(n+1)(a)|
   |..  |
   |F^(n)(a) F^(n+1)(a) F^(n+2)(a)... F^(2n)(a) |

  2)  Os números 204, 527 e 255 são divisíveis por 17. Demonstrar que
   | 2 0 4 |
   | 5 2 7 |
   | 2 5 5 |

é divisível por 17.


  Fonte: Problemas de Álgebra Superior ? D. Faddieev, I. Sominski ?
  Editorial MIR ? Moscou.
  ATT. João Carlos.


=
Instruções para entrar na lista, sair da lista e usar a lista em
http://www.mat.puc-rio.br/~nicolau/olimp/obm-l.html
O administrador desta lista é <[EMAIL PROTECTED]>
=



Re: [obm-l] Re: [obm-l] Determinantes ePermutações pares e ímpares

2003-02-05 Por tôpico JoaoCarlos_Junior
   
 
  "Cláudio \(Prática\)"
 
 
 
  ora.com.br>   cc:
 
  Enviado Por:  Assunto:  [obm-l] Re: [obm-l] 
Determinantes e Permutações   
  [EMAIL PROTECTED] pares e ímpares   
 
  .puc-rio.br  
 
   
 
   
 
  04/02/2003 06:12 
 
  Favor responder a
 
  obm-l
 
   
 
   
 






  Querido Cláudio,



  Obrigado! Com sinceridade, obrigado! O conhecimento real, presente, é

  o  que  possuímos,  fora isto, estamos no passado. Por isso, agradeço sua

  colaboração, com a qual atualizo-me e avanço.

  Cláudio,  não sei a definição de permutações pares e ímpares, não sei

  quando o sinal ? sgn(p) - será positivo ou negativo.

Desta  forma,  gostaria  de receber mais de suas belas explicações.

  Desde já, muito grato, João Carlos.




Caro João Carlos:

A fórmula geral para o determinante de uma matriz A (n x n) é a seguinte:

det(A) = SOMATÓRIO   sgn(p) * A(1,p(1)) * A(2,p(2)) * ... * A(n,p(n))
  p  em  Sn

onde A(i,j) é o elemento da linha i e coluna j, sgn(p) = sinal da
permutação
"p" (+1 se p é par, -1 se p
é ímopar) e onde a soma é tomada sobre cada permutação p dos números 1, 2,
..., n  (o conjunto de todas estas permutações é comumente denominado Sn)
ou
seja, é uma soma de n! termos, cada um deles igual ao produto de n
elementos
da matriz.

Assim, para n >= 4 esta fórmula, apesar de correta (é, de fato, a definição
de determinante) é muito trabalhosa de se aplicar. No entanto, existem
alguns teoremas sobre determinantes - tais como expansão de Laplace ou
sobre
o efeito de operações elementares com linhas e colunas - que permitem que
você reduza o problema ao cálculo de determinantes de ordem menor.

O que deve estar acontecendo é que, com n >= 4, o número de termos é >= 24
e
talvez você esteja esquecendo algum termo ou trocando algum sinal.

Espero que isso ajude.

Um abraço,
Claudio.


- Original Message -
From: <[EMAIL PROTECTED]>
To: <[EMAIL PROTECTED]>
Sent: Monday, February 03, 2003 4:08 PM
Subject: [obm-l] Determinantes e Permutações pares e ímpares


No volume 3, A Matemática do Ensino Médio da SBM, p. 137, há regra de
cálculo determinantes por meio de permutações pares e ímpares. Porém, não
estou conseguindo aplicá-la para matrizes quadradas de ordem maior ou igual
a 4. Expliquem-me.


  ATT. João Carlos

=
Instruções para entrar na lista, sair da lista e usar a lista em
http://www.mat.puc-rio.br/~nicolau/olimp/obm-l.html
O administrador desta lista é <[EMAIL PROTECTED]>
=

=
Instruções para entrar na lista, sair da lista e usar a lista em
http://www.mat.puc-rio.br/~nicolau/olimp/obm-l.html
O administrador desta lista é <[EMAIL PROTECTED]>
=




=
Instruções para entrar na lista, sair da lista e usar a lista em
http://www.mat.puc-rio.br/~nicolau/olimp/obm-l.html
O administrador desta lista é <[EMAIL PROTECTED]>
=



[obm-l] Matrizes Simétricas e Inversíveis

2003-02-03 Por tôpico JoaoCarlos_Junior
Sejam as matrizes A e P inversíveis. Seja B igual a P^-1 A P. Há forma de
provar, sem contra-exemplo, a falsidade: se A é simétrica, então B também o
é.

  ATT. João Carlos

=
Instruções para entrar na lista, sair da lista e usar a lista em
http://www.mat.puc-rio.br/~nicolau/olimp/obm-l.html
O administrador desta lista é <[EMAIL PROTECTED]>
=



[obm-l] Determinantes e Permutações pares e ímpares

2003-02-03 Por tôpico JoaoCarlos_Junior
No volume 3, A Matemática do Ensino Médio da SBM, p. 137, há regra de
cálculo determinantes por meio de permutações pares e ímpares. Porém, não
estou conseguindo aplicá-la para matrizes quadradas de ordem maior ou igual
a 4. Expliquem-me.


  ATT. João Carlos

=
Instruções para entrar na lista, sair da lista e usar a lista em
http://www.mat.puc-rio.br/~nicolau/olimp/obm-l.html
O administrador desta lista é <[EMAIL PROTECTED]>
=



[obm-l] Elogio a E. Wagner e Sugestão de Livro

2003-01-29 Por tôpico JoaoCarlos_Junior

Retorno agora de férias. 
Querido E. Wagner, sei que és um dos escritores da mais linda e bela coleção de livros já editados do ensino médio intitulada: Coleção do Professor de Matemática.
Desta forma, fico bastante lisonjeado com sua manifestação. Como aqui neste site somente posso escrever de matemática, o elogio aqui escrito, curto, não reflete a integridade e intensidade de minha satisfação. Assim, em sua homenagem, aproveito a oportunidade para perguntar: 
Que livro(s) devo adquirir sobre Matrizes, Determinantes e Sistemas de Equações Lineares, no intuito de auxiliar jovens desejosos de ingressar no IME ou no ITA? 

ATT. João Carlos 






"Eduardo Wagner" <[EMAIL PROTECTED]>
Enviado Por: [EMAIL PROTECTED]
06/01/2003 00:06
Favor responder a obm-l

        
        Para:        [EMAIL PROTECTED]
        cc:        
        Assunto:        Re: [obm-l] Um tal de Cassilã        o


Prezado Joao Carlos:

Este belo problema foi proposto por Cramer (voce lembra
de sua famosa regra?) e resolvido em 1776 por Castillon,
um geometra e engenheiro italiano. Eles foram contemporaneos
mas quando Castillon conseguiu a solucao, Cramer ja havia
morrido 25 anos antes. Na verdade, Castillon chamava-se
Salvemini, nascido na cidade de Castiglione no centro-norte
da Italia, e dai vem o nome pelo qual ficou conhecido ate surgir
nesta lista como Cassilão.
Bem, vamos ao problema.

Imaginemos o problema resolvido.
Desenhe a seguinte figura:
Uma circunferencia e um triangulo DEF inscrito.
Assinale o ponto A sobre o prolongamento do lado FE, 
o ponto B sobre o prolongamento de DE e o ponto C 
sobre o prolongamento de DF. 
Suponha agora que a circunferencia e os pontos A, B e C 
sao dados. Nosso problema eh descobrir um vertice do
triangulo DEF.
Trace FG paralelo a BC (G sobre a circunferencia).
Trace a reta GE que encontra BC em H.
O angulo EHB eh igual a EGF que eh igual a EDF.
Logo, os triangulos BHE e BDC sao semelhantes e entao
BH/BD = BE/BC, ou BH = (BD.BE)/BC.
Trace BT tangente em T a circunferencia. Pela propriedade
da potencia, BD.BE = (BT)^2.
Assim, BH = ((BT^2))/BC. Como BT e BC sao conhecidos,
BH pode ser obtido por uma construcao elementar.
Uma vez determinado H sobre BC nosso problema passa a ser
determinar o ponto E na circunferencia de forma que as retas
EA e EH determinem na circunferencia  os pontos F e G, res-
pectivamente, tais que FG seja paralela a BC.
Trace FL paralela a AH (L sobre a circunferencia) e a reta LG
que encontra AH em M. Faca uma figura nova a partir deste 
ponto. Os triangulos MGH e EAT sao semelhantes e portanto
HM/HE = HG/HA ou HM = (HE.HG)/HA. Tracando a tangente HS
a circunferencia temos que HM = ((HS)^2))/AH e o ponto M 
pode ser obtido por outra construcao elementar.
Como os angulos LFG e AHB sao iguais, nosso problema passa
a ser: tracar por M uma secante MGL de forma que um angulo
inscrito LFG seja igual ao angulo conhecido formado pelas retas
AH e BC. Mais uma construcao elementar e o problema esta
resolvido.

PS. A palavra "elementar" que foi usada varias vezes, se refere
a cada uma das construcoes que permitiram obter os pontos
intermediarios H, M e L. O problema em si eh muito dificil.

Abracos,

E. Wagner.




--
From: [EMAIL PROTECTED]
To: [EMAIL PROTECTED]
Subject: [obm-l] Um tal de Cassilão
Date: Fri, Jan 3, 2003, 9:34 AM



Como resolver o seguinte problema: 

Dados um círculo de centro O e raio r e três pontos não colineares, todos externos ao círculo dado. Construir triângulo inscrito ao círculo, cujos prolongamentos dos lados passam pelos três pontos. 
Fonte: amigo de uns 50 anos. Dizendo ele, este         é o teorema de Cassilão. 

ATT. João Carlos 




[obm-l] Um tal de Cassilão

2003-01-03 Por tôpico JoaoCarlos_Junior

Como resolver o seguinte problema:

Dados um círculo de centro O e raio r e três pontos não colineares, todos externos ao círculo dado. Construir triângulo inscrito ao círculo, cujos prolongamentos dos lados passam pelos três pontos.
Fonte: amigo de uns 50 anos. Dizendo ele, este         é o teorema de Cassilão. 

ATT. João Carlos 


Re: [obm-l] Re: [obm-l] Princípio de Dirichlet

2003-01-02 Por tôpico JoaoCarlos_Junior

Querido Larryp, 

Muito obrigado pela resposta!

Com sinceridade, 
João Carlos. 






"larryp" <[EMAIL PROTECTED]>
Enviado Por: [EMAIL PROTECTED]
02/01/2003 22:42
Favor responder a obm-l

        
        Para:        <[EMAIL PROTECTED]>
        cc:        
        Assunto:        [obm-l] Re: [obm-l] Princípio de Dirichlet


Tome um número natural "n" qualquer. 
 
Considere os números 1, 11, 111, , 1,  e 111..11 (onde o último número é formado por (n+1) algarismos 1, e os restos que cada um destes números deixa quando dividido por n.
 
Existem n+1 números mas apenas n restos possíveis (0, 1, ..., n-1). Assim, pelo princípio de Dirichlet, têm de existir na lista acima dois números formados só por algarismos ´1´ que deixam o mesmo resto (suponhamos que o maior seja formado por "p" e o menor por "q" algarismos ´1´ ( p > q) ).
 
Subtraindo o menor do maior, você obtém um número da forma 11...1100..00, formado por "(p-q)" 1´s seguido de "q" zeros, o qual é divisível por "n" (estou usando o fato de que se "a" e "b" deixam o mesmo resto na divisão por "n" então "a-b" é divi´sível por "n".
 
- Original Message - 
From: [EMAIL PROTECTED] 
To: [EMAIL PROTECTED] 
Sent: Thursday, January 02, 2003 11:52 AM
Subject: [obm-l] Princípio de Dirichlet


Caros amigos, muita paz! Feliz ano novo a todos! 

Como resolver a seguinte questão referente a Dirichlet: 

Prove que todo número natural tem um múltiplo que se escreve, na base 10, apenas com os algarismos 0 e 1. 

Fonte: Análise Combinatória e Probabilidade. Coleção do Professor de matemática. Sociedade Brasileira de Matemática. 
Autores: 
        Augusto César de Oliveira Morgado 
        João Bosco Pitombeira de Carvalho 
        Paulo Cezar Pinto Carvalho 
        Pedro Fernadez 

ATT. João Carlos 



[obm-l] Princípio de Dirichlet

2003-01-02 Por tôpico JoaoCarlos_Junior

Caros amigos, muita paz! Feliz ano novo a todos!

Como resolver a seguinte questão referente a Dirichlet: 

Prove que todo número natural tem um múltiplo que se escreve, na base 10, apenas com os algarismos 0 e 1.

Fonte: Análise Combinatória e Probabilidade. Coleção do Professor de matemática. Sociedade Brasileira de Matemática.
Autores: 
        Augusto César de Oliveira Morgado
        João Bosco Pitombeira de Carvalho
        Paulo Cezar Pinto Carvalho
        Pedro Fernadez

ATT. João Carlos